family medicine part 2
A 33-year-old man presents to the office with complaints of penile itching and burning upon urination. He recently started having unprotected intercourse with a new partner. Milking of the urethra on physical examination expresses a mucopurulent discharge. Culture is obtained and pending. What pharmacologic therapy is appropriate for the patient's suspected condition?
Ceftriaxone 500 mg intramuscular for one dose and doxycycline 100 mg twice daily for 7 days dx: urethritis
A 35-year-old man presents with complaints of swelling and pain over his left knee for the past 2 days. He lays carpet for a living, and he states he has been on his knees more lately. The patient reports he had a steroid injection into his knee due to pain about 3 days ago. The patient has no chronic medical conditions, does not take any medications, and has no known allergies. Vital signs include BP 120/80 mm Hg, pulse 70 beats per minute, respiratory rate 16 breaths per minute, temperature 98.6°F, and O2 saturation of 98%. Physical exam reveals localized warmth, tenderness, fluctuant edema, and erythema localized over the patella. Passive joint range of motion is preserved. Fluid aspiration analysis shows WBC count of 3,500/microL and a Gram stain shows gram-positive cocci. Aerobic and anaerobic cultures are pending. Which of the following is the best therapy for this patient, given the suspected diagnosis?
Cephalexin dx: Septic bursitis is inflammation of a bursa olecranon, prepatellar, and infrapatellar bursae Risk factors include trauma, preceding noninfectious bursa inflammation such as from gout or rheumatoid arthritis, loss of skin integrity from underlying dermatologic conditions, recent joint aspiration or injection, male sex, and immunosuppression. It is most commonly caused by Staphylococcus aureus. The gold standard of diagnosis is bursa fluid culture Patients with systemic signs of toxicity (fever greater than 100.5°F and tachycardia), bursitis overlying prosthetic joint or graft, rapid progression of erythema, progression after 24 hours of oral antibiotics, increased risk of methicillin-resistant Staphylococcus aureus (MRSA), prior history of MRSA, or inability to tolerate oral therapy should be admitted for IV antibiotics and surgical consultation. Risk factors for MRSA include hospitalization in the last 60 days, residence in a long-term care facility, surgery in the last 60 days, hemodialysis, and HIV infection. MRSA infection should be treated with IV vancomycin and cefazolin. IV cefazolin can be used for suspected methicillin-susceptible Staphylococcus aureus (MSSA) and Streptococcus infections. Patients without systemic signs of toxicity can be treated with oral antibiotic therapy. Trimethoprim-sulfamethoxazole is used for suspected MRSA infections and cephalexin or dicloxacillin is used for MSSA or Streptococcus infections. In general, at least 10 days of antibiotics are warranted. This patient did not present with any risk factors for MRSA and can, therefore, be treated with coverage for MSSA. True or false: aseptic bursitis can be caused by gout. Answer: True.
Epididymitis > 35 yo <35 yo tx
Patient presents with gradual-onset unilateral scrotal pain PE will show increased color flow on Doppler, relief with testicular elevation (Prehn sign) Most commonly caused by < 35 years old: C. trachomatis, N. gonorrhoeae > 35 years old: E. coli, Pseudomonas Treatment < 35 years old: ceftriaxone-doxycycline > 35 years old: fluoroquinolones
A lesion in which vessel is indicated by ST elevations in leads V1, V2, V3, and V4? Answer: Left anterior descending (LAD).
look into this
A 55-year-old woman presents to the emergency department with sudden-onset shortness of breath and pleuritic chest pain. Her past medical history is significant for a right ankle fracture 2 weeks ago. She is on hormone replacement therapy for menopausal symptoms. Vital signs are BMI 39.2 kg/m2, T 98.6°F, BP 148/88 mm Hg, HR 110 bpm, RR 23 breaths per minute, and oxygen saturation 88% on room air. Her chest X-ray is normal. Forced dorsiflexion of the right foot elicits pain in the right calf. Which of the following diagnostic studies is the best next step for the most likely diagnosis? ACatheter pulmonary angiography BComputed tomography pulmonary angiography CImpedance plethysmography DVentilation-perfusion scan
Computed tomography pulmonary angiography dx: pulmonary embolism Signs and symptoms associated with pulmonary embolism include sudden-onset dyspnea, tachypnea, pleuritic chest pain, cough, hemoptysis, palpitations, tachycardia, leg pain, and Homan sign (calf pain with dorsiflexion of the foot). No single sign, symptom, or combination of signs or symptoms is specific to pulmonary embolism. Wells criteria and the pulmonary embolism rule-out criteria are used to evaluate for pulmonary embolism What is the McGinn-White sign? Answer: An S1Q3T3 pattern seen on ECG in patients with acute right heart strain.
A 53-year-old man presents to his primary care provider with a history of fatigue. He reports a 3-month history of daytime sleepiness, and he is concerned he has begun making mistakes at work. He also notes his wife has said his snoring at night has gotten worse. His blood pressure is 140/85 mm Hg, and his BMI is 35 kg/m2. The physical exam reveals a Mallampati class III oropharynx and a neck circumference of 18 inches. Which of the following is the first-line therapy option for this patient?
Continuous positive airway pressure Obstructive sleep apnea (OSA The last-line of therapy would be surgical interventions such as nasal septoplasty and uvulopalatopharyngoplasty. Lifestyle modifications include weight loss, exercise, abstaining from alcohol, changing sleeping position, and avoiding hypnotics. Potential complications of OSA include pulmonary hypertension and dysrhythmias. What is Pickwickian syndrome? Answer: Hypoventilation due to obesity.
A 22-year-old man presents to his primary care provider with complaints about recurrent ulcers on the inside of his lip. He states he gets these ulcers about six or seven times each year. They are very painful and take over 1 week to fully heal. His past medical history is unremarkable. Physical examination reveals two discrete 2 mm ulcers on the inside of the lower and upper lip, as shown in the image above. Which of the following should be recommended to this patient ?
2% viscous lidocaine Dx: Aphthous stomatitis, also known as a canker sore Patients with recurrent oral and genital ulcers should be evaluated for Behçet syndrome, a rare disorder characterized by recurrent oral and genital ulcerations, eye lesions, skin lesions, and a positive pathergy test.
A 50-year-old man with obesity presents to the clinic for follow-up on routine laboratory testing. He has kept a blood pressure log at home and has readings of 135/87 mm Hg, 142/83 mm Hg, and 140/86 mm Hg on 3 different days over the last 2 weeks. His hemoglobin A1C is 4.8%, his total cholesterol is 170 mg/dL, his HDL-C is 55 mg/dL, his LDL-C is 70 mg/dL, and his triglycerides are 135 mg/dL. Which of the following lifestyle interventions has been shown to lead to the most significant improvement in this patient's chronic condition shown in blood pressure readings or laboratory findings? A20-pound weight loss BAerobic exercise without weight loss CDietary potassium supplementation DDietary sodium restriction Which patients with primary hypertension should not increase dietary potassium intake to lower blood pressure?
20-pound weight loss While all of these lower blood pressure, 20 pounds of weight loss would lead to the most significant reduction in blood pressure. Which patients with primary hypertension should not increase dietary potassium intake to lower blood pressure? Answer: Patients on medications that reduce potassium excretion and patients with chronic kidney disease.
A 43-year-old man presents to his primary care provider for a discussion about colorectal carcinoma screening. He is concerned because his 57-year-old uncle and 53-year-old father were both diagnosed with colorectal cancer. His past medical history is significant for diabetes mellitus, alcohol use disorder, and a 30 pack-year smoking history. He also reports consuming a lot of red and processed meats. At what age should this patient receive a screening colonoscopy according to the United States Preventive Services Task Force?
43 years United States Preventive Services Task Force (USPSTF) currently gives a grade A recommendation for colorectal cancer screening average-risk patients beginning at 50 years of age. since the patient's father was diagnosed with colorectal cancer at 53 years old, the patient should be screened at 43 years old (10 years prior to the father's diagnosis). Patients may present with painless hematochezia, change in bowel habits, tenesmus, and symptoms associated with iron deficiency anemia. Diagnosis is made with a colonoscopy and biopsy. An apple core lesion on barium enema is associated with colorectal adenocarcinoma. A complete blood count may show evidence of anemia. A serum carcinoembryonic antigenshould be measured once colorectal carcinoma is confirmed and monitored serially after treatment. Treatment for colorectal carcinoma is surgical resection. Chemotherapy with 5-fluorouracil may be added if there is nodal involvement. Which criteria are used to identify patients who are at increased risk for Lynch syndrome? Answer: The Amsterdam criteria.
A 54-year-old man with a history of hypertension and hypercholesterolemia presents to the office for his annual physical exam. He has noticed over the past few months he has been more fatigued, with increased thirst and increased urination. His body mass index is 42 kg/m2. Which of the following confirms the suspected diagnosis for this patient?
A random plasma glucose concentration ≥ 200 mg/dL dx:DM The criteria for diagnosing diabetesinclude a random plasma glucose ≥ 200 mg/dL in a patient who is symptomatic, fasting plasma glucose ≥ 126 mg/dL on two separate occasions, glucose level ≥ 200 mg/dL 2 hours after oral glucose tolerance test in the serum or plasma, or hemoglobin A1C (HbA1C) > 6.5%. The patient above is already exhibiting symptoms, so his diagnosis would be confirmed if he had a random plasma glucose ≥ 200 mg/dL. A urine glucose test may be positive for glucose in the urine, and ketosis may be evaluated using urine and blood ketones tests. The HbA1C reflects the state of glycemia over the previous 8-12 weeks and is a better test for evaluating diabetic control What type of obesity is associated with insulin insensitivity? Answer: Visceral obesity due to the accumulation of fat in the omental and mesenteric regions.
21-year-old man presents to his primary care provider with complaints of right ankle pain for 12 hours. He is a competitive swimmer and has a history of tinea pedis. He has a fever of 102°F and a complete blood count significant for leukocytosis with a left shift. He is not on any medications. Which of the following physical examination findings is most consistent with the suspected diagnosis?
A single, erythematous patch with overlying warmth dx: Cellulitis Risk factors for cellulitis include chronic edema, venous insufficiency, lymphatic obstruction, and fissuring of the interdigital spaces secondary to tinea pedis. Cellulitis begins as a single, painful, erythematous patch with poorly demarcated borders and overlying warmth. As the infection progresses, fever, chills, malaise, lymphangitis, and lymphadenopathy may develop. lesion will begin to expand and may result in septicemia, hypotension, and shock if untreated. Laboratory findings are consistent with infection and include leukocytosis, neutrophilia, left shift, What form of cellulitis is characterized by a bright red plaque with raised and sharply demarcated borders? Answer: Erysipelas.
A 12-year-old boy accompanied by his mother presents to a gastroenterologist for a discussion about colonoscopy. His father had > 1,000 polyps on colonoscopy and passed away at age 55 while he was awaiting a proctocolectomy. The gastroenterologist explains to the patient and mother that he will need an annual colonoscopy for the rest of his life because he tested positive for a certain genetic mutation. Which of the following abnormal genes is associated with the patient's diagnosis?
APC # image look at autosomal dominant disorder familial adenomatous polyposis (FAP) lifelong annual colonoscopies, beginning at 12 years of age until a colectomy is performed. Indications for colectomy include suspected colorectal carcinoma, severe symptoms such as gastrointestinal bleeding, adenomas with high-grade dysplasia, multiple adenomas > 6 mm, increasing number of polyps on consecutive colonoscopies, or inability to adequately visualize on colonoscopy due to multiple diminutive polyps. Surgical resection options include total proctocolectomy with ileal pouch anal anastomosis or total colectomy with ileorectal anastomosis. Which guidelines are used in patients with Lynch syndrome-associated colorectal cancers to determine the need for tumor testing for microsatellite instability? Answer: The revised Bethesda guidelines.
A 30-year-old receptionist presents to her primary care provider with complaints of bilateral wrist pain, tingling, and intermittent numbness. She reports her symptoms are worse at night. She also states her .symptoms only affect her thumbs, index fingers, middle fingers, and half of her ring finger. Which of the following findings on physical examination is associated with a risk factor for the most likely diagnosis?
Acanthosis nigricans dx carpal tunnel Acanthosis nigricans is a velvety hyperpigmentation of the skin seen in patients with insulin sensitivity or diabetes mellitus. Diabetes mellitus is a risk factor for carpal tunnel syndrome. Carpal tunnel syndrome is the most common upper extremity entrapment neuropathy. It is typically caused by compression of the median nerve as it passes through the carpal tunnel in the wrist. Risk factors for carpal tunnel syndrome include female sex, pregnancy, and diabetes mellitus. Atrophy of the thenar eminence may develop if carpal tunnel syndrome is untreated. A positive Phalen sign is associated with paresthesias elicited with both wrists flexed and the dorsal aspects of the hands pressed against each other. Electrodiagnostic studies include electromyography and nerve conduction studies. The first-line treatment for carpal tunnel syndrome includes the use of night splints, which immobilize the wrist in a neutral position. Other conservative treatments include anti-inflammatory medications, physical therapy, exercise, stretching, and corticosteroid injections. Definitive therapy is surgical decompression and requires an endoscopic carpal tunnel release by cutting the transverse carpal ligament.
An 84-year-old woman is brought to a primary care provider by her granddaughter with concerns for dementia. The granddaughter states the patient does not seem to recognize family members anymore and is having a hard time remembering how to perform routine daily tasks. Her Montreal Cognitive Assessment score indicates moderate impairment. Which of the following describes the pathology of the most likely diagnosis?*****
Accumulation of hyperphosphorylated tau protein dx:Alzheimer disease Definitive diagnosis requires a brain biopsy, which is rarely performed. Histopathologic findings indicative of Alzheimer disease include amyloid plaques and the accumulation of hyperphosphorylated tau protein in neurofibrillary tangles within the brain. Current hypotheses suggest these amyloid plaques and tau proteins are toxic to the neurons in the brain, resulting in progressive cognitive decline. screened with the Mini-Mental Status Examination or the Montreal Cognitive Assessment. While there is no cure for Alzheimer disease, cholinesterase inhibitors, such as donepezil, galantamine, and rivastigmine, are used to treat symptoms of cognition and global functioning. What is finger agnosia? Answer: The loss of the ability to recognize one's own fingers, the fingers of others, and any drawing or representation of fingers.
2-year-old boy who weighs 15 kg presents to the urgent care after his parents caught him eating acetaminophen 1 hour ago. They estimate he ingested between 6 and 10 tablets of acetaminophen 500 mg. The patient appears to be in no acute distress, and his vital signs are within normal limits. Which of the following clinical interventions is best to administer to this patient? AActivated charcoal BGastric lavage CMagnesium citrate DSyrup of ipecac
Activated charcoal dx Tylenol toxicity The recommended dose of acetaminophen in children is 10 to 15 mg/kg/dose, not to exceed 90 mg/kg/day for more than 1 day. In exploratory acetaminophen overdose, patients should receive emergent care if the dose exceeds 200 mg/kg or 10 g total (whichever is less) in 24 hours, signs of toxicity. Signs and symptoms of acetaminophen overdose initially tend to be very mild or nonexistent. If the initial dose was very high, patients may exhibit nausea, vomiting, malaise, or lethargy. By 24 to 72 hours postexposure, the patient may begin to manifest signs of liver toxicity, such as right upper quadrant pain, elevated liver enzymes, and elevated prothrombin time. On days 3 through 4, evidence of liver failure and multiorgan failure may present, event occurred within 4 hours of presentation is with administration of activated charcoal at 1 g/kg. Contraindications of administration of activated charcoal include intestinal obstruction and altered mental status without a protected airway. The second step in the clinical care of a child poisoned by acetaminophen is to obtain a serum acetaminophen level at least 4 hours after ingestion and then every 1-2 hours afterward to plot the elimination of acetaminophen during treatment. The third step is to administer N-acetylcysteine to the patient if the serum acetaminophen level indicates toxicity. =======================================================
A 62-year-old man presents to the office with a 2-month history of fatigue, shortness of breath, and intermittent headaches. His physical examination is pertinent for conjunctival pallor, dried blood in the nares, and ecchymosis on the shins and arms. No lymphadenopathy or organomegaly is appreciated. A complete blood count with differential is obtained and reveals pancytopenia. Peripheral smear shows the above cell. What is the most likely diagnosis? AAcute lymphoid leukemia BAcute myeloid leukemia CChronic lymphoid leukemia DChronic myeloid leukemia
Acute myeloid leukemia The malignancy is associated with exposure to radiation, tobacco, and certain genetic abnormalities (trisomy 21) Acute Myeloid Leukemia (AML) Patient will be an adult Fever, fatigue, anemia, easy bruising or bleeding, petechiae, bone and joint pain, and persistent orfrequent infections PE will show hepatosplenomegaly Labs will show Auer rods, normocytic, normochromic anemia, blast cells What are the metabolic abnormalities associated with tumor lysis syndrome? Answer: Hyperphosphatemia, hypocalcemia, hyperuricemia, and hyperkalemia. "PKU ca"
Which of the following types of lung cancers is less likely associated with smoking? AAdenocarcinoma BLarge cell carcinoma CSmall cell carcinoma DSquamous cell carcinoma screaning and when do you stop ? and types of lung cancers
Adenocarcinoma It arises from mucous glands or epithelial cells within or distal to the terminal bronchioles and presents as peripheral nodules or masses. Lung Cancer Risk factors: history of smoking, asbestos exposure Sx: cough, hemoptysis, dyspnea, chest pain, weight loss, back pain, neurologic symptoms (CNS mets, paraneoplastic syndromes like Lambert-Eaton myasthenic syndrome)Pancoast syndrome: tumor in superior sulcus - shoulder pain, Horner syndrome Labs: hypercalcemia (PTHrP or bone mets), hyponatremia (SIADH), exudative effusion Diagnosis: CXR, CT scan, and biopsy to confirm Non-small cell lung cancer (most common)Adenocarcinoma, squamous cell, large cellTx depends on staging, may include surgical resection, chemotherapy, or tyrosine kinase inhibitors if positive for ALK or EGFR mutations Small cell lung cancerAssociated with neuroendocrine and paraneoplastic syndromesMetastasizes early, tx may include chemo, XRT and whole-brain radiation Leading cause of cancer-related death among men and women Screening (USPSTF 2021)Screen adults age 50-80 years who have a 20 pack-year smoking history and currently smoke or have quit within the past 15 years with an annual low dose CT scan Discontinue screening once the individual has not smoked for 15 years or has a limited life expectancy
A 68-year-old woman presents to her primary care provider for her annual physical exam. She has no significant medical history and does not recall receiving any vaccines in the past. She is concerned that two of her friends were hospitalized with pneumonia last winter, and she wonders whether she can do anything to prevent getting pneumoniathis year. What is the appropriate course of action with this patient? A 63-year-old man with recently diagnosed chronic obstructive pulmonary disease presents to the office for his 2-month follow-up. At his last visit in July, he received a tetanus-diphtheria-acellular pertussis vaccination. He does not recall receiving the pneumococcal vaccine in the past. Which of the following should this patient receive today? What is the recommended pneumococcal vaccination for patients aged > 65 years with no other risk factors and no prior history of vaccination?
Administer the 20-valent pneumococcal conjugate vaccine ---------------------------------------------------------- The ACIP recommends two vaccines that can prevent pneumococcal pneumonia for immunocompetent adults who are older than 65 years: 15-valent pneumococcal conjugate vaccine (PCV15) and 20-valent pneumococcal conjugate vaccine (PCV20). A)If PCV15 is administered, a dose of the 23-valent pneumococcal polysaccharide vaccine should be given at least 1 year later. If PCV 15 is given, administer PPSV23 at least 1 year later (except in patients with certain risk factors where an interval ≥ 8 weeks can be considered) B)The 13-valent pneumococcal conjugate vaccine (PCV13) is used for childhood vaccination. For those who previously received PCV13 but not PPSV23: Administer PPSV23 at least 1 year after PCV13 c)For those who have not previously received PCV (or vaccine status unknown): One dose of PCV20 or PCV15 should be given Vaccines are inactivated
A 42-year-old man presents to his primary care provider with complaints of an intensely pruritic rash on both of his arms for 2 days. He reports he has been hiking for the past 3 days and was wearing pants and short sleeves. Physical examination reveals the presence of tiny, weeping vesicles and crusted lesions in a linear distribution on both arms bilaterally. There are also erythematous, maculopapular lesions extending outward from the vesicular lesions. His rash is shown in the image above. A Gram stain and culture are negative. What is the most likely diagnosis?
Allergic contact dermatitis ------------------------------------------------------ Irritant contact dermatitis is characterized by excessive exposure to soaps, detergents, or organic solvents that causes physical, chemical, or mechanical irritation to the skin. Allergic contact dermatitis is a type IV hypersensitivity reaction mediated by T lymphocytes. Common causes of allergic contact dermatitis include poison ivy, poison oak, topical antimicrobials (e.g., bacitracin, neomycin), anesthetics, nickel, and adhesive tape. Irritant contact dermatitis is limited to the sites of direct contact with the irritant, while allergic contact dermatitis may occur beyond the sites of direct contact. treatment Treatment for contact dermatitis includes removing the offending agent. If a patient has had contact with poison ivy or poison oak, washing with soap and water within 30 minutes of contact should be encouraged to remove the causative oils from the skin. Topical corticosteroids may be used for localized involvement, while widespread involvement should be treated with systemic corticosteroids. Weeping lesions can be treated with gentle cleansing and wet to dry dressing changes. Calamine lotion or zinc oxide paste may also be used between a wet dressing for areas where weeping is minimal.
A 45-year-old man presents to the office with patchy hair loss that started suddenly about 1 month ago. The patient has not noticed any hair regrowth. Upon physical examination, there are three discrete smooth oval patches throughout the scalp that are flesh colored. The patches are neither tender nor pruritic. He suffered a death in the family about 3 months ago. Which of the following is the most likely diagnosis? AAlopecia areata BAndrogenetic alopecia
Alopecia areata Alopecia Areata History of an autoimmune disorder Reports hair loss PE will show patches of smooth, nonscarring hair loss with patches of smaller hairs, called exclamation hairs hair "pull test" can help differentiate types"pull test" is done by grasping approximately 40 to 60 hairs between the thumb and forefinger and applying steady traction (slightly stretching the scalp) as you slide your fingers along the length of the hair Most common cause is autoimmune Treatment is intralesional corticosteroids ---------------------------------------------- Androgenetic alopecia (B) is hair loss found in persons with genetic predispositions that affects both men and women and causes hair thinning and eventual denudation of the affected areas. The patient in the above vignette is unlikely to have this condition because his hair loss is in patches. He does not have gradual recession of the front hairline or temporal areas that are classically found in androgenetic alopecia. Telogen effluvium (C) is a type of alopecia characterized by diffuse hair shedding most commonly in the scalp. The hair will appear thinner than before, but it does not occur in patches like the patient above. Common causes of this condition include emotional stressors, childbirth, and major illnesses among others.
A 60-year-old woman presents to your office with a complaint of epigastric pain after eating. She describes the pain as dull, aching, and gnawing. Physical exam reveals mild epigastric tenderness to palpation. Urea breath testing is positive. Which of the following is a component of first-line therapy? Where in the stomach are most benign ulcers found?
Amoxicillin dx: h pylori amoxicillin, clarithromycin, and a proton-pump inhibitor, such as omeprazole, for 14 days. Test for cure should be done on all patients who are treated for H. pylori infection at least 4 weeks after completing the regimen. Where in the stomach are most benign ulcers found? Answer: The antrum.
32-year-old woman who is pregnant presents complaining of a strange circular rash to her lower extremity. She recently went camping in Connecticut. On physical exam, she is noted to have an annular erythematous rash with central clearing. The patient has no known drug allergies. What is the most appropriate treatment for this patient?
Amoxicillin dx:Lyme disease Treatment is doxycyclineChildren: amoxicillin or doxycycline (if used for < 21 days)Pregnant: amoxicillin Bilateral facial nerve palsy is virtually pathognomonic for Lyme disease Most commonly caused by Borrelia burgdorferi carried by Ixodes tick
A 25-year-old woman presents with right-sided facial pain, nasal congestion, and purulent discharge for 2 weeks. The patient reports that her symptoms have worsened over the past 3 days. The patient reports that she has tried over-the-counter treatments, such as nasal decongestants and sinus washes, without relief. Physical exam reveals right maxillary sinus tender to palpation and no facial swelling. The patient has no known allergies and takes no medications. What is the most appropriate intervention for this patient? A 23-year-old woman with a history of nasal polyps presents with facial pain, headache, and fever. Her symptoms started approximately 12 days ago with cough, rhinorrhea, and frontal sinus pain, which improved on day 5 but then worsened after day 8. On exam, tenderness to palpation over the frontal sinus is noted as well as purulent nasal drainage. Which of the following organisms is the most common bacterial cause of the suspected diagnosis?
Amoxicillin-clavulanate 875 mg/125 mg twice daily for 7 days dx:Acute bacterial rhinosinusitis is sinusitis of less than 4 weeks duration occurring after a viral upper respiratory infection. result of impaired mucociliary clearance due to inflammation of the nasal mucosa with obstruction of the ostiomeatal complexes. This impairment leads to accumulation of mucus, secondarily infected with bacteria. maxillary sinus is the most common location= Haemophilus influenzae and Streptococcus pneumoniae Clinical diagnosis of bacterial rhinosinusitis is reserved for patients who have symptoms for more than 10 days; have purulent nasal discharge, nasal obstruction, and facial pain; or in patients whose symptoms initially improve but then worsen about 5 to 6 days later. It can also be considered in patients who present with high fever greater than 102°F, purulent nasal discharge, and facial pain for 3 to 4 consecutive days at the beginning of the illness. Patients often report localization of the pain to the involved sinus and have increased pain with leaning forward. mild to moderate symptoms, symptomatic treatment is recommended with nonsteroidal anti-inflammatories, oral and topical decongestants, nasal washing, and nasal steroids. Antibiotics should be reserved for complicated or protracted cases, as most patients' symptoms resolve within 2 weeks without antibiotics. Antibiotics can be considered in patients whose symptoms are severe or those with symptoms longer than 10 days. Complicated patients, such as those who are immunocompromised, also benefit from antibiotic therapy. The recommended first-line therapy is amoxicillin-clavulanate either 875 mg/125 mg twice daily or 500 mg/125 mg three times daily for 5 to 7 days. For those with a penicillin allergy or hepatic impairment, doxycycline 100 mg twice daily or 200 mg once daily for 5 to 7 days is a reasonable alternative. What is the initial recommended treatment for acute invasive fungal sinusitis? Answer: IV amphotericin B and surgical debridement.
A 13-year-old boy presents to his pediatrician for his first sports physical. Physical examination is significant for expiratory wheezes. Pulmonary function and bronchodilator response tests are ordered. The patient's baseline forced expiratory volume in 1 second is 75% of the predicted value, and his forced vital capacity is 3.5 L. Which of the following bronchodilator responses is most consistent with asthma?
An increase in both forced expiratory volume and forced vital capacity
What medication improves intermittent claudication in patients with peripheral artery disease? Diagnosis is made by ankle-brachial index (ABI)If limb is threatened: contrast arteriography (gold standard)
Answer: Cilostazol.
What are side effects of tadalafil? What are common side effects of topiramate?
Answer: Headache, abdominal pain, muscle pain, stuffy nose, and flushing. Answer: Paresthesias, anomia, weight loss, anorexia, metabolic acidosis, and nephrolithiasis.
Which screening test for corneal disorders is used to evaluate for the presence of anterior chamber leakage into the cornea?
Answer: Seidel test
When might endoscopy or surgery be indicated in a toxic ingestion?
Answer: When the substance ingested is a large amount of heavy metals, a pharmacobezoar, or an illicit drug packet.
A 26-year-old woman presents to her primary care physician with malaise, nausea, mild abdominal pain, and low-grade fever. The patient reports having a one-time encounter with a male sexual partner she met at a bar about 10 weeks ago. Physical exam findings show mild right upper quadrant abdominal tenderness. CMP reveals that the patient has an AST of 1,000 units/L and an ALT of 1,550 units/L. Which of the following would indicate immunity to the suspected infection?
Anti-HBs ================================================= Hepatitis B surface antigen (HBsAg) appears in serum 1 to 10 weeks after exposure and is elevated prior to symptoms or ALT elevation. If a patient recovers, HBsAg is typically undetectable after 4 to 6 months. The disappearance of HBsAg is followed by the appearance of hepatitis B surface antibody (anti-HBs). Anti-HBs antibody persists for life and is responsible for long-term immunity. IgM anti-HBc presents shortly after HBsAg and indicates a diagnosis of acute hepatitis B.
35-year-old man with a history of a bicuspid aortic valve presents to his primary care provider with concerns of being hyperaware of his pulse. He states his pulse constantly feels like it is bounding, and he has also developed shortness of breath during exertion. His blood pressure is 130/60 mm Hg. The physical exam reveals a holodiastolic high-pitched murmur at the left upper sternal border. The murmur decreases in intensity with Valsalva maneuver. What is the definitive management of this patient's condition?
Aortic valve replacement Aortic Insufficiency History of infectious endocarditis, dissection, Marfan syndrome HF symptoms: dyspnea, orthopnea, paroxysmal nocturnal dyspnea, fatigue PE will showWidening pulse pressureBounding water-hammer peripheral pulsesHead bobbing with systole (de Musset sign)Prominent nail pulsations (Quincke pulse)Hyperdynamic apical pulse displaced to the leftDiastolic blowing murmur best heard along LSBSystolic or diastolic thrill or murmur heard over the femoral arteries (Duroziez sign)In severe AR, a mid-diastolic murmur (Austin-Flint murmur) Diagnosis is made by echo to determine the severity of regurgitation Most commonly caused by abnormal leaflets or proximal aortic root Diuretics, digoxin, ACEI, salt restriction (if HF), surgery for symptomatic patients or EF < 50% 0)Which valvular disorder is associated with an Austin Flint murmur? Answer: Aortic regurgitation. A late diastolic murmur best heard at the apex. The sound heard is due to retrograde blood flow competing with antegrade flow from the left atrium.
A 67-year-old man with a history of heart failure, hypertension, and diabetes mellitus presents to your office with complaints of heart palpitations, dizziness, and fatigue. Physical exam findings include a body mass index of 35.2 kg/m2and an irregularly irregular pulse with a rate of 120 bpm. Electrocardiogram reading shows absent P waves, R-R intervals without a repetitive pattern, and irregular QRS complexes. Which of the following is the most appropriate therapy?
Apixaban atrial fibrillation is the most common chronic type. rule out thyroid disorders and diabetes mellitus Anticoagulation should be considered when the CHA2DS2-VASc score is 1 and is recommended when the score is ≥ 2. This patient has a CHA2DS2-VASc score of 4 due to his age and the presence of three comorbidities, so he should be started on an anticoagulant. Anticoagulation may be with warfarin, which requires frequent monitoring, or one of four direct oral anticoagulants: dabigatran, rivaroxaban, apixaban, or edoxaban. Patients with atrial fibrillation also typically require slowing of the ventricular rate to improve symptoms. Agents used for rate control include beta-blockers, calcium-channel blockers, and digoxin.
A 67-year-old man with diabetes presents to the clinic complaining of an episode of slurred speech and right-sided upper and lower extremity weakness this morning that lasted 30 minutes and then resolved on its own. His current vital signs are blood pressure 150/90 mm Hg, nonfasting blood glucose 200 mg/dL, pulse 80 beats per minute, and respiration rate 16 breaths per minute. Physical exam is normal, and an electrocardiogram shows normal sinus rhythm. A computed tomography of the head and a Doppler ultrasound of the carotid arteries are pending. Which of the following is the most appropriate treatment for secondary prevention of ischemic stroke in this patient? AAspirin 325 mg and clopidogrel 75 mg daily BAspirin 325 mg daily CRivaroxaban 20 mg daily DWarfarin 5 mg daily
Aspirin 325 mg and clopidogrel 75 mg daily Patients at moderate to high risk of stroke should receive both aspirin and clopidogrel daily dx:Transient ischemic attack refers to an acute episode of neurologic compromise, which could include slurred speech, vision deficit, or extremity weakness, that resolves on its own without causing tissue damage. Most transient ischemic attacks resolve within 24 hours, ABCD2 system of risk stratification can be employed. In this system, A stands for age. Patients over 60 years are awarded 1 point. B stands for blood pressure. Patients with systolic blood pressure ≥ 140 mm Hg or diastolic blood pressure ≥ 90 mm Hg receive 1 point. C stands for clinical features. Patients with unilateral weakness during the ischemic episode are awarded 2 points, and patients with speech deficits receive 1 point. D stands for duration. If the transient ischemic attack lasted for ≥ 60 minutes, the patient receives 2 points. Symptom duration of 10 to 59 minutes is awarded 1 point. D also stands for diabetes, so patients with diabetes are awarded 1 point. A total score of 6 to 7 indicates a high 2-day risk of stroke. A score of 2 to 5 indicates a moderate 2-day risk of stroke, and a score of 0 or 1 is indicative of low risk. Low-risk TIA (ABCD2 score < 4) or moderate to major ischemic stroke (National Institutes of Health Stroke Scale (NIHSS) > 3)Treatment with aspirin alone The patient in the above vignette would receive a score of 6 and thus would be at high risk of stroke in the next 2 days. Patients at moderate to high risk of stroke should receive both aspirin and clopidogrel daily while awaiting results of imaging studies to determine the etiology of the transient ischemic attack.
A 73-year-old man with a history of alcohol use disorder, COPD, and hyperthyroidism presents with heart palpitations and chest pain for 2 hours. On exam, an irregular pulse is noted. His rhythm strip is shown above. Which of the following is the most likely diagnosis?
Atrial fibrillation What is holiday heart syndrome? Answer: An irregular heart rhythm that presents in an otherwise healthy individual after consuming excessive amounts of alcohol. Atrial Fibrillation Rate will be irregular Rhythm will be irregular Notable feature: No defined P waves TreatmentUnstable: cardioversionStable: rate control is mainstay (diltiazem, metoprolol) > 48 hours: anticoagulate for 21 days prior to cardioversionDetermine the need for anticoagulation by using CHA2DS2-VASc score Most common sustained dysrhythmia in adults
A 21-year-old man presents with an insidious onset of nonproductive cough for 2 weeks. He reports sore throat, myalgias, and a high fever. Physical exam reveals a normal pulse despite a high fever. Diffuse crackles and rhonchi are noted on auscultation. Laboratory studies show mild leukocytosis but are otherwise unremarkable. His chest radiograph is shown above. Which of the following organisms is the most likely cause of his condition?
Atypical pneumonia is most commonly caused by Mycoplasma pneumoniae but less frequently by Chlamydia pneumoniae, C. psittaci, Coxiella burnetii, Legionella spp, and viruses Clinical features of atypical pneumonia include an insidious onset of dry cough (with no sputum production), accompanied by sore throat, headache, myalgias, and fevers (chills less common). Diagnosis is made by chest radiography, which demonstrates reticulonodular or interstitial infiltrates with absent or minimal consolidation. Treatment of atypical pneumonia involves the use of macrolides or doxycycline.
A 21-year-old G2P2 woman presents to her primary care provider with complaints of a facial rash, painless oral ulcers, and arthralgia. She has a history of Raynaud phenomenon. She recently moved from Zimbabwe and is unsure of her vaccination status. Physical examination reveals obesity and the facial rash shown above. Laboratory testing is positive for anti-double-stranded DNA and anti-Smith antibodies. Her vitamin D level is low. Which of the following is appropriate health maintenance education for this patient? Which antibody is associated with drug-induced lupus erythematosus?
Avoid pregnancy for 6 months after active disease dx: Systemic lupus erythematosus (SLE) malar (butterfly) rash sparing the nasolabial folds, discoid rash, mucositis (e.g., oral or nasal ulcers), serositis (e.g., pleuritis, pericarditis), vasculitis, hematologic disorders (e.g., hemolytic anemia, leukopenia, lymphopenia, thrombocytopenia), nondeforming arthritis, kidney disorders, pulmonary disorders, gastrointestinal disorders, Raynaud phenomenon, and photosensitivity. Laboratory testing will be positive for antinuclear antibodies. Anti-double-stranded DNA and anti-Smith antibodies have high specificity for SLE. Health maintenance for patients with systemic lupus erythematosus includes nonpharmacologic and preventive measures. Patients should limit their exposure to ultraviolet light, protect themselves with sunscreen, and avoid medications that cause photosensitivity. Vitamin supplementation is important if the diet is not well balanced. Vitamin D supplementation may be necessary due to limited sunlight exposure. Salt restriction is indicated in SLE patients with hypertension or nephritis. Exercise should be encouraged to maintain muscle mass, strengthen bones, and prevent fatigue. Smoking cessation is associated with active SLE disease and coronary heart disease. Both live attenuated and inactive vaccinations should be given before starting immunosuppressive therapies. Which antibody is associated with drug-induced lupus erythematosus? Answer: Antihistone antibody.
A 63-year-old man presents to his primary care provider with his wife, who is concerned about a slow-growing nodule on his nose that has been present for the past 3 years. The patient has no significant past medical history. Physical examination reveals a well-developed, well-nourished, fair-skinned man who appears his stated age. A waxy nodule measuring 1.5 cm x 1.5 cm is present on the lateral aspect of the patient's nose and is shown above. The clinician suspects the nodule is a form of skin cancer and performs a punch biopsy. Which of the following is the most likely diagnosis?
Basal cell carcinoma Basal Cell Carcinoma (BCC) Sx: painless, slow-growing lesion on the face, ears, or neck PE: pearly papule with rolled borders and telangiectasia Diagnosis: shave biopsy Histology: atypical basal cells with blue-staining nuclei Most common type of skin cancer Treatment: surgical excision for lesions with low risk of recurrence, Mohs for high risk Prevention includes sunscreen, UV light protection True or false: non-Hodgkin lymphoma is more common than Hodgkin lymphoma. Answer: True.
A 50-year-old man presents to his primary care provider with complaints of shortness of breath and a dry cough that has progressed over the last 3 years. He has worked as a field engineer on nuclear power plants for 29 years. Physical examination reveals the presence of multiple cutaneous nodules on the fingers and forearms. His chest X-ray reveals hilar lymphadenopat
Berylliosis look past image Does pneumoconiosis typically present with an obstructive or restrictive pattern on pulmonary function testing? Answer: Restrictive pattern.
A 23-year-old man presents to his primary care provider with complaints of left knee pain, dysuria, and hematuria for 10 days. He reports he had a gastrointestinal infection 3 weeks ago that was resolved after treatment with ciprofloxacin. Urinalysis is significant for a positive leukocyte esterase with 10 RBC/hpf and 7 WBC/hpf. Physical examination reveals a swollen, tender left knee. Which of the following additional findings is most consistent with the suspected diagnosis? ABilateral flank ecchymosis BBilateral injected conjunctiva CLeft-sided supraclavicular lymphadenopathy DStrawberry red tongue
Bilateral injected conjunctiva dx:Reactive arthritis Treatment includes antibiotics for infection, supportive treatment for nausea or vomiting, andnonsteroidal anti-inflammatory drugs (NSAIDs) for arthritis What four diseases are most commonly associated with HLA-B27 positivity? Answer: Psoriatic arthritis, ankylosing spondylitis, inflammatory bowel disease, and reactive arthritis (noted by the mnemonic PAIR).
A 70-year-old woman reports to her primary care physician with a 2-day history of severe back pain. She picked up a heavy load of laundry just prior to experiencing the back pain. She has a past medical history of rheumatoid arthritis, which has been treated with long-term prednisone use, and a 25 pack-year smoking history. Physical exam reveals a thin, Caucasian woman with moderate kyphosis. She is tender at the level of L1. Lower extremity strength and sensation are intact bilaterally. Her vertebral X-ray is shown above. Which of the following is the first-line treatment of this condition?**
Bisphosphonates. (e.g., alendronate, pamidronate, zoledronic acid) dx: Osteoporosis Risk factors for osteoporosis include advanced age, sex hormone deficiency, alcohol use disorder, smoking, long-term proton pump inhibitor therapy, and a history of glucocorticoid use. Which class of medications used for the treatment of osteoporosis has been associated with osteonecrosis of the jaw? Answer: Bisphosphonates. When should oral bisphosphonates be taken? Answer: In the morning on an empty stomach, and the patient should remain upright for 30 minutes afterward.
A 26-year-old man presents to the office complaining of a cough for 2 weeks. He reports no other symptoms and has no sick contacts. He works as a teller at a local bank. He had a single, similar episode last year in November. Today, his vital signs show a blood pressure of 114/68 mm Hg, pulse of 94 bpm, temperature of 98.6°F, and respirations of 10 breaths/minute. Cardiovascular exam reveals a regular heart rate without murmurs, rubs, or gallops. There are no jugular venous distention and no carotid bruits. Lungs are clear to auscultation bilaterally without crackles or wheezes. There is no use of accessory muscles during respiration. There is no fremitus. A chest radiograph is ordered, and the results are shown above. What is the most likely diagnosis?
Bronchitis What is the name of the pulmonary physical exam test performed by placing the ulnar aspects of both hands against the patient's chest while the patient says the number 99? Answer: Tactile fremitus.
A 65-year-old man with a history of congestive heart failure and hyperlipidemia presents to the office for management of symptoms related to congestive heart failure. He reports dyspnea on exertion. His vital signs are within normal limits. Electrocardiogram reveals normal sinus rhythm and left ventricular hypertrophy. A recent echocardiogram reveals an ejection fraction of 40%. His current daily medications include atorvastatin, furosemide, and lisinopril. Which of the following medications would be most appropriate to add to his medication regimen at this time?
Carvedilol dx:Congestive heart failure includes heart failure with preserved ejection fraction (ejection fraction > 50%), also referred to as diastolic heart failure, Initial pharmacologic therapy of heart failure with reduced ejection fraction includes a combination of a diuretic, an angiotensin system blocker (either an angiotensin-converting enzyme (ACE) inhibitor, angiotensin II receptor blocker (ARB), or angiotensin receptor-neprilysin inhibitor (ARNI)), and a beta-blocker The beta-blockers that have proven beneficial in heart failure randomized trials include carvedilol, extended-release metoprolol succinate, and bisoprolol.
A 28-year-old man presents with fatigue and weight gain. He states he has difficulty concentrating on tasks at work and frequently has feelings of worthlessness. He has difficulty both falling asleep and maintaining sleep. He has lost interest in social interaction and does not enjoy the activities that he once enjoyed. These symptoms have been occurring for the last 4 weeks. Which of the following historical findings would be most consistent with the diagnosis?
Childhood traumaor early-onset anxiety may contribute to the development of depression. dx: Major depressive disorder Other historical factors include low self-esteem, substance misuse, parental loss, low parental warmth, marital issues, and low education status rule out other causes of depression, including a pregnancy test for women and thyroid testing. goals should focus on restoring baseline function and the ability to participate in activities of daily living without impairment. Treatment : SSRI
A 42-year-old man presents to his primary care provider with complaints of hearing loss and painless ear drainage in his right ear for 1 month. He states that, 2 months ago, he spent a day practicing at the shooting range without any ear protection. Weber test lateralizes to the right ear. Rinne test on the right ear reveals bone conduction greater than air conduction. Otoscopic examination reveals the presence of a pearly white mass in the middle ear. Which of the following is the most likely diagnosis?
Cholesteatoma Conductive hearing loss associated with cholesteatoma is most commonly due to erosion of which of the ossicles? Answer: The distal portion of the incus. is a keratinized collection of desquamated epithelial tissue in the middle ear or mastoid. Primary acquired cholesteatoma is commonly a result of tympanic membrane inversion secondary to chronic eustachian tube dysfunction (e.g., chronic allergies, chronic sinusitis). Secondary acquired cholesteatoma is the result of tympanic membrane perforation or other iatrogenic causes (e.g., tympanoplasty) painless, malodorous otorrhea and conductive hearing loss. Audiogram testing may be used to evaluate hearing loss. Imaging with computed tomography scan is only indicated if extracranial complications are suspected, particularly in patients with systemic illness or signs of localized infection. Antibiotics are indicated in cases with underlying infection. Definitive treatment is surgical excision of the cholesteatoma with tympanoplasty. Mastoidectomy may be indicated if the lesion extends superiorly or posteriorly. Surgical reconstruction of the ossicular chain is indicated in cases of significant hearing loss. Acquired Cholesteatoma Patient will have a history of chronic ear infections or tympanostomy tubes Painless otorrhea PE will show yellow or white mass behind the tympanic membrane Treatment is tympanomastoid surgery
A 55-year-old man with a past medical history of hypertension and cigarette smoking presents to the clinic reporting bilateral lower limb pain for the past several months. He states the pain used to only occur when going on long walks, but for the past few weeks, he has had the pain when he walks to the mailbox. Physical exam reveals diffuse hair loss of the bilateral lower extremities and diminished posterior tibial and dorsalis pedis pulses bilaterally. Ankle-brachial index testing is performed in the clinic and reveals an ankle-brachial index of 0.8. The patient is counseled on smoking cessation and lifestyle changes, including exercise therapy. He is seen at a 3-month follow-up visit with minimal improvement in symptoms and no improvement in ankle-brachial index. Which of the following is the most appropriate next step in management?
Cilostazol dx:Peripheral artery disease is an occlusive atherosclerotic disease Peripheral Artery Disease Patient presents with pain in the affected extremity related to activity (intermittent claudication) PE will show cool extremity with absent or diminished pulses Diagnosis is made by ankle-brachial index (ABI)If limb is threatened: contrast arteriography (gold standard) Most commonly caused by atherosclerotic disease The gold standard for diagnosis of peripheral vascular disease is arteriography. Therapy includes control of risk factors and lifestyle modifications such as healthy diet, exercise, and smoking cessation. Pharmacological therapy is often needed and includes platelet inhibitors such as aspirin or clopidogrel. Cilostazol is a phosphodiesterase inhibitor that improves symptoms of claudication by suppressing platelet aggregation and dilating arterioles. In severe cases, revascularization via percutaneous transluminal angioplasty or bypass may be indicated. True or false: most patients with peripheral vascular disease are asymptomatic. Answer: True.
A 15-year-old girl presents with concerns for heavy menstrual bleeding. Menarche was at 13 years of age, and her periods have been regular and heavy for the last year. She states that she often soaks through more than one pad per hour on her heaviest day. The length of her cycle is typically 27 days with 8 days of menstrual bleeding. She reports that she has never been sexually active in the past. She reports no pelvic pain besides mild cramping during the first 2 days of her cycle. Which of the following diagnostic studies would be most useful in determining the diagnosis?
Coagulation studies When do women with von Willebrand disease typically present with menorrhagia? Answer: Near menarche or at menopause.
A 12-year-old girl with a history of Turner syndrome presents to her pediatrician's office for a routine visit. Her vitals include weight of 155 pounds, body mass index 21 kg/m2, oxygen 99% on room air, blood pressure 145/86 mm Hg taken on her left arm, pulse 89 beats per minute, and respiratory rate 20 breaths per minute. On physical exam, she is noted to have a new midsystolic murmur that radiates to her back, her brachial and radial pulses are 2 , she has trace femoral pulses, and she is noted to be noncyanotic. Which of the following is the most likely diagnosis?
Coarctation of the aorta area of narrowing just distal to the origin of the left subclavian artery. It is commonly associated with Turner syndrome, The diagnostic study of choice is a transthoracic echocardiogram, and a CT angiogram is the gold standard. A chest X-ray can also be done, which will show rib notching, also known as a 3 sign. Treatment for all ages involves widening the narrowed area of the aorta, which can be done by an end-to-end anastomosis, balloon angioplasty, or a stent placement. It is a continuous murmurheard in mid- to late systole and is best heard on the patient's midline upper back between the scapulae or over the left anterior chest. T '' Coarctation of the Aorta Narrowing of descending aorta Sx: headaches, dyspnea, leg claudication PE: higher blood pressure in the arms than in the legs, delayed femoral pulse EKG: LVH CXR: notching of ribs, "3 sign" (indentation of aorta) Dx: echo Treatment is balloon angioplasty with stent placement or surgical correction Associated with Turner syndrome, bicuspid aortic valve, and intracranial aneurysms ================================================== atients with atrial septal defect (A) are commonly asymptomatic until the age of 30 years or older. If symptomatic at a younger age, they will present with recurrent respiratory infections, failure to thrive, and exertional dyspnea, which the patient in the vignette did not have. Patent ductus arteriosus (C) presents with a continuous machinelike murmur heard best on the pulmonic area with wide pulse pressures. Tetralogy of Fallot (D) would present with a history of tet spells, where the patient becomes cyanotic during episodes of crying, feeding, or agitation. This presentation is due to a right to left shunt and is commonly relieved by the patient squatting.
A 32-year-old man presents with concerns about an intense fear of driving. The fear has gotten worse over the past few years, and now he has stopped driving completely, which is affecting his employment and social life. Which of the following is the most appropriate clinical intervention for this patient?
Cognitive behavioral therapy dx:Specific phobia What are the five main types of specific phobias? Answer: Animal, natural environment, blood-injection-injury, situational, and other (including loud sounds, choking, and vomiting).
A 24-year-old woman presents to her primary care provider with complaints of progressively worsening fatigue, irritability, dizziness, and headaches for the past 4 months. She states she has had a decreased appetite and has lost 15 pounds in the last 2 months. She has also started to crave salt. Vital signs are T 98.6°F, BP 92/68 mm Hg, HR 80 bpm, RR 18 breaths per minute, and oxygen saturation 99% on room air. Physical examination reveals hyperpigmented skin over the knuckles, palmar creases, gingival mucosa, and vermilion border of the lips. Her fasting blood work is significant for hypoglycemia, hyponatremia, hyperkalemia, and hypercalcemia. Which of the following diagnostic studies is most helpful in confirming the suspected diagnosis? ACosyntropin stimulation test BDexamethasone suppression test CSerum dehydroepiandrosterone level DWater deprivation test
Cosyntropin stimulation test dx: Primary adrenal insufficiency, also known as Addison disease measures cortisol levels after ACTH is injected, with low cortisol levels indicative of adrenal insufficiency. Treatment for primary adrenal insufficiency includes daily corticosteroid replacement therapy with hydrocortisone and mineralocorticoid replacement therapy with fludrocortisone. ===================================================== Dexamethasone suppression test (B) is used to diagnose Cushing syndrome. Cushing syndrome is due to increased cortisol levels. Clinical manifestations include skin atrophy, moon facies, a buffalo hump due to increased upper back fat deposition, and pigmented striae. The excess cortisol in Cushing syndrome causes abnormal fat deposition, increased androgens and aldosterone, and increased protein catabolism. In contrast, the cortisol levels in Addison disease results in decreased androgens, decreased aldosterone, and protein anabolism. Failure of dexamethasone to suppress cortisol is indicative of Cushing syndrome. The serum dehydroepiandrosterone level (C)is elevated in all patients with adrenal insufficiency but is also seen in approximately 15% of the normal population, thus it is not used to diagnose primary adrenal insufficiency because of its lack of specificity. The water deprivation test (D)is used to distinguish between central diabetes insipidus and nephrogenic diabetes insipidus. Vasopressin is given after water is withheld. An increase in urine osmolarity is indicative of central diabetes insipidus. Diabetes insipidus presents with increased thirst, increased urination that results in dilute urine, and dehydration.
A 49-year-old woman presents with reports of hot flashes. She thinks she is nearing menopause and is wondering what she can do to reduce the severity of the vasomotor symptoms. These hot flashes are bothersome, but they do not interfere with her usual activities of daily living. Which of the following is the most appropriate intervention at this time?
Counsel patient to dress in layers and avoid triggers dx: menopause look at image Hot flashes are the most common symptom occurring in most perimenopausal women. Women with mild hot flashes that do not interfere with usual daily activities can typically be treated with conservative, behavioral measures, including avoiding triggers, dressing in layers, and lowering room temperatures. Treatment of persistent perimenopausal symptoms that interfere with daily activities can include an estrogen or estrogen-progestin combination therapy. Unopposed estrogen in a woman with an intact uterus can cause endometrial hyperplasia and increase the risk of endometrial cancer.
A 60-year-old man presents to his primary care provider with new-onset painless hematochezia. He reports no recent trauma, abdominal pain, or change in bowel movements. A colonoscopy reveals the presence of a nonpolypoid, bleeding mass arising from the mucosa and protruding into the lumen of the proximal ascending colon. Which of the following laboratory findings is most often associated with the suspected diagnosis? ABasophilic stippling BDecreased mean corpuscular hemoglobin concentration CDecreased total iron-binding capacity DIncreased mean corpuscular volume
Decreased mean corpuscular hemoglobin concentration dx: Colorectal carcinoma Definitive diagnosis is made on histological evaluation of a biopsy during colonoscopy. A carcinoembryonic antigen is not used for diagnosis or screening but should be obtained after a diagnosis is made and monitored serially after treatment. Treatment for colorectal carcinoma is surgical excision. Chemotherapy is indicated if there is nodal involvement. What is the most common cause of a large bowel obstruction in an older patient? Answer: Neoplasm.
A previously healthy 58-year-old man is admitted to the hospital after an open tibia fracture resulting from a motor vehicle collision. He has no significant past medical history and no known drug allergies. He is given hydrocodone for pain management. On postoperative day 3, his family members tell the nurse the patient has been acting strangely. They report he is periodically confused and unable to remember their names. They noticed the changes began around 2 days ago and worsen around noontime and sundown. The nursing staff has also noticed the patient seems to have sporadic episodes where he becomes inattentive and forgets how to speak English, which is not his first language. Which of the following is the most likely diagnosis? what are the five features hallmark feature
Delirium from taking hydrocodone Medications associated with a risk of delirium include sedatives, anticholinergics, opioids, benzodiazepines, and antihistamines. five features with delirium: a disturbance in attention and awareness, a change in baseline over hours or days that fluctuates during the course of the day, an additional disturbance in cognition (e.g., memory deficit, disorientation, language, visuospatial ability, perception), a disturbance that is not better explained by a preexisting neurocognitive disorder, and evidence the disturbance is caused by a medical condition, substance overdose or withdrawal, or medication side effect The hallmark feature that distinguishes delirium from dementia is inattention True or false: visual, auditory, or somatosensory hallucinations may accompany delirium. Answer: True.
An 88-year-old woman is brought to the clinic by her daughter, who complains her mother is increasingly forgetful, irritable, and easily confused. The patient is currently being treated for hypertension, dyslipidemia, and osteoporosis. Her vital signs are within normal limits, including a normal urinalysis. She reports no fever, weakness, or dysarthria. Her daughter states the changes have been gradual over the past 2 years. Which of the following is an essential element in the diagnosis of this patient's condition?*** AComputed tomography of the head without contrast BDetailed history by patient and daughter, separately if possible CMagnetic resonance imaging of the head DMini-Mental Status Exam
Detailed history by patient and daughter, separately if possible What is mild cognitive impairment? Answer: A decline in memory or cognition that is noticeable to the patient or family members but does not significantly interfere with activities of daily living. ===================================================== Mini-Mental Status Exam (D) is a screening tool used to evaluate patients for cognitive decline. It is important to have a clear picture of the patient's cognitive function. It may also be useful to administer as a baseline when initiating treatment for dementia and to assess cognitive decline during treatment. The Mini-Mental Status Exam and other screening tools for dementia are not necessary to establish the diagnosis of dementia, however, they are an important piece of the overall care of the patient. The Mini-Mental Status Exam does not assess mood or thought content and cannot distinguish patients who have a mood disorder underlying their dementia. Magnetic resonance imaging of the head (C) is preferred over computed tomography of the head for evaluation of cognitive decline
A 22-year-old woman with no significant past medical history presents to the gastroenterology clinic with chronic abdominal pain and intermittent constipation and diarrhea for the past 6 months. She reports no fever, weight loss, or blood in her stool. On exam, she has diffuse abdominal discomfort with palpation but no focal tenderness. Laboratory studies reveal a white blood cell count of 7,000/µL, hemoglobin of 15 mg/dL, and a C-reactive protein of 0.4 mg/dL. Testing for antitissue transglutaminase antibodies is negative. Which of the following pharmacologic interventions is most beneficial for abdominal pain associated with the most likely diagnosis?
Dicyclomine dx: Irritable bowel syndrome Tx: lifestyle and dietary modification (low FODMAP diet), can consider antispasmodics, antidepressants. Constipation predominant: psyllium, osmotic laxatives, lubiprostone Diarrhea predominant: loperamide, cholestyramine The pharmacologic therapy should be targeted at the patient's most prominent underlying symptom (i.e., abdominal discomfort, constipation, or diarrhea). Antispasmodic agents, such as dicyclomine or hyoscyamine, are first-line agents for patients with abdominal discomfort or pain due to irritable bowel syndrome. These medications are used as needed to provide short-term relief and can also be used in anticipation of known stressors. Patients with irritable bowel syndrome who do not improve with antispasmodic medications and patients with comorbid depression can be treated with tricyclic antidepressants, such as amitriptyline, nortriptyline, and desipramine. Some of the dietary modifications that can improve irritable bowel syndrome symptoms include a regular meal pattern, avoidance of large meals, and reduced intake of caffeine and gas-producing foods, such as beans, cabbage, and onions. The FODMAP diet
A 32-year-old man presents to his primary care provider with complaints of chest burning for the past 2 months. He reports he has a burning sensation in his chest that seems to occur within 1 hour of eating a fatty or spicy meal. The symptoms occur once or twice each week and are worse when he lies down. He has been taking famotidine twice daily for the past 3 weeks but is still having symptoms. Which of the following is the best next step in the pharmacologic management of this patient?
Discontinue famotidine and begin omeprazole once daily Which electrolyte abnormality has the United States Food and Drug Administration associated proton pump inhibitor therapy with? Answer: Hypomagnesemia.
A 19-year-old woman presents with severe pelvic pain, nausea, vomiting, and fever. She also reports purulent vaginal discharge. A pelvic exam reveals bilateral adnexal tenderness and a positive chandelier sign. A urine pregnancy test is negative. After evaluation in the emergency department, she is admitted, treated with cefoxitin intravenously every 6 hours, and improves over the next 24 hours. What medication regimen should be prescribed for her upon discharge home?*****
Doxycycline for 14 days dx:Pelvic inflammatory disease (PID) pelvic Inflammatory Disease (PID) History of multiple sexual partners or unprotected intercourse Lower abdominal pain, cervical motion tenderness, painful sexual intercourse PE will show mucopurulent cervical discharge Most commonly caused by Chlamydia trachomatis Outpatient treatment is ceftriaxone + doxycycline + metronidazole Fitz-Hugh-Curtis syndrome: perihepatitis + PID What medication regimen is administered to pregnant patients with pelvic inflammatory disease? Answer: Intravenous ceftriaxone and azithromycin (instead of doxycycline).
A 33-year-old woman is found to have a low serum thyroxine level, an elevated thyroid-stimulating hormone level, and a positive thyroid peroxidase antibody on her laboratory findings. Which of the following physical exam findings is classically found in untreated patients with this condition?
Dull facial expression, dry skin, coarse and brittle hair, and decreased deep tendon reflexes dx:Hashimoto disease #hypothyroidism Treatment of choice is levothyroxine, and the dose should be titrated until the patient's TSH is within target range. Patients should have their TSH checked every 6 weeks after initiating treatment or changing levothyroxine doses. What is the main organ responsible for the conversion of T4 to T3? Answer: The liver.
A 64-year-old man with a 30 pack-year smoking history presents to the clinic complaining of shortness of breath on exertion and a chronic cough. He is afebrile and vital signs are within normal limits except for an oxygen saturation of 94%. Physical exam reveals bilateral rhonchi in the lower lung fields. Which of the following pulmonary function test results is indicative of his most likely pathology? AFEV1 over 70% of the predicted value BFEV1/FVC under 0.7 CFully reversible airflow limitation after bronchodilator therapy DHigh diffusing capacity for carbon monoxide
EV1/FVC under 0.7 dx:Chronic obstructive pulmonary disease (COPD) (look at image ) Treatment of COPD depends on the severity of the disease and the number of exacerbations and hospitalizations per year. Mild disease may be treated with short-acting bronchodilators. Moderate disease can be treated with long-acting bronchodilators, long-acting muscarinic receptor antagonists, or inhaled steroids. Severe disease and exacerbations may require hospitalization, antibiotic therapy, and oral steroids. Prevention of COPD is through smoking cessation and avoidance of environmental pollutant exposure. What radiographic findings are consistent with chronic obstructive pulmonary disease? Answer: Flat diaphragm, increased retrosternal lung space, rapidly tapering vasculature, and bullae.
59-year-old man presents to his primary care provider with increasing dyspnea on exertion and fatigue for the past 3 months. The patient states he has had to sleep in his recliner because he has difficulty breathing and occasionally wakes up gasping for air when he lies flat on his bed. Cardiac and pulmonary auscultations are unremarkable. Bilateral 1 pitting edema is noted to the distal extremities. Which of the following is the next step to confirm the suspected diagnosis? AEchocardiography BElectrocardiogram CNatriuretic peptide level DRight heart catheterization
Echocardiography dx: Heart failure The gold standard for diagnosing or excluding heart failure is a right heart catheterization, which evaluates cardiac filling pressure at rest and exercise What is the name of the horizontal lines on a chest radiograph resulting from interstitial edema that may be seen in heart failure patients? Answer: Kerley B lines. Pharmacologic medications typically include an angiotensin-converting enzyme (ACE) inhibitor (e.g., lisinopril) and a diuretic, such as a thiazide (e.g., hydrochlorothiazide) or loop diuretic (e.g., furosemide). Beta-blockers (e.g., metoprolol) have been shown to improve LVEF and decrease the incidence of dysrhythmias. Implantable cardioverter-defibrillators are considered in patients with an LVEF < 35%. Coronary revascularization is indicated for reversible ischemia, and refractory cases may be considered for cardiac transplantation. ====================================================== An electrocardiogram (B) and natriuretic peptide levels (C) are both part of the initial workup for heart failure but are not sufficient as individual tests to confirm the diagnosis of heart failure. A right heart catheterization (D), although considered the gold standard, is rarely used to diagnose heart failure because it is an invasive test. An echocardiogram is noninvasive and in most cases is sufficient to make the diagnosis.
A 29-year-old woman presents to the clinic complaining of a fainting spell yesterday while at home. She states she got up from the table after studying for some time and, upon rising, felt hot and nauseous and woke up later on the ground. She reports no loss of bowel or bladder control or any subsequent fatigue or confusion. She has no history of head trauma, is taking no medications, and her pregnancy test was negative. This is her first syncopal episode. Vital signs are within normal limits. Which of the following diagnostic studies is essential in the workup of this patient's syncope? ABasic metabolic panel BComputed tomography of the head CElectrocardiogram DElectroencephalogram What are some cardiac dysrhythmias that may be seen on electrocardiogram in patients with syncope?
Electrocardiogram What are some cardiac dysrhythmias that may be seen on electrocardiogram in patients with syncope? Answer: Brugada syndrome, short QT syndrome, long QT syndrome, preexcitation, and heart block. --------------------------------------- basic metabolic panel (A) may be of some benefit in the evaluation of a patient with syncope if the history and physical exam lead the clinician to suspect dehydration. Computed tomography of the head (B) is not necessary in this patient because she has no signs or history of head trauma Electroencephalogram (D) is used to rule out seizure disorder in patients with a history that may indicate seizure instead of syncope as the reason for collapse. This patient had a loss of consciousness that was unwitnessed, so electroencephalography may be necessary in the future if more events take place. However, with no loss of bowel or bladder control and no postictal symptoms, her history is more consistent with orthostatic syncope than a seizure.
A 40-year-old man with no significant past medical history presents to the office for routine follow-up. He has no complaints. His vitals reveal a temperature of 98.6°F, heart rate of 70 beats per minute, and blood pressure of 164/96 mm Hg. His blood pressure was 160/90 mm Hg at his previous appointment several months ago in which the importance of diet and exercise was discussed. Which of the following is the most appropriate therapy to initiate at this time?
Enalapril dx:hypertension, defined as a systolic blood pressure > 130 mm Hg or diastolic blood pressure > 80 mm Hg. Stage 1 hypertension is described as a systolic blood pressure ranging from 130 to 139 mm Hg or diastolic blood pressure ranging from 80 to 89 mm Hg. Stage 2 hypertension is described as a systolic blood pressure ≥ 140 mm Hg or a diastolic blood pressure ≥ 90 mm Hg. Patients with hypertension are often asymptomatic unless hypertension has led to end-organ damage. Which first-line pharmacologic class of medications used to treat hypertension is most notorious for causing a cough? ================================================ Diltiazem (A) is a nondihydropyridine calcium channel blocker. Nondihydropyridine calcium channel blockers cause less vasodilation compared to dihydropyridine calcium channel blockers and thus have less of an effect on lowering blood pressure. They have more of an effect on the heart rate compared to the blood pressure, which is why it is commonly used in dysrhythmias such as atrial fibrillation. Dihydropyridine calcium channel blockers, such as amlodipine and nifedipine, are preferred agents for the treatment of hypertension. Hydralazine (C) is a vasodilator that decreases systemic resistance. It is not recommended for initial management of blood pressure but may be considered as additional therapy for resistant hypertension in patients who do not respond adequately to combination therapy with first-line medications. It is also a relatively common antihypertensive used in pregnant patients. Metoprolol tartrate (D) is a beta-blocker. Like the nondihydropyridine calcium channel blockers, it has more of an effect on the heart rate compared to the blood pressure, and it is also commonly used in dysrhythmias. It is not recommended as an antihypertensive agent in the absence of specific comorbidities (such as ischemic heart disease or dysrhythmias).
A 55-year-old woman with a history of smoking presents to the clinic with upper abdominal pain, nausea, and heartburn for 2 weeks. She reports the pain is worse at night and a few hours after meals. Physical exam reveals mild epigastric tenderness to palpation. Urea breath test is negative. What is the best way to diagnose this condition?
Endoscopy with biopsy dx: Peptic ulcer disease (PUD) h pylori
62-year-old woman presents to the urgent care with complaints of four episodes of vertigo over the past 2 weeks. She states the episodes tend to occur early in the morning when she gets out of bed and last about 20 seconds each. She reports no tinnitus, recent trauma, or recent surgery. Physical examination is unremarkable, with no neurological deficits. The Weber test produces equal sounds bilaterally, and the Rinne test results in air conduction greater than bone conduction bilaterally. Which of the following clinical interventions is most likely to resolve the patient's symptoms?
Epley maneuver Benign paroxysmal positional vertigo is a peripheral vestibular disorder most common in women > 60 years old. It is caused by loose calcium crystals known as otoconia from the utricle that end up in the posterior semicircular canal. Episodes of vertigo last < 1 minute and are provoked by sudden changes in head position. Hearing loss and neurologic changes are absent. Without treatment, episodes of vertigo may recur for months. Vertigo beginning within 30 seconds and resolving within 30 seconds of performing the Dix-Hallpike maneuveris diagnostic. Horizontal nystagmus may or may not be present, but vertical nystagmus would suggest the presence of a central lesion and is not present with benign paroxysmal positional vertigo. The Epley maneuver is a particle repositioning maneuver that has a high success rate for treating benign paroxysmal positional vertigo, and most patients will require fewer than three treatments.
A 22-year-old man presents to his primary care provider with a nonpruritic, nontender rash on his hands and the soles of his feet that appeared yesterday. He reports hiking in the northeastern United States 2 weeks ago. His past sexual history is significant for unprotected intercourse with multiple male and female partners. His physical examination is unremarkable except for the rash on his palms and soles, which is shown in the image above. Which of the following infections in the patient's history is associated with the most likely diagnosis? tx vs prophylatic tx?
Erythema multiforme is a type IV hypersensitivity reaction associated with herpes simplex virus self-limiting. Supportive care with oral antihistamines and low-potency topical corticosteroids will alleviate skin discomfort. Painful oral lesions may be treated with high-potency corticosteroid gels and mouthwashes containing lidocaine Prophylactic treatment with acyclovir should be considered in patients with recurrent herpes-associated erythema multiforme outbreaks.
A 19-year-old G2P1 woman at 12 weeks gestation presents to her primary care provider with complaints of worsening pustules on her face. Physical examination reveals the presence of cystic, inflammatory lesions on the face, neck, upper chest, and back. Which of the following treatments, along with a benzoyl peroxide wash, is most appropriate at this time? ADoxycycline BErythromycin CIsotretinoin DTretinoin
Erythromycin dx:Acne vulgaris Which type of lesion is the hallmark of acne vulgaris? Answer: Comedo.
A 65-year-old woman presents to her family doctor with complaints of odynophagia, dysphagia, and mild, intermittent retrosternal chest pain over the past week. Physical exam is unremarkable. Medications include alendronate that she started 1 week ago and lisinopril for hypertension. What is the most likely diagnosis?
Esophagitis Medications responsible for esophageal mucosal injury include antibiotics, such as tetracycline, doxycycline, trimethoprim-sulfamethoxazole, and clindamycin, nonsteroidal anti-inflammatories, bisphosphonates, potassium chloride, quinidine, vitamin C, and iron compounds. Injury is more common in women, is more likely to occur if pills are swallowed without water or while supine, and is more common in patients who are hospitalized or bed bound. Patients present with symptoms of dysphagia, odynophagia, or retrosternal chest pain often several hours after taking the pill. Medication-Induced Esophagitis Remain upright after taking medications Take pills with sufficient quantities of water Use liquid formulation of medication when possible Attempt to switch the offending agent to a different medication
A 60-year-old man with a 35 pack-year history of smoking presents to your office with complaints of shortness of breath, chronic cough, and sputum production, worsening over the past year. Physical exam reveals decreased breath sounds and wheezes on auscultation of the lungs and pulse oximetry of 93%. Which of the following would be seen on spirometry?
FEV1/FVC ratio decreased, not reversing to normal with bronchodilator dx: copd Which hereditary disorder should patients with suspected COPD be evaluated for? Answer: Alpha-1 antitrypsin deficiency.
A 40-year-old man with a history of poorly controlled type 2 diabetes mellitus, obesity, and noncompliance with treatment presents to your office for his diabetes visit. His prescribed medications include metformin 1,000 mg twice daily, atorvastatin 80 mg daily, and lisinopril 10 mg daily. He says he is in his normal state of health and reports no symptoms. Lipid panel results include a triglyceride level of 900 mg/dL. Which of the following is the most appropriate therapy? what level increases risk of pancreatitis and atherosclerotic cardiovascular disease?
Fenofibrate first-line therapy due to the ability to use it with statins. dx:Hypertriglyceridemia is defined as a blood triglyceride level > 150 mg/dL severe being > 885 mg/dL. Hypertriglyceridemia PE: Skin xanthomas, tendinous xanthomas, corneal arcus Elevated triglycerides ≥ 150 mg/dL increase risk of cardiovascular disease Severely elevated triglycerides ≥ 500-1000 mg/dL increase risk of pancreatitis TreatmentMild to moderate elevationReduction in weight and carbohydrate intake, aerobic exerciseStatins +/- icosapent ethyl when indicated to reduce ASCVD riskSevere elevation: fibrates, mega-3 fatty acids, niacin, avoidance of alcohol The goals of treating elevated triglycerides are to decrease the risk of pancreatitis and atherosclerotic cardiovascular disease. All patients with hypertriglyceridemia should be counseled on lifestyle interventions. First-line behavioral interventions include weight loss for overweight or obese patients, aerobic exercise Drug therapy specific to lowering triglycerides should be started in patients found to have levels in the severe range. gemfibrozil, have a higher risk of muscle toxicity when taken together with a statin so are not recommended as first line. diagnosis of hypertriglyceridemia versus that of mixed lipoproteinemia. Hypertriglyceridemia is diagnosed in patients with a fasting triglyceride level > 200 mg/dL without an elevated low-density lipoprotein cholesterol reading. When the low-density lipoprotein is also elevated, this points to a diagnosis of mixed lipoproteinemia.
A 75-year-old man with a history of benign prostatic hyperplasia presents with worsening hesitancy, double voiding, and postvoid dribbling. A digital rectal exam reveals a smooth, firm, elastic, and enlarged prostate that has increased in size from a previous evaluation done 6 months ago. Which one of the following medications can reduce the size of the prostate and improve symptoms?
Finasteride dx: Benign prostatic hyperplasia (BPH) Benign Prostatic Hyperplasia (BPH) Risk factors: advancing age Sx: hesitancy, intermittence or incontinence, frequency or fullness, urgency, nocturia (HI FUN) PE: smooth, firm, mobile prostate without any nodules or indurations Dx: digital rectal exam, UA to rule out alternate causes Caused by stromal and epithelial cell growth in the transitional zone of the prostate Tx: alpha-blockers, 5-reductase inhibitors, surgery (TURP) Clinical manifestations of BPH are divided into obstructive (hesitancy, decreased force/caliber of urine, postvoid dribbling, a sensation of incomplete emptying, double voiding, straining to urinate) and irritative (dysuria, frequency, urgency, nocturia) symptoms. A digital rectal exam may reveal a smooth, firm, elastic enlargement of the prostate. If the induration of the prostate is palpated, prostate cancer should be excluded by further testing. First-line treatment for BPH is reassurance and lifestyle modification (decrease fluid intake before bed, reduce caffeine or alcohol intake, and timed voiding). Medical therapy (using alpha-blockers, 5-alpha-reductase inhibitors, phosphodiesterase-5 inhibitors, combination therapy, phytotherapy) is the mainstay of treatment. Finasteride is a 5-alpha-reductase inhibitor that blocks the conversion of testosterone to dihydrotestosterone. It works on the epithelial component of the prostate, reducing its size and improving symptoms.
A 7-year-old boy presents to the pediatric clinic with a focal red and painful bump on his right upper eyelid. Neither the patient nor his mother report any trauma to his eye, and he first noticed it when he woke up this morning. Which of the following describes the most likely condition?
Focal purulent inflammatory process of the eyelid Staphylococcus aureus. Hordeolum presents as a small, erythematous, and painful pustule. There are external and internal hordeolums. The external hordeolum grows from the base of the eyelash, while the internal hordeolum is located beneath the eyelid Hordeolum is a clinical diagnosis. The treatment involves warm compresses. Topical antibiotics are often used to treat hordeolum but have little benefit =============================================== Diffuse inflammation of the eyelid (A) is called blepharitis, which is often associated with rosacea or seborrheic dermatitis. Blepharitis presents with bilateral eyelid erythema, edema, and pain. Increased pressure within the eye (C) is called glaucoma. There are acute angle-closure glaucomas and open-angle glaucomas. Angle-closure glaucoma is marked by a sudden and rapid rise in intraocular pressure. Inflammation of the conjunctiva of the eye (D) is called conjunctivitis and may be caused by bacterial, viral, or allergic causes.
A 61-year-old woman with a history of alcohol use disorder and Crohn disease presents to the office for her well-woman visit with concerns of fatigue and feeling mentally foggy. On physical examination, she appears mildly pale, and loss of papillae is noted on the tongue. Baseline blood work is obtained. Her hemoglobin and hematocrit are mildly decreased at 11 g/dL and 32%, respectively. Her mean corpuscular volume is 117 fL. Vitamin B12 and folate levels are borderline low, but methylmalonic acid and homocysteine are elevated. What physical exam finding would be expected on examination given the patient's history and laboratory results?
Gait ataxia dx: vit B12 deficiency may result in decreased intake of vitamin B12 (such as veganism), decreased absorption (prior bariatric surgery, celiac disease, Crohn disease), and pernicious anemia. severity (fatigue, shortness of breath), glossitis, and neuropsychiatric symptoms, particularly paresthesia,numbness, and gait abnormalities. Gait abnormalities develop as the vitamin B12 deficiency results in subacute combined degeneration of the spinal cord, predominantly affecting the posterior columns of the spine responsible for proprioception, light touch, and vibration. Other neuropsychiatric symptoms may include depression, insomnia, cognitive impairment, and dementia. Symptoms of vitamin B12 deficiency often take years to develop but can occur rapidly in the setting of exposure to nitrous oxide.
A 22-year-old woman presents to the office for evaluation. She was in the office 3 days ago for her first urinary tract infection and was prescribed an antibiotic that she had never received before. She has been taking the antibiotic as prescribed but is now experiencing fatigue and yellowing of the skin. Her urine has also become dark in appearance. She questions whether the antibiotic is working or the infection is worsening. On physical examination, she has left upper quadrant abdominal tenderness. Sclera are mildly icteric. Peripheral smear is shown above. What is the most likely diagnosis?
Glucose-6-phosphate dehydrogenase deficiency X-linked inherited red blood cell enzyme disorder, which makes the red blood cell prone to oxidative injury. Oxidative stressors include medications (dapsone, antimalarial drugs, nitrofurantoin), certain foods (fava beans), infections, and chemicals (naphthalene). Approximately 48-96 hours after exposure to an oxidative agent, the patient experiences acute hemolysis, which manifests as fatigue, jaundice, pallor, abdominal pain, and dark urine. Neonates with severe G6PD may present in the newborn period with kernicterus. Upon laboratory evaluation, there is decreased hemoglobin, increased reticulocytes, increased bilirubin, decreased haptoglobin, and increased lactate dehydrogenas What supplement should be initiated in patients with chronic hemolysis? Answer: Folic acid.
A 20-year-old man who is unvaccinated and currently residing in a college dormitory presents with fever, stiff neck, and altered mental status. His physical examination is concerning for the rash noted in the image above. His diagnostic workup includes CT scan of the head, which is normal, and a lumbar puncture with cerebrospinal fluid analysis and culture and sensitivity, which reveals pleocytosis with predominant neutrophils, decreased glucose concentration, and elevated protein concentration. An opening pressure is elevated. Which of the following Gram stain results would you expect to find in this patient? At what age should the quadrivalent meningococcal conjugate vaccine be administered
Gram-negative diplococci Neisseria meningitidis. Bacterial meningitis i Neisseria meningitidis Meningitis Young Outbreaks in close quarters PPX: rifampin At what age should the quadrivalent meningococcal conjugate vaccine be administered? Answer: 11-12 years of age. A booster can be provided at age 16.
A 65-year-old man presents to the emergency department with decreased visual acuity. He describes the vision change as moving spots and flashing lights in the visual field of his right eye. Which of the following additional findings is most suggestive of the suspected diagnosis?
Gray curtain moving across the visual field Retinal Detachment Patient presents with painless loss of vision, floaters, flashing lights, curtain-lowering sensation PE: retina appears hazy gray with white folds Management is stat ophthalmology consult
A 42-year-old woman presents to her primary care provider with complaints of worsening left hip pain and difficulty ambulating over the past 4 weeks. She reports no history of falls or trauma. She does have a history of bilateral osteoarthritis of the knees, plantar fasciitis, and low back pain. Physical examination reveals morbid obesity and tenderness to palpation over the left lateral hip. No erythema or swelling is appreciated. Passive and active abduction of the left hip reproduces the pain over the lateral hip without causing inguinal pain. Her standing anteroposterior pelvic radiograph is shown in the image above. Which of the following is the most likely diagnosis?
Greater trochanteric pain syndrome Initial treatment includes nonsteroidal anti-inflammatory drugs (NSAIDs), physical activity modifications, and management of comorbid conditions. Glucocorticoid injections may be given if more immediate relief is required or if NSAID treatment is inadequate
A 30-year-old man presents to his primary care provider with complaints of gradually worsening shoulder, back, and hip pain for the past 7 years. He states the pain is alleviated by mild activity and does not improve with rest. The pain is most severe at night, and he feels very stiff in the morning. Review of systems is significant for recurrent bouts of acute, unilateral eye pain accompanied by photophobia and blurred vision. Physical examination reveals hyperkyphosis. Radiographs are significant for arthritic changes in the shoulders and hips bilaterally and the presence of syndesmophytes bridging across multiple vertebrae. Which of the following human leukocyte antigen markers is most likely to be positive in this patient?
HLA-B27 dx: ankylosing spondylitis age of onset is around 10-30 years of age. Signs and symptoms include insidious onset of back pain, night pain, and early morning stiffness. Pain typically improves with activity and is not relieved by rest. Hyperkyphosis may be seen within the first 10 years of disease. sacroiliitis is a hallmark of ankylosing spondylitis and occurs early on in the disease. It is usually the earliest radiographic finding. Enthesitis (inflammation at the insertion site of a tendon or ligament on bone) may also be present in ankylosing spondylitis. Other associated conditions include anterior uveitis, peripheral arthritis, temporomandibular joint pain, dactylitis, psoriasis, inflammatory bowel disease, osteopenia, pulmonary disease, and cardiovascular disease (e.g., acute coronary syndrome, thromboembolism, stroke) bamboo spine appearance. Fusion of the posterior facet joints may also be present. The first-line treatment for ankylosing spondylitis is nonsteroidal anti-inflammatory drugs. Corticosteroids should generally be avoided, as they have minimal impact and may worsen osteopenia. The tumor necrosis factor inhibitors etanercept, adalimumab, or infliximab may be used for patients who do not respond to NSAIDs.
A 2-year-old boy presents to the pediatric clinic with right ear pulling. On exam, the right tympanic membrane is bulging. Which of the following pathogens is most likely to cause this condition? tx vs alternative when anaphylaxic?
Haemophilus influenzae and Streptococcus pneumoniae are the most common bacterial causes of acute otitis media. Acute Otitis Media Patient will be an infant or young child Ear pain, fever, URI symptoms PE will show TM erythema and decreased mobility of TM Most common bacteria isolated: H. influenza (nontypable) (previously S. pneumoniae but has decreased post-PCV13 vaccination) Treatment is amoxicillin; consider amoxicillin-clavulanate in adults due to drug resistance Consider period of observation if ≥ 2 years and immunocompetent with mild symptoms Which antibiotics can be used to treat acute otitis media in children with anaphylactic reactions to amoxicillin? Answer: Azithromycin or clindamycin.
A 64-year-old man is establishing care in the family medicine office as a new patient. No prior medical records are available. His review of systems is pertinent for intermittent headaches and aquagenic pruritus. Upon further questioning, the patient reports he was previously diagnosed with a hematologic condition, but he cannot remember the name. On physical examination he is noted to have a ruddy facial plethora and palpable splenomegaly. Laboratory studies reveal a hemoglobin of 17.8 g/dL. What follow-up question would be pertinent to ask the patient, given this information? A"Do you get regular blood transfusions?" B"Do you have the HLA-B27 mutation?" C"Have you ever had a blood clot before?" D"Have you previously been exposed to Agent Orange?"
Have you ever had a blood clot before?" . Polycythemia vera is a myeloproliferative disorder that results in an increase in red blood cell mass. stuck in stcky area # prone to clots Patients with polycythemia vera may present with vasomotor symptoms, such as erythromelalgia and aquagenic pruritus. Headache, dizziness, and early satiety may also be reported. Physical examination may reveal facial plethora (ruddy cyanosis)and splenomegaly. Many patients may be asymptomatic, with the diagnosis being made based on incidental findings of elevated hemoglobin and hematocrit on a complete blood count (CBC). JAK2 gene mutation some patients presenting with Budd-Chiari syndrome. In addition to the risk of thrombosis, patients also have an increased risk of malignancy due to a risk of hematologic transformation to myelofibrosis, acute myeloid leukemia, and myelodysplastic syndrome. There is no curative therapy for polycythemia vera What is Budd-Chiari syndrome? Answer: A disorder that occurs when the hepatic venous outflow is obstructed due to a thrombus within a hepatic vein.
A 23-year-old woman presents to her orthopedic physician assistant with complaints of right heel pain for the past month. She states the pain is the most severe upon standing after a period of immobility. She is in excellent physical condition and has run four marathons in the past 2 years. She states she has been fitted for shoes by the running store because she has excessive pronation. Physical examination reveals tenderness to the medial plantar calcaneal region and excessive pronation with ambulation. Her right lateral foot radiograph is unremarkable. Which of the following clinical interventions is most appropriate at this time?
Heel shoe inserts dx:Plantar fasciitis Sx: heel and foot pain when first stepping out of bed or after period of inactivity, improves with walking orstretching the calf PE: tenderness over the sole of the foot near the calcaneus Tx: stretching plantar fascia and calf, heel inserts Conservative treatments include activity modification, rest, ice, stretching, and nonsteroidal anti-inflammatory drugs. Heel shoe inserts ma Patients who do not respond to the previously mentioned treatments may be considered for extracorporeal shock wave therapy or radiation therapy. Surgical plantar fascia and gastrocnemius release is considered for patients with continued pain despite 6-12 months of nonsurgical management. True or false: an ultrasound on a patient with plantar fasciitis will typically show a plantar fascia thickness > 4 mm. Answer: True.
A 42-year-old man presents to his primary care provider with complaints of recurrent nausea, abdominal pain, and dyspepsia for the past 5 years. His medical history is significant for recurrent gastric ulcers treated with long-term omeprazole. Endoscopy reveals mucosal erythema, friable gastric mucosa, and diffuse antral nodularity. Gastric pH is 3.5. A rapid urease test of the biopsied specimen is positive. His blood work is significant for iron deficiency anemia. Which of the following is the most likely diagnosis?
Helicobacter pylori gastritis Patients with chronic H. pylori gastritis are at increased risk of gastric and duodenal ulcers. They may also present with extraintestinal conditions such as iron deficiency anemia, idiopathic thrombocytopenic purpura, and vitamin B12 deficiency. Iron deficiency anemia occurs secondary to decreased iron absorption from H.pylori-associated gastric atrophy and hypochlorhydria (gastric pH > 3.0). Long-term proton pump inhibitor (PPI) use is associated with an increased risk of H. pylori gastritis. Triple therapy includes clarithromycin, amoxicillin, and a PPI for 14 days. Penicillin-allergic individuals can be treated with metronidazole instead of amoxicillin. Quadruple therapy consists of bismuth subsalicylate, metronidazole, tetracycline, and a PPI for 14 days. Gastritis Diagnosis is confirmed histologically Most common acute cause: NSAIDs > alcohol Type A chronic: pernicious anemia Type B chronic : H. pylori Type A gastritis, the most common type, is a mild diffuse gastritis characterized by inflammation of the gastric fundus that is associated with pernicious anemia. Type B gastritis is characterized by inflammation of the gastric antrum. The gastric body, where gastric acid is secreted, is unaffected. Patients with type B gastritis have increased gastrin and acid production. Type B gastritis is associated with peptic ulcers and adenocarcinoma. Type C gastritis, the more rare type, is characterized by inflammation predominantly in the gastric body, resulting in destruction of acid-secreting glands, decreased gastric acid, mucosal atrophy, and intestinal metaplasia. It is associated with an increased risk of gastric ulcers and gastric cance
A 28-year-old woman presents to the gastroenterology clinic for reevaluation of painless rectal bleeding with bowel movements. She has no personal or family history of colorectal cancer or inflammatory bowel disease. On physical exam, she has visible hemorrhoids originating below the dentate line. Her hemoglobin level is 14.0 mg/dL. She has been treating her condition with increased water intake, increased dietary fiber, sitz baths, and docusate for 12 weeks without improvement. Which of the following is the recommended treatment?
Hemorrhoidectomy (look at image too) The treatments for irritation and pruritus include sitz baths, local anesthetics (benzocaine rectal ointment), topical corticosteroids, vasoactive substances (nitroglycerin or phenylephrine), and zinc oxide. It is important for patients to clean and dry the surrounding area prior to applying topical treatments. Patients with persistent hemorrhoids sometimes need procedural treatment. External hemorrhoids that fail conservative treatment require surgery with hemorrhoidectomy. What is the recommended procedural treatment for patients with refractory grade II internal hemorrhoids who are on anticoagulants? Answer: Sclerotherapy because the risk of bleeding is very low.
A 75-year-old man with a history of hypertension, dyslipidemia, and coronary artery disease presents to his primary care provider with worsening shortness of breath. He reports increasing fatigue, a productive cough, and trouble sleeping at night. He feels like he is unable to breathe when lying flat and has been sleeping in a recliner chair. Physical exam reveals 2+ pitting edema, bibasilar crackles, and elevated jugular vein distention. Which of the following physical exam findings would you expect to note on this patient?
Hepatojugular reflux dx: Acute decompensated heart failure, also known as congestive heart failure, The echocardiogram shows reduced ventricular function and reduced ejection fraction. A chest X-ray is useful in making the diagnosis of congestive heart failure. The chest X-ray may reveal vessel cephalization, Kerley B lines, a "batwing" pattern in the hilar area, and pulmonary edema. Regarding lab tests, B-type natriuretic peptide is elevated Medical management of congestive heart failure includes diuretics, nitrates, oxygen, and positioning. If patients are not perfusing peripheral tissues well, inotropic support (e.g., digoxin or dobutamine) is indicated. Lifestyle management includes salt restriction and cardiac rehabilitation. Beta-blockers should be avoided in patients with congestive heart failure. If the patient is on a beta-blocker, the dose should be lowered until the patient is back to baseline. Patient presents with exertional dyspnea, orthopnea, paroxysmal nocturnal dyspnea PE will show pitting edema, S3 heart sound Labs will show ↑ BNP Chest X-ray will show cardiomegaly, cephalization, Kerley B lines, effusions Diagnosis is made by echo (most useful study) TreatmentBPAP: ↑ oxygenation, ↓ work of breathing, ↓ preload, afterloadNitroglycerin: ↓ preload, afterloadFurosemide: diuresisHypotension without signs of shock: dobutamine (may worsen hypotension)Severe hypotension with signs of shock: norepinephrine (↑ systemic vascular resistance, ↑ HR, ↑ BP, ↑ myocardial O2 demand)
A 55-year-old man presents to his primary care physician for a physical. He reports it has been a number of years since he has been to the doctor. The patient has no history of adult illness and takes no medication. He says he is drinking three beers most nights of the week. Laboratory values include ALT of 80 U/L, AST of 70 U/L, and alkaline phosphatase of 230 U/L. Liver biopsy is consistent with cirrhosis, and Prussian blue stain shows intense iron staining in the hepatocytes. Serum ferritin is 2,500 ng/L. Which of the following is the most likely diagnosis? these people are at increase risk of what infection ?
Hereditary hemochromatosis disorder of iron storage. It is an autosomal recessive disease HFE gene on chromosome 6. Early symptoms are nonspecific and can include fatigue and arthralgia. In later stages, arthropathy, hepatomegaly, skin pigmentation, hepatic dysfunction, cardiac enlargement or failure, diabetes mellitus, or erectile dysfunction may be present. Cirrhosis is more likely to develop in patients with concomitant alcohol use disorder, Routine iron studies include serum iron, total iron binding capacity, transferrin saturation (TSAT), and serum ferritin. A TSAT indicative of excessive iron stores is greater than 40%, and a ferritin level above the upper limit of normal is indicative of excessive iron stores. Patients with elevated liver enzymes should undergo a liver biopsy to assess the extent of fibrosis and to look for the presence of cirrhosis and other causes of liver disease. Liver biopsy will show intense iron deposits in the hepatocytes when exposed to Prussian blue stain. Patients who have evidence of organ injury, significant iron on imaging likely to cause injury, or ferritin greater than 1,000 ng/mL should start treatment with phlebotomy to prevent progression. Phlebotomy is performed to remove 1 unit of blood over 2 or more hours. Target of treatment is for ferritin to be within normal limits. Patients with ferritin levels less than 500 ng/mL and no tissue iron deposits are monitored on a yearly basis for increasing ferritin level, TSAT, or evidence of disease progression. Iron chelation is an alternative treatment for patients who also have anemia or do not tolerate continuous phlebotomy. alcohol avoidance, screening for hepatocellular carcinoma in patients with cirrhosis, avoiding vitamins or supplements containing iron, and avoiding uncooked seafood as patients with hemochromatosis are at increased risk of infections with siderophilic organisms (Yersinia enterocolitica, Listeria monocytogenes, Vibrio vulnificus). Untreated hemochromatosis can lead to an early death, but with the advent of early detection and treatment, survival appears to be normal in patients with a serum ferritin of less than 2,000 ng/mL at the time of diagnosis.
A 64-year-old lawyer presents to his primary care doctor with concerns of chest "flutters." For the last 2 weeks, he reports daily symptoms of chest flutters or palpitations that occur both during exercise and at rest. His only medication is a daily multivitamin. Today, his vital signs show a blood pressure of 112/71 mm Hg, pulse of 82 bpm, temperature of 98.6°F, and respirations of 12 breaths/minute. He is athletically built and in no acute distress. On auscultation, his lungs are clear. Cardiac exam reveals a regular rate and rhythm without murmur. Carotid pulses are equal and strong without bruits or jugular venous distention. An ECG was performed and showed no abnormalities. Which of the following is the best intervention at this time?
Holter monitors the device is more likely to identify spontaneous, variable cardiac dysrhythmias or other abnormalities. There are many indications for Holter monitor use. They are commonly used to determine adequate rate control in patients with atrial fibrillation, to assess for premature ventricular or atrial beats, and to evaluate nonsustained ventricular tachycardia in hypertrophic cardiomyopathy. ======================================================== Event (loop) monitors (B) record events only at the time of symptoms (unlike Holter monitors, which constantly measure and record heart activity over a set period of time). They are indicated for evaluating patients with weekly or twice weekly symptoms. They are also used to assess for the source of dysrhythmia in instances of palpitations, dizziness, syncope, or presyncope
A 25-year-old woman presents with neck pain and stiffness for 1 week. The patient denies any acute injury or precipitating factor and has never had symptoms like this before. The patient works as a secretary and has recently been working long hours. On physical exam, there is tenderness of bilateral paraspinal muscles. The patient has full but painful range of motion. Upper extremity sensation is intact, and she has full strength of all myotomes. Which of the following is the most appropriate next step in management?
Ibuprofen dx: Cervical strain presents with symptoms of pain or stiffness with neck movement. identify any red flags, such as recent major trauma, weakness, gait changes, bowel or bladder dysfunction, Lhermitte phenomenon, fever or chills, history of IV drug use, immunosuppression, unexplained weight loss, history of cancer, visual changes, or anterior neck pain. . Patients should be educated about the natural history of neck pain and about appropriate posture when sitting or standing, as well as sleep posture where the neck should be aligned with the body. A home exercise program includes early mobilization to maintain cervical range of motion with gentle stretching. First-line pharmacotherapy is with nonsteroidal anti-inflammatories, such as ibuprofen, or acetaminophen A nonbenzodiazepine skeletal muscle relaxant can be used for symptoms not well managed with nonopioid analgesics or for cervical spasm. Pain and stiffness can last for up to 6 weeks. Patients with red flags, moderate to severe pain not responding to conservative management in 6 weeks, or progressive neurological dysfunction should undergo MRI for further evaluation.
A 35-year-old man presents to the clinic complaining of a progressive chest pain that he has had for about 1 week. He describes the pain as a severe, sharp pain to the center of his chest. He states the pain worsens when he takes a deep breath and when lying down. He has been leaning forward because he says this is the only position in which the chest pain gets better. On physical exam, there is a pericardial friction rub. The patient has not tried any medications to help with his pain. Which of the following is the most appropriate therapy for this patient?
Ibuprofen and colchicine acute pericarditis, which is an inflammation of the pericardium. The most common causes of acute pericarditis are viral, due to enteroviruses such as coxsackie three Ps: pleuritic, persistent, and postural, meaning it worsens when supine and improves with leaning forward. Commonly, the patient is noted to have a fever as well. On physical exam, there will be an end-expiratory pericardial friction rub that is heard best when the patient is leaning forward. An ECG will show PR elevation in aVR, PR depression, and diffuse ST elevations in the precordial leads. First-line treatment is with anti-inflammatories such as aspirin or NSAIDs plus colchicine to prevent recurrence. For refractory or severe cases, the addition of corticosteroids is appropriate due to its anti-inflammatory effects What is the most common side effect of colchicine? Answer: GI upset including diarrhea, nausea, vomiting, and abdominal cramping. --------------------------------------------------- Ibuprofen (C) should not be used alone due to the increased chance of recurrence risk if used as monotherapy. Addition of colchicine prevents recurrence.
A 55-year-old woman presents to her primary care provider with worsening right foot pain over the last 3 months. She reports the onset of her symptoms coincided with a new weight-loss program that she started 3 months ago. She states the first few steps after getting out of bed or out of a chair are always the most painful. Physical examination reveals a woman with obesity, pes planus, tight gastrocnemius muscles bilaterally, and excessive pronation with ambulation. Her foot X-ray is shown above. Which of the following clinical interventions is most appropriate at this time?
Ice, stretching, and rest dx; plantar fasciitis Risk factors include anatomical variations of the lower extremities (i.e., flat feet, high arches, tight Achilles tendon, tight gastrocnemius, tight intrinsic foot muscles, limb length discrepancy), obesity, poor-fitting shoes, prolonged standing or walking, excessive training, and improper gait (e.g., excessive foot pronation). Patients typically present with heel pain upon rising in the morning or after prolonged immobilization that improves with movement. Physical examination may reveal tenderness to palpation of the medial plantar calcaneal region, pain with ankle dorsiflexion, and tightness of the Achilles tendon or gastrocnemius. Diagnosis is primarily clinical. What medical term is used to describe the pain that occurs after a period of rest and is associated with plantar fasciitis? Answer: Poststatic dyskinesia. Plain radiographs may be used to rule out bony abnormalities. Calcaneal spurs may be seen on plain radiographs.
A 34-year-old woman with obesity presents to the primary care office for an annual exam. She has no concerns and feels healthy. She is a long-time smoker but reports cutting down to five cigarettes per day. Her family history includes hypertension, dyslipidemia, hypercholesterolemia, and breast cancer in her maternal grandmother. Her mother has obesity, smokes daily, and had a myocardial infarction at 40 years of age. What is the current recommended age for lipid screening for this patient?
Immediate screening What drug class is considered first-line pharmacotherapy for dyslipidemia that is recalcitrant to lifestyle modifications? Answer: Statins.
A 28-year-old woman presents to the clinic complaining of vaginal spotting for the past week with lower abdominal cramping and heavy bleeding for the past day. She states her last menstrual period was 60 days ago. On physical exam, her cervix is dilated to 1 cm, copious blood is seen in the vault, and products of conception are seen protruding from the os. Bimanual exam reveals a slightly enlarged, mildly tender uterus with no palpable adnexa. Which of the following is the appropriate diagnosis?
Incomplete spontaneous abortion What is a septic abortion? Answer: Retained products of conception have not been fully expelled, and endometritis develops
Which of the following best describes the effect of low serum estradiol and progesterone levels on gonadotropin-releasing hormone, follicle-stimulating hormone, and luteinizing hormone at the beginning of the menstrual cycle?
Increased gonadotropin-releasing hormone, increased follicle-stimulating hormone, and increased luteinizing hormone What amount of blood loss during a menstrual cycle defines heavy menstrual bleeding? Answer: > 80 mL.
A 21-year-old man presents to the urgent care with complaints of a pruritic rash that comes and goes. He reports the rash started 3 days ago and occurs on his trunk and lower extremities. Stroking of the skin results in localized erythematous plaques. Which of the following in the patient's history is consistent with the most likely diagnosis?
Individual lesions last < 24 hours dx: Urticaria intensely pruritic wheals or hives. The pathophysiology of urticaria is an immunologic reaction mediated by immunoglobulin E or activation of the complement cascade. What sign refers to localized urticaria caused by stroking skin lesions seen in cutaneous mastocytosis? Answer: Darier sign. Etiology: IgE-mediated reaction to an allergen Treatment: supportive care, antihistamines, glucocorticoids (if associated with angioedema)
A 42-year-old man presents to his primary care provider with acute onset of severe left toe pain. He states the pain woke him up around 4 AM after an evening of excessive alcohol intake. Physical examination reveals a swollen, erythematous, warm, and extremely tender metatarsophalangeal joint of the left great toe. Ultrasonography reveals the presence of urate crystal deposition. What is the most appropriate initial treatment for this patient?
Indomethacin dx:acute gout attack and should be treated with a nonsteroidal anti-inflammatory drug (NSAID), such as indomethacin. The metatarsophalangeal joint of the great toe is most commonly affected. Definitive diagnosis can be made with joint fluid aspirate showing sodium urate crystals, which appear as needle-shaped and negatively birefringent. Early radiographs are unremarkable, but later findings include rat-bite erosions. Management of gout includes lifestyle modifications (e.g., decrease dietary purine, decrease alcohol, decrease high fructose corn syrup, increase water intake) and avoidance of hyperuricemia medications. Medications that decrease the production of uric acid (e.g., xanthine oxidase inhibitors such as allopurinol) and increase urinary excretion of uric acid (e.g., colchicine, probenecid) are used to decrease the frequency of gout attacks. These medications may be used in combination or as monotherapy.
An 86-year-old man presents complaining of breathlessness and fever of 101°F for 3 days. He states his breathlessness began about 2 years ago and has become progressively worse. He feels well and is otherwise asymptomatic. He reports no sick contacts. His medical history includes hypertension and anxiety. He was a heavy smoker until he quit in his late 70s. He was a navy industrial worker from 1930 to 1940, during which time he traveled to many foreign countries. He reports having a chest radiograph performed in his 60s that revealed abnormalities, but he cannot remember specific details. His vitals are normal except for a temperature of 100.7°F. On physical exam, the patient appears well-developed and well-nourished. Cardiac exam reveals a regular rate and rhythm without murmurs, rubs, or gallops. There is dullness to percussion at the lung bases and decreased air movement during respiration. The patient is breathing easily without accessory muscle activation at a rate of 14 breaths per minute. Skin is without rashes. There is no nail clubbing. A chest radiograph is ordered and is shown above. Which aspect of this patient's history and physical exam is the most important risk factor for the suspected diagnosis?
Industrial worker asbestos plant workers, construction workers, firefighters, industrial workers, insulators, and factory workers. Asbestos exposure can result in asbestosis(slowly progressive diffuse pulmonary fibrosis), pleural disease, and malignancy (non-small cell lung cancer, small cell lung cancer, and mesothelioma). no curative treatment for patients with asbestosis. Management should focus on prevention of additional respiratory disorders, including smoking cessation, vaccination against pneumonia, and rapid diagnosis and treatment of respiratory infections.
A 52-year-old man presents to your office with a complaint of groin pain that worsens with coughing. He tells you that, when the pain occurs, he can see a bulge that disappears when he lies down. Which of the following is the most likely diagnosis? AEpididymitis BHydrocele CInguinal hernia DLipoma
Inguinal hernia Which type of inguinal hernia is most common? Answer: Indirect. ==================================================== Hydrocele (B) is a mass located in the inguinal canal or scrotum that transilluminates on physical exam. Lipoma (D) is a soft mass that does not change in size and is otherwise asymptomatic in nature.
post-traumatic stress disorder (PTSD) tx= What is the appropriate treatment for a patient with acute stress disorder?
Initiate both cognitive behavior therapy and pharmacotherapy with a selective serotonin reuptake inhibitor (SSRI) such as sertraline or paroxetine. SSRIs are the only FDA-approved class of medications and considered the pharmacotherapy of choice for PTSD. For patients who are experiencing insomnia, the addition of trazodone nightly may be helpful. Overall, the sooner therapy is initiated in patients with post-traumatic stress disorder, the better the prognosis. What is the appropriate treatment for a patient with acute stress disorder? Answer: Psychotherapy.
A 30-year-old woman presents to her primary care provider with complaints of widespread, achy musculoskeletal pain, fatigue, and chronic headache for the past 9 months that have severely interfered with her activities of daily living. She states her pain is worse in the morning and aggravated by minor activities. She reports no known sick contacts but states she feels like she has had the flu for 9 months. Her past medical history is significant for anxiety and depression. Physical examination is significant for muscle tenderness over the neck, shoulders, bilateral arms and legs, and upper and lower back. Which of the following is most likely to be found in the patient's history? What are the three FDA-approved medications for treatment of this dx?
Insomnia dx:fibromyalgia. Rheumatoid arthritis, hypothyroidism, anxiety, and depressionare associated with fibromyalgia. multiple tender points # look at image Risk factors: age 20-55, female sex Widespread or multisite musculoskeletal pain for > 3 months, nonrestorative sleep, and generalized fatigue PE: areas of soft tissue tenderness (newer guidelines move away from palpating specific tender points), no joint swelling Labs: normal CBC, chemistry, acute phase reactants Diagnosis is made clinically Treatment is education, exercise, antidepressants, avoid opioids [Nonsteroidal anti-inflammatory drugs, corticosteroids, and opioids are generally ineffective.] patient education, sleep hygiene, and low-impact aerobic exercise (swimming). What are the three FDA-approved medications for treatment of fibromyalgia? Answer: Duloxetine, pregabalin, and milnacipran. Fibromyalgia in the absence of other disorders will not have any objective signs of inflammation, and laboratory studies will be normal. Nonpharmacologic treatment includes cognitive behavioral therapy, patient education, sleep hygiene, and low-impact aerobic exercise (swimming). Pharmacologic treatment includes tricyclic antidepressants (amitriptyline), selective serotonin reuptake inhibitors (fluoxetine), serotonin and norepinephrine reuptake inhibitors (duloxetine, milnacipran), cyclobenzaprine, pregabalin, and gabapentin. Nonsteroidal anti-inflammatory drugs, corticosteroids, and opioids are generally ineffective.
What are the absolute contraindications to administering alteplase in the treatment of an acute ischemic stroke?
Intracranial hemorrhage on CT; neurosurgery, head trauma, or stroke within the past 3 months; uncontrolled hypertension (> 185/110 mm Hg); history of intracranial hemorrhage; known arteriovenous malformation, aneurysm, or neoplasm; suspected or confirmed endocarditis; known bleeding diathesis; and glucose < 50 mg/dL.
A 31-year-old man presents to the urgent care with pain in the right ankle after twisting it while playing basketball. On physical exam, the lateral aspect of the right ankle is swollen and tender to palpation at the tip of the lateral malleolus. There is moderate laxity at the ankle joint. The patient is able to bear weight on his right foot. A right ankle radiograph is negative for fracture. What is the mechanism of injury for the most likely diagnosis?
Inversion of a plantar flexed foot A lateral ankle sprain may be treated with rest, ice, compression, and elevation. Oral or topical nonsteroidal anti-inflammatory drugs are recommended for pain control. Range of motion exercises are important early in recovery. A brace or splint may be provided as well. What orthopedic test is performed by squeezing the calf muscle and observing for plantar flexion? Answer: Thompson test.
A 51-year-old man with a 35 pack-year smoking history presents to the office for his annual physical. He has a history of a lung nodule on a prior chest X-ray. The patient is now complaining of right shoulder pain and hand weakness. On physical examination, atrophy of the intrinsic muscles of the right hand is noted. What other physical examination finding would be characteristic of his suspected diagnosis? AHeliotrope rash BHyperreflexia of bilateral upper extremities CIpsilateral ptosis and miosis DPalpable adrenal mass
Ipsilateral ptosis and miosis dx:Horner syndrome #pancoast tumor may be observed in patients with Pancoast syndrome. Lung cancer is the most common cause of cancer-related deaths in the United States in men and women. It is strongly associated with cigarette use. Additional risk factors include previous radiation, asbestos exposure, occupational exposures, and pulmonary fibrosis. Lung cancer is often associated with a specific physical exam syndrome, such as superior vena cava syndrome and Pancoast syndrome. Pancoast syndrome refers to lung tumors, most commonly non-small cell lung cancer, that occur in the superior sulcus. Patients with a Pancoast tumor may develop dermatomal symptoms stemming from C8, T1, and T2 due to involvement of the brachial plexus. This can result in muscle atrophy, weakness, and pain. Additionally Horner syndrome can be present. Lung cancer frequently metastasizes, particularly to the liver, bones, adrenal glands, and brain. Patients may present with signs and symptoms related to their metastatic disease in aggressive disease, such as back pain, headaches, or focal neurological symptoms. Paraneoplastic syndromes are also a common feature of lung cancer and may consist of hypercalcemia, syndrome of inappropriate antidiuretic hormone secretion (SIADH), and Lambert-Eaton myasthenic syndrome CT scan of the chest with contrast should be performed What is the location of a Pancoast tumor? Answer: The superior sulcus of the lung.
A 22-year-old man presents to his primary care provider with a 3-week history of a productive cough. He recently immigrated to the United States from an urban city in Brazil to begin school. He also reports a history of night sweats and a 10-pound unintentional weight loss. Physical exam reveals rales in the middle of the left lung. A chest X-ray is ordered and shows left middle lobe consolidation. An interferon-gamma assay test reveals evidence of Mycobacteriuminfection. Which of the following medications should be included in this patient's initial treatment regimen? A 45-year-old man with a history of diabetes and newly diagnosed active pulmonary tuberculosis presents to the office for evaluation of numbness and tingling in his feet. He describes the sensation as "pins and needles." Which medication used to treat active tuberculosis is the most likely culprit of this patient's symptoms? what can be given to prevent this specific side effect A 36-year-old man presents to the office with a persistent cough. He reports the cough began months ago and was not initially productive, but it now produces a foul-smelling greenish sputum. He also notes occasional fever at night and a 15-pound unintentional weight loss over the last 6 months. On physical exam, he is well-developed and well-nourished and in no acute distress. Lungs are clear bilaterally without rales, wheezes, or rhonchi. A chest radiograph shows bilateral hilar adenopathy without consolidation. There is a 2 cm cavity in the right lung apex. Sputum cultures are collected and reveal a rod-shaped, aerobic, acid-fast bacteria. What is the most likely diagnosis?
Isoniazid dx: TB In asymptomatic individuals with one positive tuberculosis screen, a confirmatory test with IGRA is recommended. If both tests are positive, individuals should next be evaluated for active tuberculosis, which may include chest X-ray, other imaging, and sputum tests. The gold standard diagnosis for active TB is a culture of the acid-fast bacilli bacteria from respiratory secretions. Which RIPE antibiotic (rifampin, isoniazid, pyrazinamide, and ethambutol) is associated with higher rates of hepatitis? is known to cause neuropathy, Answer: Isoniazid. Vitamin B6 may be prescribed with it to prevent neuropathy. What is the common name of a pulmonary aneurysm found in tuberculosis that begins in the infected cavity and spreads to the bronchial arteries, causing massive hemoptysis upon rupture? Answer: Rasmussen aneurysm.
45-year-old man with a history of a recent dental infection presents to the office for evaluation of fever, chills, and anorexia. Vital signs reveal a temperature of 101°F and a heart rate of 110 beats per minute. Physical examination reveals petechiae on the extremities as well as the conjunctivae and a new diastolic heart murmur. What additional physical exam finding would most likely be present in this patient? A 60-year-old man with a history of intravenous drug use presents to the emergency department with complaints of chills, chest pain, shortness of breath, myalgias, and confusion. Physical exam findings include a temperature of 102.3°F, a new-onset heart murmur, and painless erythematous macules on the soles of his feet. Three sets of blood cultures are drawn. Which of the following diagnostic tests is used to help confirm the diagnosis?
Janeway lesions Echocardiogram dx: Infective endocarditis, Which valve is most likely to be affected in patients with endocarditis secondary to intravenous drug use? Answer: Tricuspid valve. Bacterial Endocarditis Risk factors: injection drug use, valvular heart disease Sx: fever, rash, cough, and myalgias PE: fever, Roth spots, Osler nodes, murmur, Janeway lesions, anemia, nailbed hemorrhages, emboli (FROM JANE) Diagnosis is made by echocardiography and Duke criteria Most commonly caused by:IVDA: Staphylococcus aureus, tricuspidNative valve: Staphylococcus aureus, viridans streptococci (most common in previously diseased), mitral Tx: antibiotics GI malignancy: Streptococcus bovis Dental prophylaxis in some cases
A 38-year-old woman presents to her primary care provider with complaints of joint pain in her wrists, metacarpophalangeal joints, and proximal interphalangeal joints bilaterally. She states her symptoms are worse in the morning and with inactivity. She reports morning stiffness lasting longer than 1 hour. Light activity alleviates her pain. Physical examination reveals soft, warm, tender, and swollen proximal interphalangeal joints bilaterally. Ulnar deviation of the fingers and swan-neck deformities are present. Which of the following findings seen on hand radiograph is most consistent with the suspected diagnosis?
Joint erosions dx;Rheumatoid arthritis Rheumatoid Arthritis More common in women and those aged 40-50 years Morning stiffness lasting > 30 minutes PE will show symmetrical soft, red, tender swelling in joints: MCP, PIPBilateral ulnar deviation at MCP, boutonnière deformity, and swan-neck deformity Labs will show positive RF or anti-cyclic citrullinated peptide antibodies; ↑ ESR; ↑ CRP Most commonly caused by autoimmune destruction of synovial joints Treatment is DMARDs What syndrome is characterized by splenomegaly, anemia, neutropenia, thrombocytopenia, and severe rheumatoid arthritis? Answer: Felty syndrome. =========================================== Osteophytes (B) and subchondral sclerosis (D) on radiograph are associated with osteoarthritis. Pencil-in-cup deformity (C) is a characteristic radiograph finding indicative of psoriatic arthritis.
A 55-year-old man presents to his primary care provider with complaints of increasing peripheral vision loss for the past 3 months. His past medical history is significant for diabetes mellitus. Physical examination reveals optic disk cupping and a cup to disc ratio of 0.7. Tonometry reveals an intraocular pressure of 40 mm Hg. Visual field testing confirms significant peripheral field defects. Which of the following medications is most appropriate at this time?
Latanoprost First-line pharmacologic treatment for chronic glaucoma is a prostaglandin analog (e.g., latanoprost), which decreases IOP by increasing outflow of aqueous humor. dx: Chronic glaucoma increased ocular pressure (IOP), optic disk or retinal nerve abnormalities, and peripheral visual field defects. Insidious process, and signs and symptoms are typically first detected on a routine eye examination. Optic disk cupping is a hallmark of glaucoma and is characterized by an absolute increase or asymmetry between the two eyes of the cup to disc ratio. A cup to disc ratio > 0.5 or asymmetry > 0.2 is indicative of glaucoma. A topical beta-blocker (e.g., timolol) may be used alone or in combination with a prostaglandin analog and decreases IOP by decreasing the production of aqueous humor. An alpha-2 agonist (e.g., brimonidine) or carbonic anhydrase inhibitor (e.g., acetazolamide) may be used if prostaglandin analogs and beta-blockers are contraindicated. The goal of pharmacologic treatment is to reduce intraocular pressure to a level that reduces the progression of visual field defects. Surgical treatment is indicated if pharmacologic control is inadequate or if glaucoma is advanced on initial diagnosis. Trabeculectomy is the standard procedure for chronic open-angle glaucoma. What surgical procedure is used as definitive treatment for angle-closure glaucoma? Answer: Peripheral iridotomy. Chronic glaucoma can be categorized as open-angle glaucoma, angle-closure glaucoma, or normal-tension glaucoma Open-angle glaucoma is the most common=reduced drainage of aqueous fluid through the trabecular meshwork. Chronic angle-closure glaucoma is characterized by obstruction of aqueous fluid into the interior chamber angle. Normal-tension glaucoma is characterized by peripheral to central visual field defects and optic disk or retinal nerve changes in the presence of normal IOP.
A 24-year-old woman with HIV infection presents to the clinic for a checkup and annual vaccines. She has no current concerns and needs refills of her raltegravir and tenofovir-emtricitabine. Her current CD4 cell count is 180/microL. Which of the following vaccines is indicated for this patient's health maintenance?
Meningococcal vaccine no MMR or vercila cd 4 <200
Q155. A 54-year-old man presents to the ED with confusion and slurred speech. Physical examination reveals the presence of asterixis and hyperactive deep tendon reflexes. Laboratory results are significant for an ammonia level of 140 mcg/dL. The patient is treated with lactulose, and his symptoms begin to improve. His past medical history is significant for cirrhosis, and this is the third time he has had this complication. Which of the following treatments is most effective for preventing recurrence of the patient's condition?
Long-term rifaximin therapy in preventing recurrent hepatic encephalopathy and improving quality of life, and it is associated with better improvements in morbidity and mortality when given with lactulose compared to lactulose alone. Which grade of hepatic encephalopathy is characterized by asterixis? Answer: Grade 2. Hepatocellular carcinoma is characterized by elevated serum alpha-fetoprotein levels. ==================================================== Terlipressin therapy with albumin (D) is recommended for patients with hepatorenal syndrome secondary to portal hypertension and cirrhosis to reverse acute kidney injury.
A 19-year-old man presents to his primary care provider with complaints of frequent sneezing, rhinorrhea, itchy eyes, itchy nose, and excessive tearing for the past 3 weeks. He states he just started a lawn mowing business and spends a lot of time outside. Physical examination is significant for pale, boggy turbinates and conjunctival injection. A transverse nasal crease is present, as shown in the image above. Which of the following medications would be most appropriate at this time, in addition to an intranasal corticosteroid spray?
Loratadine dx:Allergic rhinitis Brompheniramine (A), diphenhydramine (B), and hydroxyzine (C) are all first-generation antihistamines and are considered second-line agents for allergic rhinitis because they are associated with more sedative and anticholinergic side effects than second-generation antihistamines. Vasomotor rhinitis, a common cause of clear rhinorrhea in older patients, is associated with an increased sensitivity of which nerve? Answer: The vidian nerve.
A 45-year-old woman presents with unilateral facial drooping on her right side. She says it has been getting worse over the past 2 days and is not painful. Physical exam findings include right eyebrow drooping, inability to close the right eye, and drooping of the right corner of the mouth. Which of the following additional physical exam findings would be consistent with the suspected diagnosis? AForehead sparing BHearing loss CLoss of taste DRight arm weakness
Loss of taste dx: Bell palsy drooping at the affected corner of the mouth, drawing of the mouth to the unaffected side, inability to close the eye, eyebrow sagging, hyperacusis, decreased tearing, loss of taste on anterior two-thirds of the tongue, and disappearance ofnasolabial fold. Diagnosis is typically made on clinical characteristics. Treatment should be initiated with oral prednisone 60-80 mg/day for 1 week. Ideally, treatment should be initiated within the first 3 days of symptom onset. No proven benefit has been found with the addition of the antiviral agents valacyclovir and acyclovir, Management should also include ophthalmologic care to prevent damage to the cornea with taping of the eyelid or covering the eye during sleep and artificial tears. Massage of the weakened facial muscles and physical therapy are also beneficial. Prognosis is favorable if some recovery is seen within 21 days, and most patients have some recovery by 4 months. Patients should have a full neurological exam and should be evaluated for rashes or lesions on the face or external ear, which would indicate varicella zoster and a possible alternative diagnosis of Ramsay Hunt syndrome. Forehead sparing should prompt evaluation for a possible central lesion such as a stroke Bilateral: Lyme disease, infectious mononucleosis
A 58-year-old man presents to your office for his annual exam and wants to make sure he is up to date on all of his preventive screenings. His colonoscopy last year was normal. He tells you he started smoking cigarettes at 20 years old and smokes one pack per day. Which of the following is recommended screening for this patient? AAbdominal ultrasound BBone density scanning CFecal immunochemical test DLow-dose computed tomography
Low-dose computed tomography One of the most common types of cancer in the United States is lung cancer, and it is the leading cause of cancer-related death for both men and women. Most cases of lung cancer are due to smoking or environmental smoke exposure.
13-year-old boy presents to his primary care provider complaining of frequent episodes of wheezing and shortness of breath. His mother states over the last few months he has needed his albuterol inhaler daily. His teacher recently notified the patient's mother that he has been sitting out of recess sometimes due to his asthma. He has also been waking up from sleep at least once per week with symptoms. The patient's current asthma treatment is a short-acting beta-agonist, albuterol, as needed and a low-dose inhaled corticosteroid. What is the appropriate therapy for this patient's asthma? AHe is already on appropriate therapy BHigh-dose inhaled corticosteroid and a long-acting beta-agonist CLow-dose inhaled corticosteroid and a long-acting beta-agonist DShort-acting beta-agonist as needed only
Low-dose inhaled corticosteroid and a long-acting beta-agonist mild persistent asthma would present with symptoms > 2 days per week (but not daily), they would have three to four episodes of night awakenings due to symptoms per month, they would use their short-acting beta-agonist inhaler more than two times per week (but not daily and not more than once per day), and they would have minor limitations to daily activity. These patients would be appropriately treated with a low-dose inhaled corticosteroid and a short-acting beta-agonist as needed. moderate persistent asthma. These patients would have symptoms daily, have night awakenings due to symptoms at least once per week (but not nightly), use their short-acting beta-agonist inhaler on a daily basis, and have some limitations to their daily activity. These patients, such as the patient in the vignette, are appropriately treated with a low-dose inhaled corticosteroid plus a long-acting beta-agonist or a medium-dose inhaled corticosteroid.using a short-acting beta agonist as needed for acute symptoms severe persistent asthma. These patients would have asthma symptoms on a daily basis, they would have episodes of night awakening due to asthma symptoms on a nightly basis, they would use their short-acting beta-agonist inhaler multiple times during the day, and they would have extreme limitations on their daily activity. These patients are treated with a medium- or high-dose inhaled corticosteroid and a long-acting beta-agonist. They can also be on controller medications such as intramuscular omalizumab or theophylline, along with an as-needed short-acting beta agonist during acute symptoms.
A 38-year-old woman presents to her primary care provider with complaints of recurrent episodes of vertigo and tinnitusfor the past 4 months. She states this has never happened before. Her episodes last about 2 hours and are accompanied by nausea. She is put on a low-sodium diet and prescribed triamterene-hydrochlorothiazide, which significantly improves her symptoms. Which of the following additional physical examination findings is most consistent with the suspected diagnosis?
Low-frequency sensorineural hearing loss Ménière disease, also known as endolymphatic hydrops, is an endolymphatic compartment disorder characterized by episodic vertigo, tinnitus, and low-frequency sensorineural hearing loss. treatment The Weber and Rinne tests are used to identify sensorineural hearing loss, and an audiogram is used to confirm low-frequency sensorineural hearing loss. Management of Ménière disease includes limiting sodium, caffeine, alcohol, nicotine, and monosodium glutamate intake. A diuretic (e.g., acetazolamide, triamterene-hydrochlorothiazide) or chronic betahistine may be used for preventative management. Patients managed with lifestyle modifications and preventative pharmacotherapy may use benzodiazepines (e.g., clonazepam, diazepam) for treatment of acute vertigo attacks. Promethazine or ondansetron may be used for associated nausea or vomiting. Refractory cases may be treated with systemic glucocorticoids for 1 to 2 weeks. Ménière Disease Patient presents with episodic low-frequency hearing loss, tinnitus with aural (ear) fullness, and vertigo lasting 20 minutes up to 24 hours Diagnosis is made clinically Most commonly caused by too much inner ear endolymph and increased pressure within the inner ear Treatment is low-salt diet, diuretics (HCTZ + triamterene)
A 66-year-old man with a history of coronary heart disease presents to the emergency department after collapsing in the parking lot. His monitor shows the rhythm above. There is no palpable pulse. Cardiopulmonary resuscitation is initiated, and intravenous access is established. A defibrillator is attached, and 200 joules of biphasic energy are delivered. After 2 minutes of cardiopulmonary resuscitation, epinephrine 1 mg IV is given. Return of spontaneous circulation is achieved, but the patient is not following commands. Blood glucose is 170 mg/dL. Temperature is 38.0°C. Mean arterial pressure is 90 mm Hg. Which of the following clinical interventions is most appropriate at this time? AAdminister hypotonic fluids to reduce cerebral edema BLower blood glucose to < 100 mg/dL CLower core body temperature to 32-36°C DLower mean arterial pressure to < 65 mm Hg
Lower core body temperature to 32-36°C dx: V fib ventricular fibrillation or pulseless ventricular tachycardia requires immediate cardiopulmonary resuscitation (CPR) and maintenance of the airway and oxygen. 2) A defibrillator should be attached as soon as possible to monitor the rhythm. If the monitor shows a shockable rhythm (e.g., ventricular fibrillation, pulseless ventricular tachycardia), 120-200 joules of biphasic shock or 360 joules of monophasic shock are given, followed by 2 minutes of CPR. 3) If the patient is still in ventricular fibrillation or pulseless ventricular tachycardia, epinephrine 1 mg IV or IO is given. 4)CPR is resumed for another 2 minutes. An advanced airway or capnography should be considered. If the patient remains in ventricular fibrillation or pulseless ventricular tachycardia, amiodarone 300 mg IV or IO is given, followed by another 2 minutes of CPR. If return of spontaneous circulation is achieved, the advanced cardiac life support postcardiac arrest care algorithm is followed. Oxygen is maintained between 94-100% and systolic blood pressure is treated if < 90 mm Hg or mean arterial pressure is < 65 mm Hg. If the patient does not follow commands, inducing hypothermia to lower core body temperature to 32-36°C for at least 24 hours will improve clinical outcomes. An electrocardiogram should be attained. If there is evidence of a myocardial infarction, the patient should be transferred for percutaneous coronary intervention. If there is no evidence of a myocardial infarction, the patient should be transferred to the intensive care unit What is the appropriate second dose of amiodarone after the initial 300 mg dose, according to the advanced cardiac life support algorithm? Answer: 150 mg.
A 25-year-old man is referred to a urologist for a testicular mass. He has no significant past medical history and has never been sexually active. Which of the following physical examination findings is most concerning for metastatic disease?
Lower extremity edema 15-35 years of age. Approximately 95% of primary testicular tumors are germ cell tumors, which are divided into nonseminomas and seminomas. Risk factors include a history of cryptorchidism, personal or family history of testicular cancer, and HIV infection The most common symptoms are painless enlargement of the testis or sensation of heaviness. A nontender, discrete mass or diffuse testicular enlargement is usually appreciated on physical examination. Signs and symptoms concerning for metastatic di sease include lower extremity edema (vena cava obstruction), back pain (retroperitoneal metastases), and cough Abnormal laboratory findings associated with testicular cancer include elevations in human chorionic gonadotropin, alpha-fetoprotein, and lactate dehydrogenase. Definitive diagnosis is established by inguinal orchiectomy. Is cryptorchidism more likely to occur on the right or left side? Answer: Right side. Testicular Cancer Risk factors: cryptorchidism, age 15-35 Sx: testicular lump PE: painless, hard, fixed mass Labs: beta-hCG, alpha-fetoprotein (AFP), or lactate dehydrogenase (LDH) may be elevated based on tumor type Diagnosis starts with ultrasound Tx: radical inguinal orchiectomy usually curative, XRT or platinum-based chemo if metastatic
A 45-year-old man presents with complaints of pain and weakness in his right leg. The patient states he was bending forward to get groceries out of his car last week when he felt pain down the back of his leg. He states the pain has gotten progressively worse, and now he feels like his leg is weak. On physical exam, the patient has 2 out of 5 strength with right hip extension and with plantar flexion. There is decreased sensation over the lateral aspect of his right foot, and ankle jerk reflex is absent. Straight leg raise reproduces pain down the posterior portion of his leg on the right side. Which of the following is the most appropriate diagnostic test for the most likely diagnosis?
MRI Lumbar radiculopathy nerve root compression caused by spondylosis or a disc herniation. What condition is a kyphoplasty used to treat? Answer: Vertebral compression fracture.
A 62-year-old man with a history of hypertension and sleep apnea presents to the emergency department via ambulance after experiencing a sudden onset of right-sided arm weakness and difficulty speaking when giving a presentation at work 1 hour ago. Physical examination reveals expressive aphasia, decreased strength in the right upper extremity, and right-sided pronator drift. Which of the following diagnostic studies would be the most sensitive for detecting the suspected diagnosis in this patient?
MRI is more sensitive than CT scan in detecting acute ischemic stroke. Cerebrovascular accidents, or strokes, What is Wallenberg syndrome? Answer: A lateral medullary infarction resulting in an array of manifestations, including dizziness, difficulty sitting upright, diplopia, nystagmus, hypotonia and limb ataxia of the ipsilateral arm, and loss of pain and temperature sensation in the ipsilateral face and contralateral trunk and limbs.
A 48-year-old woman presents to the clinic complaining of copious amounts of diarrhea and intolerable abdominal cramps for the past 2 months. She was started on several medications at her last visit for management of her type 2 diabetes mellitus. Which of the following agents is the most likely cause of her current symptoms?
Metformin The most common side effects of metformin are gastrointestinal, including nausea, vomiting, diarrhea, flatulence, and abdominal cramping. Switching to the extended-release form of this drug is sometimes adequate to control these symptoms. More serious side effects of metformin include lactic acidosis and vitamin B12 deficiency. Metformin is excreted by the kidneys and should not be prescribed to patients with a glomerular filtration rate under 30 mL/min/1.73 m2. What is the mechanism of action of saxagliptin? Answer: Saxagliptin is a dipeptidyl peptidase-4 inhibitor that prolongs incretin levels and decreases glucagon secretion.
A 37-year-old man presents to his primary care provider with intermittent chest discomfort, palpitations, and worsening dyspnea on exertion over the past 5 months. The patient states he has also noticed decreased exercise tolerance and occasional lightheadedness while exercising. Physical examination is significant for a midsystolic click. When the patient is asked to squat, the click is heard later in systole. Which of the following valvular disorders is most likely to be seen on echocardiography?
Mitral valve prolapse Mitral Valve Prolapse Patient presents with palpitations, dyspnea, nonexertional CP, and fatigueMost cases are asymptomatic PE will show midsystolic click and late systolic murmur ↑ preload → ↓ murmur ↓ preload → ↑ murmur Marfan syndrome, Ehlers-Danlos Diagnosis is made on clinical exam and confirmed by echo
A 43-year-old woman presents to your office complaining of worsening asthma. She was diagnosed with asthma 10 years ago. Since then, she reports having very few symptoms and has not needed to use her albuterol inhaler often. Over the last several months, her symptoms have worsened, and she has had many instances of breathlessness and agitation. Last month, these symptoms occurred while at work almost daily and were usually associated with wheezing. During that time, she also found herself waking up coughing and feeling out of breath at least 5 times. She reports no fever, chills, nausea, or vomiting and no recent illness or sick contacts. She is an elementary school teacher, and while she is still able to perform at her job, she is concerned her symptoms may start limiting her activities with her students. Which of the following diagnoses is most appropriate? What condition presents with an expiratory monophonic wheeze best heard over larger airways?
Moderate persistent asthma Moderate persistent asthma indicates daily symptoms, nightly symptoms more than once a week, and symptoms that interfere with daily activities. Severe persistent asthma indicates symptoms throughout the day, nightly symptoms often more than seven times a week, and symptoms that have extremely limited the patient's daily activities. What condition presents with an expiratory monophonic wheeze best heard over larger airways? Answer: Tracheomalacia.
A 16-year-old girl presents to urgent care with her father after injuring herself when she was bounced off of a trampoline. The father states the patient fell onto her outstretched left arm with her elbow extended and her forearm hyperpronated from a height of about 8 feet. Her X-ray is shown above. What is the most likely diagnosis?
Monteggia fracture fracture of the proximal ulnar shaft with a radial head dislocation. All Monteggia fractures are considered unstable and will need surgical intervention.
A previously healthy 30-year-old woman presents to your office with complaints of fatigue, weight loss, and skin changes. She says her skin has become significantly darker in the past several months without changes in diet or activities. Vital signs reveal a blood pressure of 85/50 mm Hg. Initial laboratory test results include decreased sodium and increased potassium. Which of the following tests helps to establish the diagnosis?
Morning serum cortisol level Primary adrenal insufficiency, also called Addison disease, Treatment is expected to be lifelong with a combination of glucocorticoids andmineralocorticoids. Glucocorticoids include prednisone, hydrocortisone, and dexamethasone. Mineralocorticoid replacement is typically with fludrocortisone. Primary Adrenal Insufficiency (Addison Disease) Patient presents with abdominal pain, nausea, vomiting, diarrhea, fever, and confusion PE will show hyperpigmentation of skin and mucus membranes and hypotension Labs will show hyponatremia and hyperkalemia Most commonly caused by autoimmune destruction of the adrenal cortex Treatment is hydrocortisone or other glucocorticoid
A 22-year-old woman presents with lower abdominal pain that has been worsening over the last 3 days. She reports thick, vaginal discharge for the last week. Physical exam reveals abdominal tenderness in the lower quadrants of the abdomen bilaterally and cervical motion tenderness. Purulent endocervical discharge is noted. Which of the following is the greatest risk factor for developing the most likely diagnosis?
Multiple sexual partners How long should outpatient oral antibiotic treatment be continued when treating pelvic inflammatory disease? Answer: For 14 days dx:Pelvic inflammatory disease (PID)
A 66-year-old woman presents to the emergency department with complaints of a dry cough, rhinorrhea, and shortness of breath for the past 3 days. She states she got home today after a long flight back from Japan. She was home for less than an hour when she suddenly felt lightheaded and passed out about 2 hours ago. Her medical history is significant for diabetes mellitus, which is well controlled. Her vital signs are within normal limits. Her chest X-ray is unremarkable. Her ECG is shown above. Her D-dimer is 120 ng/mL, her blood glucose is 110 mg/dL, and her troponin I is 0.95 ng/mL. Which of the following is the most likely diagnosis?
Myocardial infarction True or false: nitroglycerin is contraindicated in right ventricular infarction. Answer: True.
A 62-year-old man presents to his primary care provider with complaints of right knee pain for the past 3 months. He reports morning stiffness of his knee that lasts about 20 minutes. The pain is worse in the evening, aggravated by activity, and alleviated by rest. All vital signs are within normal limits. Physical examination of the right knee reveals tenderness to palpation along the medial joint line, bony swelling, and crepitus with movement. No erythema or warmth is noted. His right knee radiograph is shown in the image above. Which of the following treatments is the best next step in the management of this patient?
Naproxen dx: Osteoarthritis/degenerative joint disease First-line pharmacologic therapy is oral or topical nonsteroidal anti-inflammatory drugs (NSAIDs), such as naproxen. Glucocorticoid injections may relieve joint pain associated with osteoarthritis for several weeks but should not be given more than four times each year for each joint Osteoarthritis Monoarticular or polyarticular joint pain and stiffness (knees, hips, CMC, DIP often affected) Sx: pain worse with activity, improves with rest PE: Swelling, deformity ( ex: Bouchard nodes, Heberden nodes), crepitus, decreased ROM Dx: X-ray - subchondral sclerosis, joint space narrowing, subchondral cysts, and osteophytes Tx options: weight management, physical therapy, NSAIDs, acetaminophen (lack of benefit), corticosteroid joint injections, joint replacement No constitutional symptoms
A 55-year-old man presents with complaints of daytime sleepiness. The patient denies insomnia or nighttime awakenings. The patient does report that his wife complains about his loud snoring, but that has been going on for years. Which of the following findings would support your suspected diagnosis?
Neck circumference of 18 inches dxObstructive sleep apnea Physical exam findings can include obesity with a body mass index (BMI) greater than 29 kg/m2, crowded oropharyngeal airway, increased neck circumference (over 17 inches for men and 16 inches for women), or increased waist to height ratio (greater than 100.4 cm for men and greater than 95.5 cm for women). Positive airway pressure therapy is the main treatment for patients with sleep apnea. Positive airway pressure stabilizes the upper airway throughout expiration to prevent upper airway collapse. Patients should also be counseled on weight loss and exercise, alcohol avoidance, and avoidance of medications that inhibit the central nervous system, such as benzodiazepines. What is Cheyne-Stokes breathing? Answer: An abnormal pattern of breathing characterized by periods of tachypnea and hyperpnea alternated by periods of apnea.
A 22-year-old woman who is otherwise healthy presents to the office due to red, raised, and painful bumps to the inside of the thighs, starting about 2 days ago. She works as a physical trainer and typically wears tight-fitting workout leggings and has been sweating in the affected areas due to warmer weather. A physical exam shows multiple erythematous pustules to the inner aspect of the thighs with hair emanating from the center of each pustule. Which of the following is the most likely diagnosis? What is a complication of folliculitis that causes chronic lesions to the head and neck that are refractory to treatment?
Nonbacterial folliculitis What is a complication of folliculitis that causes chronic lesions to the head and neck that are refractory to treatment? Answer: Sycosis. ================================================== Eosinophilic folliculitis (A) is a type of folliculitis common in patients with AIDS, but this patient is otherwise healthy. Pseudofolliculitis (C) occurs when there is pustular formation caused by ingrown hairs secondary to shaving. The lesions are located to the side of the hair follicles and not in them. It is seen most often in the beard region in men but can also be seen in women in commonly shaved areas. Pseudomonas folliculitis (D) commonly occurs on the trunk in healthy patients and is associated with hot tub use with contaminated water. Patients present with follicular papules and pustules that occur 8 to 48 hours after exposure to contaminated water.
A 46-year-old man presents to the emergency department with severe flank pain beginning 1 hour ago. The pain has been intermittent, but when it occurs, the patient has difficulty standing up straight or performing any activity. He took acetaminophen 1,000 mg at the onset of the pain without relief. He reports nausea and two episodes of nonbloody, nonbilious emesis. Physical exam demonstrates left-sided costovertebral angle tenderness. Which of the following diagnostic studies has the highest diagnostic accuracy for confirming the suspected diagnosis?
Noncontrast CT of abdomen and pelvis dx:Nephrolithiasis common condition often associated with hypertension, diabetes, decreased fluid intake, hyperparathyroidism, and gout. Sx: flank pain radiating to groin PE: patient won't lie still, hematuria Diagnosis: noncontrast helical CT, most common location is the ureterovesical junction (UVJ) Most commonly caused by calcium oxalate Struvite: staghorn calculi, urease-producing bacteria Uric acid: radiolucent on X-ray, gout Cystine: children with metabolic diseases Treatment< 5 mm: likely to pass spontaneously> 5 mm: medical expulsive therapy (tamsulosin), urology consultation in certain cases> 10 mm: urology consultation, shock wave lithotripsy, ureteroscopy
A 52-year-old man presents to the clinic complaining of bilateral decreased visual acuity. He has missed his last three appointments and was last seen 12 months ago. He currently takes metformin 500 mg twice daily to manage diabetes mellitus and lisinopril 10 mg once daily to control blood pressure. Vital signs are within normal limits and blood pressure is 127/82 mm Hg. A glycated hemoglobin measurement performed in the office is 9.7%. On funduscopic exam, dilated, tortuous vessels are noted in the maculae, and there are intraretinal hemorrhages bilaterally. Which of the following is the most likely diagnosis?
Nonproliferative diabetic retinopathy In proliferative diabetic retinopathy, new blood vessels are seen arising from the optic disc or retinal vessels on physical exam using an ophthalmoscope. These new vessels may cause preretinal or vitreous hemorrhages, retinal fibrosis, or retinal detachment. Nonproliferative diabetic retinopathy does not demonstrate neovascularization but shows tortuous, dilated retinal vessels in the macula and may cause retinal hemorrhages.
A 24-year-old woman presents with an injury to her right wrist that occurred 2 days ago. After some discussion, she reports her husband pushed her down, and she fell on an outstretched hand. He initially did not allow her to seek help for her injury and does not know she is here today. There were no witnesses to the event. Which of the following is the best next step in addressing the intimate partner violence that has occurred?
Offer empathy and assistance What is an important consideration when giving patients who have reported intimate partner violence written summaries of their visit? Answer: Do not include any written material or resources regarding intimate partner violence. ---------------------------------------------------------- Administering the nine-item Patient Health Questionnaire (PHQ-9) (A) is not helpful in identifying intimate partner violence, but administering the 20-item Danger Assessment is useful in identifying the likelihood of lethality within the relationship. Admitting the patient to the hospital for safety reasons (B) is not appropriate. This could escalate the intimate partner violence if the partner discovers why the patient is being admitted, and it is not a long-term solution to the situation. Calling the police and reporting the interpersonal violence (C) could escalate the violence and worsen the situation if the patient is not willing to leave or press charges against the partner. When interpersonal violence involves a person with a disability, a person in the older population, or a child, or when weapons are involved, a clinician has the mandatory responsibility to report to authorities.
A 68-year-old man with a history of coronary artery disease and acute myocardial infarction 2 months ago presents to the clinic complaining of nonpainful, complete vision loss in his left eye with acute onset 8 hours ago. He reports no headache or fever. Physical exam reveals an afferent pupillary defect on the left with no ability to count fingers. Funduscopic exam reveals a pale retina with a cherry-red spot in the macula. Current medications include metoprolol tartrate 50 mg twice daily, lisinopril 10 mg daily, atorvastatin 20 mg daily, sildenafil 50 mg as needed, and aspirin 81 mg daily. Erythrocyte sedimentation rate is within normal limits. Which of the following is the best next step for this patient
Ophthalmology consult and ocular massage Acute retinal artery occlusio acute-onset, painless, unilateral vision loss that may involve most or all visual fields. On exam, there will be an afferent pupillary defect (dilation of the affected pupil when a swinging flashlight moves toward it) and retinal pallor on funduscopic exam. A cherry-red spot may appear in the macula. In general, all interventional efforts should commence prior to 24 hours postocclusion. In a patient with central retinal artery occlusion presenting to a primary care clinician 8 hours after symptom onset, as in the above scenario, prompt ophthalmology referral is essential. While waiting, ocular massage can be applied to the affected eye in an effort to dislodge the embolism and improve retinal perfusion. O Treatment is globe massage, ↓ IOP, ophthalmology consultatio What are the possible ocular sequelae of retinal artery occlusion? Answer: Vision loss, vitreous hemorrhage, and neovascular glaucoma. ==================================================== Intra-arterial recombinant tissue plasminogen activator (A) is contraindicated in a patient with a recent (within the past 3 months) history of myocardial infarction. This intervention is best employed 6 to 12 hours after ischemia onset. Oral high-dose prednisone (C) is indicated in cases of retinal artery occlusion that are secondary to giant cell arteritis. The patient in this vignette does not have a headache or palpable temporal artery and has a normal erythrocyte sedimentation rate. All patients over the age of 50 years with acute monocular vision loss should have an erythrocyte sedimentation rate performed to rule out giant cell arteritis, as this is a very treatable condition. Patients with giant cell arteritis respond well to high-dose steroids. Oral nitroglycerin (D) is a vasodilator that is sometimes used to rapidly decrease intraocular pressure and possibly dislodge a retinal artery embolus. However, this patient takes sildenafil and is not a candidate for oral nitroglycerin therapy.
A 4-year-old girl presents to the office for severe diarrhea that began yesterday. She attends daycare. The patient's caregiver reports her symptoms started with a flu-like illness of fever and malaise before she developed frequent bloody, watery stools occurring eight times yesterday and four times today. She reports lower abdominal pain and tenesmus. Physical examination reveals a temperature of 100.5°F. Stool microscopy is performed and reveals fecal leukocytes and red blood cells. The sample is sent for culture. What intervention, in addition to oral rehydration therapy, should be considered in this patient, given the most likely diagnosis?
Oral azithromycin dx: Shigella dysenteriae in children Which Shigella species is associated with the development of reactive arthritis? Answer: Shigella flexneri. Shigellosis Patient presents with fever, bloody and mucoid diarrhea, and seizures (more common in children) Labs will show fecal RBCs and WBCs Treatment is based on resistance patterns but commonly azithromycin or ciprofloxacin Complications: HUS, reactive arthritis
A 31-year-old man presents to his primary care provider with complaints of chest discomfort for the past 4 months. He states he feels a burning sensation in his chest and throat about 30 minutes after meals and when he lies down. Antacids seem to relieve the discomfort for about 2 hours. He eats a healthy diet but is wondering if there are any dietary changes he can make to improve his symptoms. Which of the following foods should be eliminated from the patient's diet?
Oranges dx GERD Lifestyle factors that may exacerbate GERD include smoking, alcohol, ingestion of large meals, acidic foods (e.g., tomatoes, oranges [citrus], lemons), fatty foods, spicy foods, chocolate, and peppermint.
A 40-year-old woman presents with fatigue. She notes it takes her an hour to fall asleep at night, and she wakes up frequently during the night. This has been ongoing for the past year and began when she started a new job. She thinks it has gotten worse over time, which has prompted her visit. Which of the following would be the best next step in managing this patient's care?
Order polysomnography dx:Chronic insomnia hat are the two most common relaxation-based strategies used for insomnia disorder? Answer: Progressive muscle relaxation and diaphragmatic breathing.
A 68-year-old man with a history of chronic kidney disease and hypertension presents to the office with an abrupt onset of fever, headache, myalgias, malaise, and nonproductive cough that started less than a day ago. His coworkers have similar symptoms. He is febrile to 101.2°F. His lungs are clear to auscultation. What is the most appropriate treatment for this patient?***
Oseltamivir dx:influenza Patients with influenza often report abrupt onset of fever, malaise, myalgia, headache, and symptoms of respiratory tract illness, such as nasal discharge, nonproductive cough, and sore throat. Physical examination of the chest is typically unremarkable in uncomplicated influenza Gastrointestinal illness, such as vomiting and diarrhea, sometimes occurs among children Antiviral therapy may be considered in select patients: 1) such as those with severe disease, those at high risk of complications, and those who are not high risk but present within 48 hours of onset of illness. 2) Patients in populations with a high likelihood of complications include those 65 years of age or older, those who are pregnant or postpartum, residents of long-term care facilities, American Indian and Alaska Native populations, those with extreme obesity, 3) those with certain chronic medical conditions (such as immunocompromising conditions, chronic lung disease, chronic heart disease, and chronic kidney disease), and those receiving glucocorticoids or other immunosuppressive medications Oseltamivir is a neuraminidase inhibitor and the antiviral medication that is most commonly used among patients with influenza. Treatment of influenza includes symptom management. Acetaminophen or nonsteroidal anti-inflammatory drugs may be used to treat symptoms including fever, headache, and myalgia.
A 65-year-old woman presents to her primary care provider for her annual visit. She states she broke her hip 5 months ago after falling down her basement stairs. The patient is scheduled for bone mineral density testing, and a dual-energy X-ray absorptiometry scan reveals a T-score of −2.25. Which of the following is the most likely diagnosis?
Osteopenia look at image The United States Preventive Services Task Force currently recommends screening all women ≥ 65 years of age for osteoporosis and all postmenopausal women < 65 years of age at increased risk for osteoporosis to take prophylactic measures to decrease the risk of fractures. Risk factors: female sex, advancing age, chronic steroid use, alcohol or tobacco use, family history of fragility fracture A T-score ≤ −2.5 standard deviations is consistent with osteoporosis, while a T-score of < −1.0 and > −2.5 standard deviations is consistent with osteopenia. Pharmacologic therapies include bisphosphonates (e.g., alendronate, risedronate) for glucocorticoid-induced osteoporosis, estrogen therapy for osteoporosis secondary to hypogonadism, and parathyroid hormone analogs for women taking gonadotropin-releasing hormone agonists or glucocorticoids Second line: SERMs, recombinant PTH, denosumab Which class of medications used for the treatment of osteoporosis has been associated with osteonecrosis of the jaw? Answer: Bisphosphonates.
A 23-year-old man presents to urgent care with complaints of pain, tenderness, and pruritus on his penis for the past 5 days. Vital signs are BMI 36.2 kg/m2, T 98.6°F, BP 122/74 mm Hg, HR 77 bpm, RR 18 breaths per minute, and oxygen saturation 100% on room air. Physical examination reveals the condition shown above. Which of the following is a complication of the suspected diagnosis?
Paraphimosis dx:Balanitis is a condition in which the glans penis becomes inflamed. Paraphimosis is a medical emergency Phimosis is a condition in which the constricted foreskin cannot be retracted over the glans, while paraphimosis is a condition in which the constricted foreskin is retracted over the glans and cannot be reduced. Treatment for candidal balanitis includes a topical imidazole (e.g., clotrimazole, miconazole), nystatin cream, or oral fluconazole.
A 55-year-old woman presents to her primary care provider for a routine visit. Physical examination is significant for a resting tremor, bradykinesia, infrequent blinking, and dysdiadochokinesia. A trial of high-dose levodopa results in significant symptom improvement. What is the most likely diagnosis?
Parkinson disease Clinical manifestations include resting or pill-rolling tremor, cogwheel or lead pipe rigidity, bradykinesia, postural instability, masked facies or infrequent blinking, cognitive changes, sleep disorders, shuffling gait, and loss of autonomic arm swing during ambulation. Dysdiadochokinesia is a loss of the ability to perform rapid alternating movements and may be present in patients with Parkinson disease. What sign seen in Parkinson disease is characterized by a sustained blink response to repetitive tapping over the bridge of the nose? Answer: Myerson sig
A 54-year-old man presents to his primary care provider with complaints of intermittent epigastric pain, postprandial fullness, nausea, and vomiting for the past 3 weeks. He reports no heartburn, regurgitation, or dysphagia. An upright chest X-ray is ordered and shows a retrocardiac air-fluid level. Which of the following in the patient's history is associated with the suspected diagnosis?
Partial gastrectomy dx:hiatal hernia
A 65-year-old woman presents to her primary care provider for her annual visit. She is concerned about developing osteoporosis because of her age. She has no significant past medical or surgical history and is not currently taking any medications. Which of the following should be recommended to this patient? AAvoid fatty foods BBegin estrogen hormone replacement CLimit exposure to ultraviolet radiation D Participate in an exercise program
Participate in an exercise program look at image for 3 key things What is the difference between the T-score and Z-score? Answer: The T-score compares a patient with a healthy 30-year-old of the same sex, while the Z-score compares a patient with an average person of the same age and sex. Osteoporosis Decline in bone mass that results in increased bone fragility and fracture risk Risk factors: female sex, advancing age, chronic steroid use, alcohol or tobacco use, family history of fragility fracture Diagnosis is made by DXA scan: T-score ≤ −2.5 or presence of a fragility fractureOsteopenia: T-score -1.0 to -2.5 Tx: Lifestyle: calcium, vitamin D, weight bearing exercise, smoking cessation First line pharmacotherapy: bisphosphonates ((e.g., alendronate, pamidronate, zoledronic acid)) Second line: SERMs, recombinant PTH, denosumab Most common fracture: vertebral body compression fractures ============================== begin estrogen hormone replacement (B), as this would increase her risk of endometrial hyperplasia and cancer. limit exposure to ultraviolet radiation (C), as this would increase her risk of vitamin D deficiency and osteoporosis. avoid fatty foods (A). Avoiding fatty foods is not directly correlated to the reduction of osteoporosis risk.
A 25-year-old woman presents with complaints of intermittent headache. The patient describes the headache as a band tightening around her head. Headaches are relieved with ibuprofen. Which of the following additional physical exam findings would be present given the most likely diagnosis?
Pericranial muscle tenderness Patients typically present with complaints of a mild tomoderate, nonthrobbing headache that is bilateral, with pain in the frontal and occipital regions. Physical exam findings are typically vague and include tenderness of pericranial myofascial tissues andincreased myofascial trigger points . Initial recommended acute treatment is with nonsteroidal anti-inflammatory drugs and aspirin. Amitriptyline is the recommended first-line treatment to prevent tension-type headache.
An 82-year-old man with a past medical history of hypertension and dyslipidemia presents to his primary care provider with left leg pain. He reports a severe, burning pain in his left leg when he walks. The pain is relieved by rest. The physical exam reveals a 1+ posterior tibial pulse and no hair on the left foot. Which of the following is the most likely diagnosis?
Peripheral arterial disease cramping or tightening of muscles in the extremities with activity that is relieved with rest. As the disease progresses, patients may report limb pain at rest and numbness and tingling, which is a sign of limb-threatening ischemia. Lateral malleolus ulcers are also a sign of poor tissue perfusion. Peripheral Artery Disease Patient presents with pain in the affected extremity related to activity (intermittent claudication) PE will show cool extremity with absent or diminished pulses Diagnosis is made by ankle-brachial index (ABI)If limb is threatened: contrast arteriography (gold standard) Most commonly caused by atherosclerotic disease
A 68-year-old woman presents with concerns about a feeling of "fullness" in the vaginal area. She notes associated leakage of urine with sneezing or coughing for the last year. She works as a delivery person and is frequently lifting heavy objects. A pelvic exam shows a smooth mass protruding from the introitus with vaginal rugae that worsens with Valsalva. Which of the following is the most appropriate intervention at this time? ABiopsy the mass BPessary placement CPrescribe oxybutynin DRefer for surgical correction
Pessary placement dx:cystocele What exercise is recommended for urinary incontinence associated with cystocele? Answer: Kegel exercises.
A 46-year-old man presents to his primary care provider for a follow-up on his laboratory results. His fasting glucose level is 232 mg/dL, and his hemoglobin A1C is 9.2%. His past medical history is significant for a treated hepatitis B infection and diabetes mellitus type 2. Which of the following should be recommended to the patient at this time?
Pneumococcal vaccine #health maintanence Patients with diabetes should receive a yearly influenza vaccination, the pneumococcal vaccine, and the hepatitis B vaccine if previously unvaccinated. under 65 years of age, either pneumococcal conjugate vaccine 15 or 20 may be used dx: type 2 diabetes mellitus. Diabetes mellitus is characterized by a fasting plasma glucose ≥ 126 mg/dL, a random plasma glucose ≥ 200 mg/dL, a hemoglobin A1C level ≥ 6.5%, or a plasma glucose level ≥ 200 mg/dL 2 hours after an oral glucose load is given. ophthalmology follow-up at the time of diagnosis to evaluate for diabetic retinopathy. Routine foot examinations should be conducted, as patients with diabetes are at higher risk for peripheral neuropathy, decreased sensation of the distal extremities, and unknown infections. Screening for diabetic nephropathy with a urine albumin to creatinine ratio is also indicated at the time of diagnosis.
A 52-year-old man with a history of diabetes and hypertension presents to the clinic complaining of a productive cough and shortness of breath that he has had for the past 3 days. He also notes intermittent fevers and chills. His wife is concerned because he has been more tired than usual. She states he has been sleeping on the couch all day. A chest X-ray shows significant consolidation to the right middle and lower lobe. Which of the following would most likely be found on this patient's physical exam? tx outpatient vs inpatient
Positive egophony dx:community-acquired pneumonia physical exam= increased tactile fremitus, dullness to percussion, inspiratory crackles, bronchial breath sounds, and positive egophony Patients with community-acquired pneumonia present with fevers, chills, shortness of breath, productive cough, and possibly pleuritic chest pain. On their vitals, they would possibly have a fever, tachycardia, and tachypnea. treatment outpatients should be treated with either a macrolide, such as azithromycin, erythromycin, or clarithromycin, or with doxycycline. Patients with a significant past medical history who are being treated as outpatients or those being treated as inpatients should be treated with either a fluoroquinolone alone or with a combination of a beta-lactam plus either a macrolide or doxycycline.
A 7-year-old boy presents to his pediatrician with sudden-onset hematuria, periorbital edema, and peripheral edema. His father notes he has only urinated twice in the past 36 hours. The father states his son has not had any illnesses in the past year other than a skin infection 2 weeks ago. Laboratory testing is significant for low serum complement C3 and C4. Urinalysis is significant for azotemia and macroscopic hematuria. Which of the following is the most likely diagnosis?
Poststreptococcal glomerulonephritis sudden onset of hematuria, mild proteinuria, and red blood cell casts secondary to glomerular inflammation. streptococcal pharyngitis, scarlet fever, or a skin infection caused by group A Streptococcus infection. Patients with poststreptococcal glomerulonephritis typically present with hypertension, periorbital edema, peripheral edema, and oliguria. Poststreptococcal glomerulonephritis is treated with antibiotics, such as penicillin or erythromycin if the streptococcal infection is still present. Supportive care includes management of hypertension and volume overload if present. Loop diuretics (e.g., furosemide) and oral nifedipine ================================================ IgA nephropathy (C), also known as Berger disease, is the most common cause of acute glomerulonephritis worldwide. It is characterized by gross hematuria, flank pain, rash, arthralgias, and abdominal pain and most often affects young men within 2 days following an upper respiratory or gastrointestinal infection.
The patient reports she has experienced numerous struvite stones in the past after urinary tract infections. What type of bacteria would you expect to see in her culture?
Proteus mirabilis How many colony-forming units of a single organism on urine culture are suggestive of a urinary tract infection? Answer: 105.
A 32-year-old man presents to the clinic with acute onset of persistent vertigo for the past 2 days. He also complains of nausea, vomiting, hearing loss, and tinnitus. The symptoms are constant. Neurologic examination reveals a horizontal nystagmus that is suppressed with visual fixation, with no focal neurologic deficits. Which of the following is the recommended treatment?
Prednisone dx: labrithitis sudden onset of acute and persistent vertigo, nausea and vomiting, nystagmus, hearing loss, and tinnitus. Patients may also complain of gait ataxia The treatment for vestibular neuritis involves corticosteroids in patients who do not have contraindications. A prednisone 10-day taper is often the steroid administered. Symptomatic treatments are also used to reduce vertigo, nausea, and vomiting. These treatments include antihistamines (meclizine or diphenhydramine), benzodiazepines (alprazolam or diazepam), and antiemetics (ondansetron or metoclopramide)
A 32-year-old woman is hospitalized for bilateral pulmonary embolisms following a period of immobilization after suffering a femur fracture. She has a family history of blood clots. Her medical team makes the decision to start her on warfarin while first bridging her with heparin. Several days into her admission, the heparin is discontinued. The patient develops large, demarcated violet areas of necrotic skin tissue on her breasts. What hypercoagulable disorder should be considered given the course of the patient's illness?
Protein C deficiency dx: Warfarin-induced skin necrosis What can cause reduced levels of protein C, besides protein C deficiency? Answer: Liver disease, ongoing infection, uremia, malignancy, ongoing chemotherapy, disseminated intravascular coagulation, use of vitamin K antagonists, and vitamin K deficiency. Patients with protein C deficiency have an increased risk of venous thromboembolism (VTE), particularly deep vein thrombosis in warfarin-induced skin necrosis due to the transient hypercoagulable state secondary to depletion of protein C that occurs when first starting warfarin. Neonatal purpura fulminans can also be observed. treatment : warfarin should be discontinued immediately. The warfarin antagonist, vitamin K, should be administered, and unfractionated heparin can be given to continue anticoagulation. Additionally, protein C should be administered to the patient either via protein C concentrate or through fresh frozen plasma.
A 45-year-old man presents with complaints of fever and painful urination. The patient reports he has been experiencing pain and burning with urination for about 7 days. The patient did not take his temperature at home but complains of subjective fever and chills. Vital signs are BP 120/80 mm Hg, pulse 110 bpm, temperature 101.2°F, and oxygen saturation 98%. Physical exam reveals costovertebral angle tenderness. A urinalysis shows blood 2+, nitrites 2+, leukocyte esterase 3+, and bacteria 3+. Urine culture is pending. What is the most likely diagnosis?
Pyelonephritis
A 25-year-old woman presents after experiencing a depressed mood, loss of pleasure in almost all activities, weight loss, poor concentration, and insomnia for the last 2 weeks. Prior to this, she had a period of euphoric mood for 10 days with associated inflated sense of self, decreased need for sleep, and racing thoughts. She was unable to work because of the severity of these symptoms. She reports no substance use, and her medical history is benign. Which of the following would be the most appropriate FDA-approved treatment for this patient at this time? ACarbamazepine BLamotrigine CLithium DQuetiapine
Quetiapine dx:Bipolar I disorder What is the first-line treatment for bipolar disorder in pregnant patients? Answer: Lamotrigine. First-line treatment for bipolar major depression (a major depressive episode in either bipolar I disorder or bipolar II disorder) includes the FDA-approved medications quetiapine or lurasidone as monotherapy. If this treatment is ineffective, second-line treatment options include olanzapine plus fluoxetine (also FDA approved), valproate, or combination therapy with a first-line agent plus lithium or valproate. Third-line treatment regimens include monotherapy with lamotrigine, lithium, olanzapine, or carbamazepine. Third-line combination therapy regimens include olanzapine plus lithium or valproate, lithium plus a selective serotonin reuptake inhibitor, or a second-generation antipsychotic (i.e., quetiapine or olanzapine) plus an antidepressant. Patients who are refractory to drug therapy may be referred for electroconvulsive therapy. Medications are chosen based on the patient profile and the side effect profile. ------------------------------------------------------- Carbamazepine (A) is an older mood stabilizer that is not generally a first-line treatment of choice due to its many drug-drug interactions, and it is not FDA-approved for bipolar depression. Lamotrigine (B) is indicated for maintenance treatment for bipolar disorder but is not FDA approved for acute bipolar depression, and there is a risk of Stevens-Johnson syndrome with this medication. Lithium (C) is approved for the treatment of acute mania, mixed mood episodes, and maintenance treatment in bipolar disorder, but it is not FDA approved for bipolar depression.
1)First-line treatment for acute uncomplicated cystitis involves a short course of sulfamethoxazole-trimethoprim twice daily for 3 days or nitrofurantoin 100 mg twice daily for 5 days. Acute uncomplicated cystitis describes a bacterial infection of the bladder Pregnancy: asymptomatic bacteriuria should be treated Complications: ↑ risk of preterm birth, low birth weight, perinatal mortality 2)What are some complications of untreated acute pyelonephritis? Answer: Kidney abscess, chronic pyelonephritis, and urosepsis. 3)Which virus causes a flesh-colored, cauliflower-like genital lesion? Answer: Human papillomavirus. 4)What is the first-line treatment for major depressive disorder? Answer: A selective serotonin reuptake inhibitor (SSRI). 5)Which vaccinations are recommended for patients aged > 65? Answer: Varicella, zoster, and influenza. 6)Varenicline (D) is an available treatment option for this patient. Varenicline has the highest cessation rate (roughly 30%). It should be used with caution in patients with manic depression, schizophrenia, or alcohol use disorder. 7)When do nicotine withdrawal symptoms peak? Answer: 3 days. 8)A fixed, split S2 sound (A) is commonly caused by a right bundle branch block or an atrial septal defect. 9)What intravenously administered nutritional supplement has been shown to improve functional outcomes in patients with heart failure? Answer: Iron. 10)True or false: eosinophilia is a common finding in Hodgkin lymphoma. Answer: True. Increased production of chemokines induces eosinophilia. 11)When do chlamydial and gonococcal neonatal conjunctivitis generally occur? Answer: Chlamydial generally occurs between 5-14 days of life, and gonococcal generally occurs between 2-5 days of life. 12)What is the bacterial spirochete responsible for Rocky Mountain spotted fever? Answer: Rickettsia rickettsii. 13)
Q/A 10
1)What is the appropriate treatment for a patient with acute stress disorder? Answer: Psychotherapy. 2)At what point of gestation is chorionic villus sampling typically performed? Answer: 10-13 weeks gestation. 3)What are the three FDA-approved medications for treatment of fibromyalgia?***** Answer: Duloxetine, pregabalin, and milnacipran. 4)Which culture medium is used to isolate Neisseria gonorrhoeae from a throat swab? Answer: Thayer-Martin medium. 5)Which two human papillomavirus types are most commonly associated with condyloma acuminata? Answer: Human papillomavirus types 6 and 11. 6)What are potential complications of untreated exophthalmos? Answer: Corneal dryness and damage, keratoconjunctivitis, compression of the optic nerve or optic artery, and blindness. 7)What form of lymphoma is most closely associated with Epstein-Barr virus? Answer: Hodgkin lymphoma. 8)What are the metabolic abnormalities associated with tumor lysis syndrome? Answer: Hyperphosphatemia, hypocalcemia, hyperuricemia, and hyperkalemia. "PKU ca" 9)Which antibiotics can be used to treat acute otitis media in children with anaphylactic reactions to amoxicillin? Answer: Azithromycin or clindamycin. 10)Which valvular disorder is associated with an Austin Flint murmur? Answer: Aortic regurgitation. A late diastolic murmur best heard at the apex. The sound heard is due to retrograde blood flow competing with antegrade flow from the left atrium. 11) True or false: it is important to urgently check intraocular pressures in patients with suspected open globe injuries. Answer: False, it is contraindicated. 12)What type of obesity is associated with insulin insensitivity? Answer: Visceral obesity due to the accumulation of fat in the omental and mesenteric regions. 13)Which electrolyte abnormality has the United States Food and Drug Administration associated proton pump inhibitor therapy with? Answer: Hypomagnesemia. 14)What is the most common cause of a large bowel obstruction in an older patient? Answer: Neoplasm. 15)True or false: most patients with peripheral vascular disease are asymptomatic. Answer: True.
Q/A 3
1)What is status asthmaticus? Answer: A medical emergency during which a severe acute asthma attack is not improving using traditional treatments. 2)What antibiotic used for COPD exacerbations has been shown to have anti-inflammatory properties in the lungs? Answer: Azithromycin. 3)What is the initial recommended treatment for acute invasive fungal sinusitis? Answer: IV amphotericin B and surgical debridement. 4)What can cause reduced levels of protein C, besides protein C deficiency? Answer: Liver disease, ongoing infection, uremia, malignancy, ongoing chemotherapy, disseminated intravascular coagulation, use of vitamin K antagonists, and vitamin K deficiency. 5)Which criteria are used to identify patients who are at increased risk for Lynch syndrome? Answer: The Amsterdam criteria. 6)True or false: aseptic bursitis can be caused by gout. Answer: True. 7)Which class of medications used for the treatment of osteoporosis has been associated with osteonecrosis of the jaw? Answer: Bisphosphonates. 8)What inflammatory syndrome involving the Glisson capsule can occur in patients with pelvic inflammatory disease secondary to Chlamydia trachomatis and Neisseria gonorrhoeae? Answer: Fitz-Hugh-Curtis syndrome (perihepatitis). 9)What is the mechanism of action of saxagliptin? Answer: Saxagliptin is a dipeptidyl peptidase-4 inhibitor that prolongs incretin levels and decreases glucagon secretion. 10)Which atypical organism is associated with exposure to livestock and transaminitis? Answer: Coxiella burnetii. 11)Conductive hearing loss associated with cholesteatoma is most commonly due to erosion of which of the ossicles? Answer: The distal portion of the incus.
Q/A 4
1)What class of medications is recommended as the initial treatment for symptomatic benign prostatic hyperplasia? Answer: Alpha-1 adrenergic antagonists (e.g., tamsulosin, terazosin). 2)What physical exam findings are consistent with a pleural effusion? Answer: Dullness to percussion, decreased tactile fremitus, and decreased breath sounds. 3)What is a complication of folliculitis that causes chronic lesions to the head and neck that are refractory to treatment? Answer: Sycosis. 4)What is mild cognitive impairment? Answer: A decline in memory or cognition that is noticeable to the patient or family members but does not significantly interfere with activities of daily living. 5)What are the five main types of specific phobias? Answer: Animal, natural environment, blood-injection-injury, situational, and other (including loud sounds, choking, and vomiting). 6)What is the first-line treatment for benign esophageal strictures? Answer: Esophageal dilation. 7)Which class of medication is montelukast? Answer: Leukotriene inhibitor. 8)What are macrophages called after ingesting LDL at the site of a fatty streak? Answer: Foam cells. 9)What is metamorphopsia? Answer: Distortion of straight lines, such as perceiving doors or blinds as curved. Metamorphopsia can be seen in wet macular degeneration. 10)What is finger agnosia? Answer: The loss of the ability to recognize one's own fingers, the fingers of others, and any drawing or representation of fingers. 11)What is the most common side effect of colchicine? Answer: GI upset including diarrhea, nausea, vomiting, and abdominal cramping. 12): What anatomical structure is infected in preseptal cellulitis? Answer: Anterior portion of the eyelid. 13)What are some common complications of Crohn disease? Answer: Perianal disease, including fistulas, strictures, granulomas, and abscesses, as well as iron and vitamin B12 deficiencies secondary to malabsorption. 14)Which disorder is characterized by progressive unilateral sensorineural hearing loss, disequilibrium, and facial numbness secondary to compression of the vestibulocochlear nerve? Answer: Vestibular schwannoma, also known as acoustic neuroma. 15)What is the most common cause of tricuspid stenosis? Answer: Rheumatic heart disease. the most common symptoms include dyspnea on exertion or decreased exercise tolerance, exertional dizziness (presyncope) or syncope = aortic stenosis 16)Which pathogen is responsible for pneumonia with mild symptoms and is commonly called walking pneumonia? Answer: Mycoplasma pneumoniae. Streptococcus pneumoniae will produce a rust-colored sputum, Haemophilus influenzae and Pseudomonas aeruginosa will produce a green sputum, and Klebsiella pneumoniae will produce a red currant-jelly sputum. Of these, Streptococcus pneumoniae is the most common causative pathogen. Streptococcus pneumoniae is an alpha-hemolytic, gram-positive bacterium that is also the most common causative pathogen of otitis media, bacteremia, and meningitis. 17)Which orthopedic condition is associated with ulnar deviations, swan-neck deformity, and boutonnière deformity of the hand? Answer: Rheumatoid arthritis. 18)What is canalithiasis? Answer: Calcium debris in the semicircular canal. 19)What medication regimen is administered to pregnant patients with pelvic inflammatory disease? Answer: Intravenous ceftriaxone and azithromycin (instead of doxycycline). 20)True or false: non-Hodgkin lymphoma is more common than Hodgkin lymphoma. Answer: True. 21)What condition presents with an expiratory monophonic wheeze best heard over larger airways? Answer: Tracheomalacia. 22)What form of cellulitis is characterized by a bright red plaque with raised and sharply demarcated borders? Answer: Erysipelas. 23)What antidepressant should be avoided in patients with eating disorders? Answer: Bupropion, due to lowering of the seizure threshold. 24)What is second-line treatment for persistent anal fissures? Answer: Topical nitroglycerin or nifedipine ointment. 25)What sign refers to localized urticaria caused by stroking skin lesions seen in cutaneous mastocytosis? Answer: Darier sign.
Q/A 5 (image leukemia)
1)At what age should the quadrivalent meningococcal conjugate vaccine be administered? Answer: 11-12 years of age. A booster can be provided at age 16. 2)True or false: bronchodilators may aid in the removal of an aspirated foreign body. Answer: False. Bronchodilators are sometimes used erroneously in patients who present with a wheeze from a foreign body aspiration. 3)What condition is a kyphoplasty used to treat? Answer: Vertebral compression fracture. 4)When do women with von Willebrand disease typically present with menorrhagia? Answer: Near menarche or at menopause. 5)Which of the following represents appropriate health maintenance for this patient, according to the U.S. Preventive Services Task Force? age 50 years and repeating the screening every 2 years until the patient is 74 years old Patients at high risk for breast cancer should receive annual magnetic resonance imaging of the breasts. 6)What is the name of the horizontal lines on a chest radiograph resulting from interstitial edema that may be seen in heart failure patients? Answer: Kerley B lines. 7)True or false: visual, auditory, or somatosensory hallucinations may accompany delirium. Answer: True. 8)An 18-year-old woman presents with her parents who are concerned about her dietary habits. She has been binge eating and then either vomiting or using laxatives as a compensatory behavior for the past several months. Which of the following historical findings would be consistent with the most likely diagnosis? Specific phobia disorder dx= Bulimia nervosa 9)What emetic is often misused by patients with bulimia nervosa? Answer: Syrup of ipecac. 10)Percutaneous transluminal coronary angioplasty definitive tx of CAD What is the gold standard diagnostic test for coronary artery disease? Answer: Coronary angiography. 11)What are paraneoplastic syndromes? Answer: They are organ dysfunctions associated with the immune-mediated or secretory effects of neoplasms and include Cushing syndrome, hypercalcemia, digital clubbing, syndrome of inappropriate antidiuretic hormone secretion (SIADH), and Eaton-Lambert syndrome. 12)What serum level of bilirubin is associated with jaundice? Answer: ≥ 3 mg/dL. 13) What is the definition of binge drinking? Answer: Drinking enough that the blood alcohol concentration levels reach 0.08 g/dL within 2 hours. 14)Which grade of hepatic encephalopathy is characterized by asterixis? Answer: Grade 2. 15)What is the recommended procedural treatment for patients with refractory grade II internal hemorrhoids who are on anticoagulants? Answer: Sclerotherapy because the risk of bleeding is very low. 16)Symptomatic ganglion cysts may require treatments such as immobilization with a brace, needle aspiration, or surgical excision. Needle aspiration is initially preferred over surgical therapy but is associated with a 50% recurrence within 1 year. Surgical excision of the cyst and stalk is the definitive treatment and is indicated in patients with physical impairment or who are symptomatic despite immobilization and aspiration. 17)What condition secondary to ligamentous injury is suspected when the Terry-Thomas sign is seen on wrist X-ray? Answer: Scapholunate dissociation. 18)What is the most common type of lung cancer? Answer: Non-small cell lung cancer. 19)Which heart valve is most affected by infective endocarditis in intravenous drug users? Answer: Tricuspid valve.
Q/A 6
1)Where in the body is follicle-stimulating hormone (FSH) synthesized? Answer: The gonadotropic cells of the anterior pituitary gland. 2)What are the most common causes of neurotrophic ulcers? Answer: Diabetes mellitus, chemical burns, and overuse of topical eye anesthetics. 3)What is the most common isolated focal dystonia? Answer: Cervical dystonia. 4)Vasomotor rhinitis, a common cause of clear rhinorrhea in older patients, is associated with an increased sensitivity of which nerve? Answer: The vidian nerve. 5)What syndrome is characterized by splenomegaly, anemia, neutropenia, thrombocytopenia, and severe rheumatoid arthritis? Answer: Felty syndrome. 6)True or false: an ultrasound on a patient with plantar fasciitis will typically show a plantar fascia thickness > 4 mm. Answer: True. 7)What is the name of the pulmonary physical exam test performed by placing the ulnar aspects of both hands against the patient's chest while the patient says the number 99? Answer: Tactile fremitus. 8)What is balanoposthitis? Answer: Inflammation of the glans penis and the foreskin. 9)What diagnosis are positively birefringent, rhomboid-shaped crystals in synovial fluid consistent with? Answer: Pseudogout, also known as calcium pyrophosphate deposition. 10)What disorder is a severely elevated triglyceride level (> 1,000 mg/dL) a risk factor for? Answer: Pancreatitis. 11)What medication improves intermittent claudication in patients with peripheral artery disease? Answer: Cilostazol. 12)What other dermatological conditions can tazarotene cream 0.1% be used to treat? Answer: Psoriasis, acne, and wrinkles. 13)What procedure is also known as a pancreaticoduodenectomy and is indicated for treatment of pancreatic cancer? Answer: Whipple procedure. 14)What is the average age of menopause in U.S. women? Answer: Around age 50. 15)Which Shigella species is associated with the development of reactive arthritis? Answer: Shigella flexneri. 16)What are some complications of acute paronychia in immunocompromised patients? Answer: Eponychia, tenosynovitis, osteomyelitis, and felon formation. 17)What are the four types of trigeminal autonomic cephalgias? Answer: Cluster headache, paroxysmal hemicrania, short-lasting unilateral neuralgiform headache attacks, and hemicrania continua. Trigeminal neuralgia (D) does not present with autonomic symptoms. It presents as a sudden, severe stabbing pain that occurs unilaterally in one or more divisions of the trigeminal nerve. Pain frequently begins near the mouth and shoots towards the eye, ear, or nostril on the same side. Triggers include light touch, talking, cold air, and facial movement including talking and chewing. 18)What gastrointestinal disorder is autoimmune pancreatitis commonly associated with? Answer: Celiac disease.
Q/A 7
1)What would an ECG for a patient with a coarctation of aorta show? Answer: Left ventricular hypertrophy. 2)What are the absolute contraindications to administering alteplase in the treatment of an acute ischemic stroke? Answer: Intracranial hemorrhage on CT; neurosurgery, head trauma, or stroke within the past 3 months; uncontrolled hypertension (> 185/110 mm Hg); history of intracranial hemorrhage; known arteriovenous malformation, aneurysm, or neoplasm; suspected or confirmed endocarditis; known bleeding diathesis; and glucose < 50 mg/dL. 3)What are the six Ps of an acute arterial embolism? Answer: Paresthesia, pain, pallor, pulselessness, paralysis, and poikilothermia. 4)What class of medication used to treat otitis externa should be avoided in the presence of tympanic membrane perforation? Answer: Aminoglycosides. 5)Which of the radiographic stages used to classify lung involvement in sarcoidosis is associated with the least favorable prognosis? Answer: Stage 3. 6)What organisms make up the HACEK group of organisms? Answer: Haemophilus, Actinobacillus, Cardiobacterium, Eikenella, and Kingella. 7)hat are the two most common relaxation-based strategies used for insomnia disorder? Answer: Progressive muscle relaxation and diaphragmatic breathing. 8) Which orthopedic test detects tightness of the iliotibial band, tensor fascia lata, and gluteus maximus? Answer: Ober test. 9)What is the preferred imaging study for appendicitis in pregnant women? Answer: MRI of the abdomen and pelvis. Diagnosis is made by CT (adults), ultrasound (pediatric or pregnant patients), MRI (pregnant patients with nondiagnostic ultrasound) Most commonly caused by fecolith (fecalith) 10)What are the blood pressure treatment goals for patients with concurrent diabetes mellitus and hypertension, according to the American Diabetes Association? Answer: Systolic blood pressure under 140 mm Hg and diastolic blood pressure under 90 mm Hg. 11)How long should outpatient oral antibiotic treatment be continued when treating pelvic inflammatory disease? Answer: For 14 days 12)What pear-shaped protozoan has four flagella at its anterior end and can be seen on urine microscopy? Answer: Trichomonas vaginalis. 13)Which test can be used to confirm the diagnosis of trichomoniasis? Answer: Nucleic acid amplification testing. 14)What is Budd-Chiari syndrome? Answer: A disorder that occurs when the hepatic venous outflow is obstructed due to a thrombus within a hepatic vein. 15)What supplement should be initiated in patients with chronic hemolysis? Answer: Folic acid. 16)What is the typical number of leads on a traditional Holter monitor? Answer: Three (although 12-lead monitors are available). 17)What are the alarm features that suggest an alternative diagnosis to irritable bowel syndrome? Answer: More than minimal rectal bleeding, weight loss, unexplained iron deficiency anemia, nocturnal symptoms, and family history of colon cancer, inflammatory bowel disease, or celiac disease. 18)What amount of blood loss during a menstrual cycle defines heavy menstrual bleeding? Answer: > 80 mL.
Q/A 9
1)Which cytoplasmic antibodies are associated with microscopic polyangiitis? Answer: Perinuclear antineutrophil cytoplasmic antibodies (pANCA). 2)What condition is often associated with placenta previa, particularly with the increasing number of cesarean deliveries, and should be excluded? Answer: Placenta accreta spectrum. 3)Does pneumoconiosis typically present with an obstructive or restrictive pattern on pulmonary function testing? Answer: Restrictive pattern. 4)True or false: the mortality rate for patients with a small bowel obstruction and ischemic bowel is 100% if left untreated. Answer: True. 5)What is Wallenberg syndrome? Answer: A lateral medullary infarction resulting in an array of manifestations, including dizziness, difficulty sitting upright, diplopia, nystagmus, hypotonia and limb ataxia of the ipsilateral arm, and loss of pain and temperature sensation in the ipsilateral face and contralateral trunk and limbs. 6)You are concerned the patient may have salmonellosis. Reported ingestion of what food would raise your suspicion even higher?poultry and eggs. Common cause of osteomyelitis in children with sickle cell disease 7)Which virus is associated with outbreaks of gastroenteritis on cruise ships? Answer: Norovirus. 8)What is Pickwickian syndrome? Answer: Hypoventilation due to obesity. 9)What endocrine disorder is strongly associated with HLA-DR3 and HLA-DR4? Answer: Type 1 diabetes mellitus. 10)What are the possible ocular sequelae of retinal artery occlusion? Answer: Vision loss, vitreous hemorrhage, and neovascular glaucoma. 11)
Q/A infinity
A 64-year-old woman presents to the office due to some bumps on her forearms that have been present for several months and have not resolved. She reports no pain or itching to the lesions. Upon physical exam, she has multiple light brown to dark brown macules and papules that have well-defined borders. The smallest ones are about 0.5 cm in diameter and the largest are about 1.5 cm in diameter. Some have a "stuck on" appearance and are nodular and rough to touch. Which of the following is the recommended treatment for this condition?
Reassurance and observation dx:Seborrheic keratosis Seborrheic Keratosis Risk factors: advancing age PE: velvety, waxy lesions with stuck-on appearance on the face, shoulders, chest, and back Caused by a benign, epidermal neoplasm Treatment is not necessary Leser-Trelat sign: rapid appearance of multiple SK lesions associated with GI malignancy What other dermatological conditions can tazarotene cream 0.1% be used to treat? Answer: Psoriasis, acne, and wrinkles.
A 42-year-old woman presents to the clinic complaining of nausea, vomiting, and diarrhea for the past 2 days. She has never experienced this before. She takes no daily medications and is not pregnant. She states she has six unformed, nonbloody stools per day with three or four episodes of emesis daily. Vital signs are within normal limits, and physical exam reveals good turgor, hyperactive bowel sounds, and mild, diffuse abdominal tenderness to palpation. Which of the following physiologic mechanisms is likely responsible for this patient's diarrhea?
Reduced absorption of electrolytes across the intestinal epithelium dx: Acute gastroenteritis nausea, vomiting, fever, or abdominal pain. Most cases of acute gastroenteritis are of viral etiology, with norovirus being the most common culprit.
A 19-year-old man presents to his primary care provider with recurrent episodes of mild intermittent jaundice. He states these episodes began several years ago and seem to recur when he is stressed. He reports no other symptoms. Physical examination reveals mild scleral icterus and yellowing of the skin. Laboratory testing is significant for a slightly elevated indirect bilirubin. Which of the following best describes the pathophysiology of the most likely diagnosis?***
Reduced production of uridine 5'-diphospho-glucuronosyltransferase dx: Bilirubin What serum level of bilirubin is associated with jaundice? Answer: ≥ 3 mg/dL. Gilbert syndrome is the most common inherited disorder of bilirubin glucuronidation. It is a benign, autosomal recessivecondition characterized by reduced production of uridine 5'-diphospho-glucuronosyltransferase that results in elevated unconjugated, or indirect, bilirubin levels. Patients experience episodic jaundice that may be triggered by periods of stress, illness, physical exertion, hemolysis, fasting, menses, or alcohol use. Physical examination during an acute episode will show scleral icterus and jaundice of the skin. Diagnosis can be made by laboratory tests showing repeatedly elevated indirect bilirubin levels in the presence of normal complete blood counts, liver enzymes, and alkaline phosphatase. Definitive diagnosis is made with genetic testing. No treatment is needed for Gilbert syndrome. =================================================== . Causes of unconjugated hyperbilirubinemia include hemolysis, Gilbert syndrome, Crigler-Najjar syndrome, and drug reactions. Causes of conjugated hyperbilirubinemia include Dubin-Johnson syndrome, Rotor syndrome, intrahepatic cholestasis, hepatitis, cirrhosis, sepsis, and biliary obstruction (e.g., choledocholithiasis, sclerosing cholangitis, pancreatitis). Cholestasis (A) is a condition in which there is retention of bile in the liver. Conjugated hyperbilirubinemia can occur from cholestasis. Intrahepatic cholestasis of pregnancy is usually seen in the third trimester. Reduced excretory function of hepatocytes (B) describes the pathophysiology of Dubin-Johnson syndrome. Dubin-Johnson syndrome is a benign, autosomal recessive condition characterized by conjugated hyperbilirubinemia. Characteristic findings associated with Dubin-Johnson syndrome are an inability to visualize the gallbladder on oral cholecystography and the presence of a black liver caused by lack of multidrug resistance-associated protein 2. Reduced hepatic reuptake of bilirubin conjugates (C) describes the pathophysiology of Rotor syndrome. Rotor syndrome is a benign, autosomal recessive condition with conjugated hyperbilirubinemia. It is similar to Dubin-Johnson syndrome except the gallbladder can be visualized on oral cholecystography and there is no black liver.
A 45-year-old woman presents to her primary care provider with complaints of worsening right shoulder pain and stiffness for 2 months. She describes the pain as diffuse, dull, and achy. Her pain is worse at night and with overhead movements. She has been taking ibuprofen, which only alleviates the pain occasionally. Her past surgical history is significant for a rotator cuff repair 3 months ago. Physical examination reveals a well-developed, well-nourished woman with her right arm in a sling. Passive range of motion of the right shoulder is decreased in all planes and causes the patient significant pain. Grip strength and neurovascular testing are normal bilaterally. Which of the following is the most appropriate clinical intervention at this time?
Refer to physical therapy dx:Adhesive capsulitis, Patient presents with diffuse shoulder pain, with progressive stiffness that results in severe range of motion limitations Treatment is gentle range of motion exercise that increases in intensity, intra-articular corticosteroid injections for severe symptoms women between 40-60 years old. Risk factors include diabetes mellitus, immobilization of the shoulder joint following injury or surgery, thyroid disease, autoimmune disease, stroke, and protease inhibitors What nerve passes through the quadrangular space of the shoulder? Answer: The axillary nerve.
A 45-year-old man presents with complaints of bilateral shaking in his hands for over 5 years. The patient is concerned because it is becoming obvious when he points at the board during his lectures. Which of the following additional findings is most consistent with the suspected diagnosis?
Relief of symptoms with alcohol dx:essential tremor. Patients present with a high-frequency postural or action tremor (6-12 Hz) that preferentially affects the upper extremities. First-line treatment is with propranolol. Primidone can also be used if the patient does not respond to propranolol or is unable to tolerate it. The two medications can also be used in combination. Second-line treatment options include gabapentin, topiramate, or nimodipine. What is the most common isolated focal dystonia? Answer: Cervical dystonia.
A 50-year-old man presents to the office for evaluation of progressive dyspnea on exertion and a persistent nonproductive cough. Symptoms began about 6 months ago. He has no significant medical history other than atrial fibrillation, which has been well-controlled on amiodarone for the past 12 months. He does not take any other medications. He reports no history of tobacco or illicit drug use or exposure to occupational agents. On physical examination, his heart rate and rhythm are regular, and no murmur or gallop is heard. Inspiratory crackles are noted diffusely in both lungs. His jugular venous pressure is normal, and he has no peripheral edema. Which of the following findings is most likely to be seen on chest radiography?
Reticular opacities dx:interstitial lung disease exposure to occupational and environmental agents, drug-induced pulmonary toxicity, and radiation-induced lung injury. One medication that is notorious for causing interstitial lung disease is the antidysrhythmic agent amiodarone. Patients with interstitial lung disease often report progressive dyspnea on exertionand a persistent nonproductive cough. Lung examination often reveals crackles. Clubbing of the digits may also be noted on physical examination. Patients with interstitial lung disease display lung function abnormalities on spirometry, particularly a restrictive ventilatory pattern (reduced forced vital capacity and total lung capacity with a normal or increased FEV1/FVC ratio). Chest imaging is abnormal in these patients as well. Chest X-ray may be useful in suggesting the presence of interstitial lung disease and can be useful to rule out other differential diagnoses. The most common radiographic abnormality seen on chest X-ray among patients with interstitial lung disease is a reticular pattern. Nodular or mixed patterns may be seen as well. High-resolution computed tomography is the best diagnostic test to confirm interstitial lung disease.
A 70-year-old man with a 50 pack-year history of cigarette smoking is being evaluated for possible community-acquired pneumonia. He has been coughing for 2 days but is currently afebrile. Physical exam is unremarkable. His chest radiograph reveals a single, well-circumscribed nodule, approximately 1 cm in diameter, at the periphery of the right lung. Which of the following is the best next step in management? AComputed tomography scan BPositron emission tomography scan CRepeat chest imaging in 3 monthsYour Answer DReview of a previous chest radiograph
Review of a previous chest radiograph ( image distinguishes benign vs maliganant ) What are the differential diagnoses of a pulmonary nodule? Answer: Infectious granuloma, bronchogenic carcinoma, hamartoma, malignancy, and bronchial adenoma.
A 44-year-old man presents to his primary care provider for a follow-up on his lipid panel results. His lipid panel results are: Total cholesterol: 320 md/dL Low-density cholesterol: 240 mg/dL High-density cholesterol: 30 mg/dL Triglyceride: 250 mg/dL The patient has a history of diabetes mellitus type 2, and his 10-year atherosclerotic cardiovascular disease risk is 10.1% Which of the following medications is indicated at this time?
Rosuvastatin 20 mg The patient in the vignette above needs high-intensity statin therapy due to his diabetes, ASCVD score, and LDL > 190 mg/dL. Atorvastatin 10 mg (A), pravastatin 40 mg (B), and simvastatin 20 mg (D) are all moderate-intensity statin therapy regimens. ------------------------------------------------------- . High-intensity statin regimens aim to reduce LDL by ≥ 50% and include atorvastatin 40-80 mg or rosuvastatin 20-40 mg. Moderate-intensity statin regimens aim to reduce LDL by 30-50% and include atorvastatin 10-20 mg, rosuvastatin 5-10 mg, simvastatin 20-40 mg, and pravastatin 40-80 mg.
A 29-year-old woman presents to the office with a complaint of progressive fatigue over the past 3 months. She reports intermittent low-grade fevers and night sweats on rare occasions. Chart review from her last visit indicates an unintentional 16-pound weight loss over the past 3 months. On physical examination, the left deep cervical and supraclavicular lymph nodes are enlarged, firm, and nontender to palpation. What finding on lymph node biopsy is most consistent with the patient's most likely diagnosis?
Rounded bilobed cells with two nuclei aka Reed-Sternberg cells, a classic finding in Hodgkin lymphoma Common presenting symptoms include fatigue, lymphadenopathy (the cervical and supraclavicular lymph nodes are most commonly affected), constitutional symptoms, and pruritus. Constitutional or B symptoms include fever, night sweats, and unintentional weight loss complain of pain in sites of bony involvement following ingestion of alcohol. On chest radiograph, a mediastinal mass is often found. What is the Waldeyer ring? Answer: Pharyngeal ring of lymphoid tissue made up of the adenoid, tubal, palatine, and lingual tonsils.
A 19-year-old man presents to his primary care provider after an episode of dyspnea, chest pain, and postexertional syncope. He reports his father and brother have had similar symptoms in the past. Echocardiography is significant for left ventricular hypertrophy with a thickened septum, left ventricular hypercontractility and hypertrophy, left atrial enlargement, and diastolic dysfunction. Physical examination findings reveal a bisferiens carotid pulse, triple apical impulse, and prominent a wave. Which of the following cardiac auscultation findings is most consistent with the patient's condition?
S4 gallop dx:Hypertrophic cardiomyopathy . Physical examination findings include pulsus bisferiens (biphasic pulse due to aortic regurgitation), a triple apical impulse (prominent atrial filling with early and late systolic impulses), a prominent a wave due to reduced right ventricular compliance, and a loud S4 gallop due to a stiff left ventricle. The murmur associated with hypertrophic cardiomyopathy decreases with factors that increase the preload (e.g., squatting, hand-grip) and increases with factors that decrease the preload (e.g., standing, Valsalva maneuver) mainstay of treatment for hypertrophic cardiomyopathy is beta-blocker (e.g., metoprolol) or calcium channel blocker (e.g., verapamil) therapy.
A 24-year-old woman presents to her primary care provider with complaints of multiple episodes of shortness of breath and wheezing. Diffuse expiratory wheezing is heard on lung auscultation. Pulmonary function testing reveals an obstructive pattern. Bronchodilator administration results in an increased FEV1 of 30%. The patient reports no other medical conditions other than a drug allergy to aspirin. Which of the following additional findings completes the suspected clinical triad?
Samter triad is characterized by asthma, aspirin sensitivity, and nasal polyps. What is the mainstay of therapy for salicylate poisoning? Answer: Alkalinization with sodium bicarbonate.
26-year-old man presents to the office with frequent cough and fatigue that have been ongoing for the past few months. He reports difficulty breathing that has interfered with his ability to exercise. He notes no sick contacts or recent travel. Physical examination reveals small flesh-colored papules surrounding a tattoo on the right calf. A chest X-ray is obtained and shown above. What is the most likely diagnosis? A 38-year-old fireman presents to the clinic with complaints of a persistent, nonproductive cough and new skin lesions. He reports no fever, chills, nausea, vomiting, or any sick contacts. He has felt more fatigued than usual and reports occasional joint pain. On physical exam, you see multiple 3-5 cm erythematous, warm, painful patches and nodules on his shins. Chest radiography reveals the above findings. What is the most appropriate clinical intervention for this patient?
Sarcoidosis Transbronchial biopsy Sarcoidosis Systemic disease with unknown etiology, can affect all organ systems Sx: cough, dyspnea, fever, fatigue, skin lesions, arthralgias PE: lupus pernio (chronic, violaceous, raised plaques and nodules commonly found on the cheeks, nose, and around the eyes) is pathognomonic for sarcoidosis Labs: hypercalcemia, hypercalciuria, elevated serum ACE CXR: bilateral hilar adenopathy, possible reticulonodular opacities in upper lung fields Biopsy: noncaseating granulomas Tx: spontaneous remission may occur, oral glucocorticoids Comments: Löfgren syndrome: fever, erythema nodosum, hilar adenopathy, migratory polyarthralgia
What is the appropriate screening guideline for an abdominal aortic aneurysm according to the U.S. Preventive Services Task Force? AScreen all men ages 65 to 75 years old who have at least a 10 pack-year history of smoking with a one-time CT scan BScreen all men ages 65 to 75 years old who have ever smoked with a one-time abdominal CT scan CScreen all men ages 65 to 75 years old who have ever smoked with a one-time ultrasonography DScreen all men and women ages 65 to 75 years old who have ever smoked with a one-time ultrasonography
Screen all men ages 65 to 75 years old who have ever smoked with a one-time ultrasonography What are the two greatest interventions to reduce the risk for developing an abdominal aortic aneurysm? Answer: Smoking cessation and blood pressure control. Abdominal Aortic Aneurysm (AAA) Risk factors: male sex, older patients, smoking, HTN Abdominal pain or asymptomatic Physical exam will show pulsatile abdominal mass If ruptured: hypotension Diagnosis is made by US: excellent screening tool Management Monitor progression (Society for Vascular Surgery guidelines) 4.0-4.9 cm: US annually 5.0-5.4 cm: US every 6 months, can also use CT or MRI (MRI is preferred over time due to less radiation) Surgical repair would be indicated if the patient is symptomatic> 5.5 cm or aneurysms with rapid expansion rate: elective surgery The USPSTF recommends one-time screening for AAA by ultrasonography in men aged 65-75 who have ever smoked
A 3-year-old boy presents to the clinic with a cough and wheeze for the past 3 hours. He is in no apparent distress. The patient's mother reports he was eating peanuts and then began to cough and wheeze. The patient was not ill prior to this event and is currently taking no medications. Physical exam reveals decreased breath sounds in the right lower lung. Pulse oxygen saturation is within normal limits. Which of the following signs or symptoms would you also expect to encounter in this patient?
Segmental hyperresonance Foreign body aspiration Signs and symptoms of foreign body aspiration include tachypnea, cough, stridor, focal monophonic wheezing, regional variation in lung sounds, diminished lung sounds, choking, dyspnea, voice hoarseness, or hyperresonance to percussion over the affected area of the lung due to air trapping. Chest X-ray may show a hyperinflated lung, atelectasis, mediastinal shift, pneumonia, or a radiopaque object. True or false: bronchodilators may aid in the removal of an aspirated foreign body. Answer: False. Bronchodilators are sometimes used erroneously in patients who present with a wheeze from a foreign body aspiration.
A 4-year-old boy presents to the office with his parents, out of concern for sudden bruising, spots all over his body, and blood in his stool that developed suddenly today. There is no report of recent trauma or abuse. The patient's mother reports no fever but does report he was ill in the past month with an upper respiratory infection. On physical examination, the boy is noted to be afebrile, pale, and nontoxic-appearing. Ecchymosis is noted on the patient's legs and left arm along with the finding in the image above. No lymphadenopathy or splenomegaly is appreciated. Complete blood count reveals a platelet count of 12,000/microL. Hemoglobin and hematocrit are within normal range. Coagulation studies reveal no prolongation. What intervention would be appropriate for this patient?
Send to emergency department for further evaluation dx: Immune thrombocytopenia (ITP) Emergency evaluation and hospital admission are appropriate in patients with severe, non-life-threatening bleeding due to immune thrombocytopenia. Immune thrombocytopenia (ITP) is an immune-mediated disorder characterized by isolated thrombocytopenia with normal hemoglobin, hematocrit, and white blood cell counts. ITP is characterized by a platelet count of < 100,000/microL, but patients often have platelet counts < 30,000/microL. ITP is commonly preceded by a recent viral infection and often affects those between the ages of 2 and 5 years spring and summer months. The pathogenesis is related to the development of autoantibodies against the glycoprotein IIb/IIIA complex on platelets, resulting in hastened destruction by macrophages in the spleen. Patients presenting with ITP will often have a petechial rash, ecchymosis, or bleeding. treatment Treatment for ITP is dependent on the severity of bleeding and the degree of thrombocytopenia. Patients with life-threatening bleeding require platelet transfusions, glucocorticoids (such as methylprednisolone), intravenous immune globulin (IVIG), and anti-D immune globulin in Rh-positive patients. Romiplostim, a thrombopoietin receptor agonist, may also be given to promote platelet response. Patients with life-threatening bleeding that is refractory to medical management should undergo splenectomy.
A 29-year-old G3P1 woman at 25 weeks gestation presents to urgent care with significant painless vaginal bleeding, which began 2 hours ago. She reports no trauma. This has not occurred in her previous pregnancies. She did not have a second trimester ultrasound. Blood pressure is 92/56 mm Hg. A digital vaginal examination is deferred in favor of an abdominal ultrasound, which reveals a low-lying placenta over the cervical os. What intervention would be appropriate at this time?
Send to the hospital for admission to labor and delivery dx: placenta previa Cesarean delivery is performed between 36 weeks to 37 6/7 weeks gestation in women with asymptomatic placenta previa. Antenatal steroids should be given for deliveries before 37 weeks gestation for fetal lung development, and magnesium sulfate should be given between 24-32 weeks gestation for scheduled cesarean deliveries for neuroprotection.
A previously healthy 22-year-old woman presents to your clinic with a complaint of shortness of breath while exercising. She has a history of participation in cross-country skiing since childhood and only experiences respiratory symptoms with exercise. Which of the following is the most appropriate therapy?
Short-acting beta-agonist use before exercise dx:exercise-induced asthma and exercise-induced bronchospasm Treatment involves nonpharmacologic interventions, including adequate warm-up and cool-down before and after exercise, wearing a face mask in cold temperatures, and engaging in short-burst type exercise, such as basketball, baseball, or tennis, instead of longer-duration exercise, such as distance running. First-line therapy is with short-acting beta-agonist use before exercise to prevent onset of symptoms. True or false: montelukast is effective in treating exercise-induced asthma. Answer: True.
A 24-year-old woman presents to the office for an employment physical. She initially reports a normal diet and no concerns on evaluation. Her body mass index is 15.6 kg/m2. Lanugo is noted on examination. Follow up questioning reveals the patient has restricted her caloric intake each day out of fear of gaining weight. She reports no self-induced purging activities. What is the most common electrocardiogram finding associated with the suspected diagnosis?
Sinus bradycardia dx: Anorexia nervosa Diagnostic and Statistical Manual of Mental Disorders (DSM-IV), bradycardia, brittle hair and nails, hypotension, hypothermia, and lanugo. What antidepressant should be avoided in patients with eating disorders? Answer: Bupropion, due to lowering of the seizure threshold.
A 58-year-old man with a history of heart failure presents to the ED with worsening shortness of breath and pitting edema of the bilateral lower extremities. Vital signs are T 98.6°F, BP 202/112 mm Hg, HR 105 bpm, RR 24 breaths per minute, and oxygen saturation 94% on room air. A chest X-ray and echocardiogram show pulmonary edema and acute left ventricular dysfunction. His ECG shows sinus tachycardia with no ST segment or T wave changes. The patient is given furosemide for pulmonary edema. Which of the following blood pressure medications is most appropriate at this time?
Sodium nitroprusside The two main cardiac emergencies associated with severe hypertension are acute heart failure and acute coronary syndrome. Patients with acute heart failure often present with acute left ventricular dysfunction and pulmonary edema. . Patients with acute intracranial hemorrhage and a systolic blood pressure > 200 mm Hg should have aggressive reduction to 140-160 mm Hg. Aggressive blood pressure reduction within 20 minutes to 100-120 mm Hg is indicated in patients with acute aortic dissection.
A 45-year-old woman presents to the clinic with infrequent bowel movements. She says that for most of her adult life she has had one or two bowel movements per week and has had to strain during bowel movements. She reports no rectal bleeding, weight loss, or family history of colorectal cancer. Physical examination reveals her abdomen is soft and nontender. She has tried increasing her water and fiber intake with minimal improvement. Which of the following pharmacologic treatments would improve her condition by increasing the flow of water into the intestines? ABisacodyl BDocusate CLoperamide DSorbitol
Sorbitol dx: constipation (look at image and know the categories) Osmotic laxatives are a common class of laxatives used. These include polyethylene glycol, magnesium citrate, lactulose, and sorbitol. Osmotic laxatives work by drawing water into the intestine, which increases the frequency of bowel movements and improves the consistency of the stool. What are the possible adverse effects of using sodium phosphate enemas in older adults? Answer: Volume depletion, hypotension, hyperphosphatemia, hypokalemia, hypocalcemia, metabolic acidosis, and acute kidney injury.
A 30-year-old man presents to the clinic complaining of fatigue, night sweats, and a 20-pound unintentional weight loss in the past 4 months. He takes no daily medications and does not smoke. Physical exam reveals diffuse nontender lymphadenopathy and a temperature of 100.8°F. Which of the following additional physical exam findings is most consistent with the presumed diagnosis?
Splenomegaly dx:Hodgkin lymphoma is a disease of malignant lymphocytes True or false: eosinophilia is a common finding in Hodgkin lymphoma. Answer: True. Increased production of chemokines induces eosinophilia.
A 3-year-old boy presents with his mother for an annual exam. He is in the 98th percentile for height and the 86th percentile for weight. His mother describes him as a very active child who makes friends easily and is responsive to his parents. She reports no recent illnesses or sick contacts. He appears well-developed and well-nourished. On cardiac auscultation, there is a soft, high-pitched, crescendo-decrescendo murmur heard during midsystole along the left sternal border. What physiological maneuver would you include in the physical exam to confirm your suspected diagnosis?
Standing dx:Innocent murmurs are common childhood murmurs affecting up to 85% of children. Innocent murmurs are typically found in children between 2 and 7 years of age. There are three types of innocent murmurs: Still murmurs, pulmonic murmurs, and venous hums. Still murmurs are most common. What are the three other causes of midsystolic murmurs excluding innocent murmurs of childhood? Answer: Hypertrophic cardiomyopathy, aortic stenosis, and pulmonic stenosis.
A 75-year-old woman develops acute pain and erythema in the left preauricular area while she is hospitalized following surgery. Vital signs are T of 101.1°F, HR of 108 bpm, BP of 150/92 mm Hg, RR of 20/min, and oxygen saturation of 95% on room air. Physical examination reveals focal tenderness extending from the left preauricular area to the angle of the mandible, and there is purulent drainage from the Stensen duct. Laboratory findings reveal a white blood cell count of 14,000/microL. Which of the following pathogens is the most likely cause?
Staphylococcus aureus dx:Parotitis abrupt onset of unilateral erythematous, firm, painful swelling from the preauricular area to the angle of the mandible. The risk factors for acute bacterial suppurative parotitis include dehydration and poor oral hygiene. Patients with acute bacterial suppurative parotitis should be admitted to the hospital for hydration and intravenous antibiotics. Admission is indicated because the infection can spread to the deep fascial spaces of the head and neck and lead to life-threatening complications. treatment= Acute bacterial suppurative parotitis is frequently hospital-acquired since it is common in postoperative older patients. The recommended empiric regimen for community-acquired cases is either ampicillin-sulbactam or cefuroxime and metronidazole. Hospital-acquired infection or immunocompromised patients should be initially treated empirically with a combination antibiotic regimen of either vancomycin or linezolid and either cefepime and metronidazole, imipenem, meropenem, or piperacillin-tazobactam The oral step-down regimen should be guided by culture and susceptibility data, Surgical incision and drainage should be considered in patients who have not improved within 2 or 3 days. What are the risk factors for methicillin-resistant Staphylococcus aureus? Answer: Intravenous drug use, residence in a long-term care facility, discharge from a hospital within the preceding 12 months, and hemodialysis.
A 38-year-old man with a prosthetic heart valve presents with fever, cough, and rash. His symptoms started approximately 3 months after his prosthetic heart valve placement surgery. On the physical exam, he is noted to have a holosystolic murmur (heard best at the left lower sternal border) and splinter hemorrhages of the fingernail beds. He also has painful, erythematous papular lesions on his palms and soles. A blood culture is obtained. Which of the following organisms is most likely to be isolated?
Staphylococcus epidermidis dx:Infective endocarditis the initial implantation (< 2 months) include Staphylococcus aureus and coagulase-negative staphylococci, followed by gram-negative bacilli and Candida spp. Between 2 to 12 months after implantation, the most common pathogens include coagulase-negative staphylococci (Staphylococcus epidermidis), S. aureus, and streptococci, followed by enterococci. ===================================== staphylococcus aureus (A) can cause early-onset prosthetic valve endocarditis, but it is not the most common cause. However, it is the most common cause of endocarditis in intravenous drug users, particularly affecting the tricuspid valve. Streptococcus bovis (C) and Streptococcus viridans (D) are the most frequent causative organisms in cases of late-onset prosthetic valve endocarditis, with symptoms typically appearing more than 12 months after surgery. Streptococcus bovis is highly associated with an increased risk of active colonic malignancy, whereas Streptococcus viridans is also the most common cause of native valve endocarditis and subacute endocarditis.
A 72-year-old man presents with bloody diarrhea for 3 days. He complains of crampy abdominal pain and eight loose stools tinged with blood per day. The patient reports no dietary changes and states his wife also has similar symptoms. His vital signs are within normal limits, and he takes no daily medications. On exam, the patient has good turgor, diffuse abdominal tenderness to palpation with no rebound or guarding, and hyperactive bowel sounds. Which of the following is the best next step in diagnosing this patient's illness? AFecal hemoccult testing BStool bacterial culture for Clostridioides difficile CStool bacterial culture for Salmonella, Campylobacter, and Shigella DStool microscopy for ova and parasites
Stool bacterial culture for Salmonella, Campylobacter, and Shigella
An 18-year-old man presents with an acute onset of sore throat and fever. He reports no coughing, nasal congestion, and hoarseness. Additionally, he reports no history of oral sex. On exam, tonsillar exudates and anterior cervical lymphadenopathy are noted. A rapid antigen detection assay is positive. Which of the following is the most likely diagnosis?
Streptococcal pharyngitis caused by group A beta-hemolytic Streptococcus. high fever (> 100.4°F), sore throat, lack of cough, and white pharyngotonsillar exudates. Patients who meet three out of the four criteria should undergo testing using a rapid streptococcal test. A negative result should be confirmed by a throat culture, which is the gold standard. Complications of streptococcal pharyngitis include peritonsillar abscess, mastoiditis, acute rheumatic fever, poststreptococcal glomerulonephritis, and rheumatic heart disease. Treatment of bacterial pharyngitis can be achieved using penicillins. Cephalexin, cefadroxil (avoid in individuals with immediate-type hypersensitivity to penicillin) Clindamycin Azithromycin, clarithromycin (resistance of group A strep to these agents is known well and varies geographically and temporally) ================================================= Viral pharyngitis (D), the most common cause of sore throat, is an inflammation of the pharynx and surrounding tissues with a viral etiology (rhinovirus, adenovirus, coronavirus, echovirus, parainfluenza). Patients would present with coryza, malaise, conjunctivitis, cough, fatigue, and low-grade fever. A rapid streptococcal antigen and throat culture would be negative. infectious mononucleosis (B)The presence of posterior cervical lymphadenopathy is highly suggestive of infectious mononucleosis, and its absence makes the diagnosis less likely.
A 23-year-old woman presents to the clinic reporting severely painful bowel movements and bright red blood per rectum for 4 days. Patient reports she is developing constipation due to fear of having a painful bowel movement. Which of the following signs on a physical exam would be most consistent with the suspected diagnosis? What is second-line treatment for persistent of current dx?
Superficial crack in the distal anal canal dx:Anal fissure reatment includes supportive measures, including sitz baths, analgesics, increased fiber and water intake, stool softeners, and laxatives. Most fissures resolve spontaneously within 6 weeks. What is second-line treatment for persistent anal fissures? Answer: Topical nitroglycerin or nifedipine ointment.
A 65-year-old man presents with a changing skin lesion on the trunk. On exam, you note a raised discolored plaque with irregular borders on the trunk. You suspect melanoma. Which of the following subtypes of melanoma is described above? AAcral lentiginous BLentigo maligna CNodular DSuperficial spreading
Superficial spreading There are four subtypes of melanoma, including superficial spreading, nodular, lentigo maligna, and acral lentiginous. Superficial spreading is the most common subtype of melanoma. It appears as a flat macule or slightly raised discolored plaque with irregular borders most frequently found on the trunk (in men) and legs (in women). Most cases of superficial spreading melanoma often arise de novo, although it is most likely to be associated with a preexisting nevus. Lesions display some or all of the following features: asymmetry, border irregularity, color variegation The diagnosis is made by an excisional biopsy and pathological examination. Treatment can be achieved by complete full-skin-depth excision using margin determination by Breslow depth (tumor thickness from the granular layer of the epidermis to the point of deepest invasion). For advanced disease, immunotherapy or targeted therapies may be effective. ====================================================== Acral lentiginous (A) predominantly affects dark-skinned individuals who are at a lower risk for more sun-related subtypes of melanoma. It frequently arises on the palmar, plantar, or subungual surfaces and typically appears first as a dark brown to black, irregularly pigmented macule or patch. When it is raised, ulcerated, bleeding, or involving a larger diameter, it may signify a deeper invasion into the dermis. Occasionally, acral melanoma may mimic benign lesions such as warts, calluses, tinea pedis, or ingrown toenail. Lentigo maligna (B) most commonly affects chronically sun-damaged areas of the skin in older individuals and often begins as a tan or brown macule. It then progresses gradually into a larger, raised, darker, asymmetric focus of pigmentation and color variegation, which can signify a vertical growth phase. Nodular melanoma (C) appears as a darkly pigmented, pedunculated, polypoid papule or nodule but has a uniform color, a symmetric border, and a relatively smaller diameter.
A 24-year-old man presents to urgent care with complaints of shortness of breath that started last night while he was sitting on the couch. The symptoms have since resolved. He also complains of some intermittent mild chest pain on his right side. The patient currently smokes one pack of cigarettes per day. Vital signs are BP 120/80 mm Hg, oxygen saturation 95%, pulse 115 beats per minute, respiratory rate 20 breaths per minute, height 5'10", and weight 160 pounds. The patient is well appearing and is able to speak in full sentences. Chest X-ray shows a visceral pleural line with the distance from chest wall to the visceral pleural line less than 2 cm at the level of the hilum. What is the most appropriate treatment for this patient?
Supplemental oxygen and observation dx pneumothorax What drug is used for the treatment of Pneumocystis pneumonia? Answer: Trimethoprim-sulfamethoxazole. A supine chest X-ray may show an increased radiolucent costophrenic sulcus, known as the deep sulcus sign. Findings of a tension pneumothorax on chest X-ray include a large amount of air in the affected hemithorax and a contralateral shift in the mediastinum. CT scan of the chest may be needed for further evaluation or if diagnosis is unclear on chest X-ray. Treatment of a first-time spontaneous mild pneumothorax (less than 15% of hemithorax) in a stable patient is typically with supplemental oxygen and observation, as it will resolve spontaneously when air is reabsorbed. Simple drainage with a small-bore catheter can be performed for larger or progressive spontaneous primary pneumothoraces. A small-bore chest tube with a one-way Heimlich valve can permit observation at home with serial chest radiographs every 24 hours. A chest tube with suction is required for patients who have large pneumothorax, tension pneumothorax, secondary pneumothorax, or those with severe symptoms. Thoracoscopy or openthoracotomy is indicated for recurrent spontaneous pneumothorax, bilateral pneumothorax, and failure of tube thoracostomy. Prevention includes smoking cessation, avoiding exposure to high altitudes, avoiding flying in unpressurized aircraft, and avoiding scuba diving. About 30% of patients with spontaneous pneumothorax will experience recurrence after observation or tube thoracostomy. There are no long-term complications with successful treatment.
A 35-year-old man presents to the urgent care with 2 weeks of a nonproductive cough. He initially had rhinorrhea, sore throat, and malaise, but these symptoms have since resolved. He reports no history of smoking cigarettes or prior chronic obstructive lung disease diagnosis. Vital signs are T 99.2°F, HR 84 bpm, BP 120/80 mm Hg, RR 20 breaths per minute, and oxygen saturation 98% on room air. Physical exam reveals clear breath sounds. Chest X-ray shows no acute infiltrate. Which of the following is the recommended intervention?
Supportive care dx= Acute bronchitis is a lower respiratory tract Pharmacologic medications for the cough include guaifenesin, dextromethorphan, benzonatate, and codeine Throat lozenges, hot tea, honey, and smoking cessation help improve the cough. Acute Bronchitis Patient presents with a productive cough for > 5 days Most commonly caused by viruses Treatment is symptomatic management Most common cause of minor hemoptysis Routine Abx therapy not indicated
A 23-year-old woman presents to the office with increasing fatigue and inability to concentrate. She has been gaining a lot of weight in her face and abdomen and has had multiple yeast infections over the past year. The patient reports no history of exogenous steroid use. A physical exam reveals some purple striae around her abdomen and thighs. She also has very thin extremities. Initial testing reveals elevated cortisol levels and no suppression with low-dose dexamethasone suppression test. Which of the following further tests will confirm the diagnosis of Cushing disease?
Suppression with high-dose dexamethasone suppression test Opportunistic bacterial or fungal infections are common, as well as impaired wound healing and easy bruising. This syndrome is also characterized by psychological symptoms, such as inability to concentrate, mood lability, and even frank psychosis What drugs increase the metabolism of dexamethasone leading to impaired cortisol suppression? Answer: Antiseizure drugs (e.g., phenytoin, phenobarbital) and rifampin.
What is the definitive treatment for rectocele?
Surgical correction with posterior colporrhaphy. Rectocele History of childbirth, trauma, previous surgeries PE will show a vaginal bulge at posterior vaginal wall or anterior rectum wall Most commonly caused by weak pelvic muscles Management includes managing constipation (high-fiber diet), pessary device, and surgery when conservative measures fail
A 65-year-old man with a history of hypertension, hypercholesterolemia, and a 40 pack-year smoking history presents to his primary care provider 1 week after stent placement during a percutaneous transluminal coronary angioplasty. He states he has been feeling well since the procedure. He also reports he lost his postvisit summary and is unsure of any medication changes he should be aware of. What medication should be initiated in this patient? ADabigatran BDipyridamole CHeparin DTicagrelor
Ticagrelor (look at image ) Patients who recently had a stent placed should be prescribed dual antiplatelet therapy with aspirin and a P2Y12 inhibitor (e.g., ticagrelor or clopidogrel).
A 25-year-old woman presents with patches of skin discoloration on her chest and abdomen that she noticed about 2 weeks ago. She does not report pain or itching to the skin. Upon physical exam, there are scattered hypopigmented, velvety, white, oval-shaped macules around the patient's trunk. A topical scraping examined under potassium hydroxide preparation reveals short hyphae with spores. Which of the following is the most likely diagnosis?
Tinea versicolor also known as pityriasis versicolor, is a fungal infection characterized by the presence of hypo- or hyperpigmented macules and patches that are typically limited to the trunk Tinea Versicolor Patient presents with hypopigmented areas that do not tan PE will show scaly patches on the chest and trunk Diagnosis is made by KOH preparation of skin scraping Most commonly caused by Malassezia species (e.g., M. furfur) Treatment is topical selenium sulfide 2.5% or topical ketoconazole Which of the following oral antifungals is no longer recommended as first-line oral treatment due to the risk of drug-induced hepatitis? Answer: Ketoconazole. ====================================================== Vitiligo (D) is characterized by the presence of well-circumscribed, depigmented macules and patches that are neither painful nor pruritic. This disease is associated with certain autoimmune conditions, particularly thyroid abnormalities. The patches are not limited to the trunk but typically affect the face, neck, arms, feet, fingers, and scalp.
A 70-year-old man presents to his primary care provider to discuss his chronic obstructive pulmonary disease. His last pulmonary function test revealed FEV1 of 60%. He has not had any exacerbations in the past year, however, he reports an increase in symptoms in the past 2 months. He is coughing more, has noticed an increase in sputum production, and is becoming breathless with minimal exertion. He currently only uses ipratropium as needed. What medication should be initiated next for this patient?
Tiotropium dx: COPD What antibiotic used for COPD exacerbations has been shown to have anti-inflammatory properties in the lungs? Answer: Azithromycin. Class A patients experience minimal symptoms, have mild to moderate airflow obstruction, and have had ≤ 1 exacerbation in the past 12 months. Short-acting bronchodilators, such as ipratropium, should be used as needed. Class B patients are highly symptomatic, have mild to moderate airway obstruction, and have had ≤ 1 exacerbation in the past 12 months. Class B patients should be on a short-acting bronchodilator and a long-acting bronchodilator, such as tiotropium (long acting muscarinic antagonist, LAMA) or salmeterol (long acting bronchodilator, LABA). Class C patients have minimal symptoms but severe to very severe airway obstruction and ≥ 2 exacerbations in the past 12 months. Class C patients should be prescribed a short-acting bronchodilator and a long-acting bronchodilator. An inhaled corticosteroid may be added to the regimen if needed. Class D patients are highly symptomatic, have severe to very severe airway obstruction, and have had ≥ 2 exacerbations in the past 12 months. Class D patients are treated the same as class C patients, with the addition of oxygen supplementation if they show signs of cor pulmonale or their O2 saturation is less than 88%.
A 25-year-old woman presents with a rash for the last 2 weeks. She reports it started with an approximately 2 cm oval salmon-colored patch on her anterior abdomen that began to clear about 1 week before more lesions appeared on her trunk. Lesions are pruritic. On physical exam, the patient has multiple oval-shaped salmon-colored patches with a collarette of scale over her trunk that follow the lines of cleavage in the skin. An image of her rash is shown above. Physical exam is otherwise negative. Which of the following is the best therapy given the suspected diagnosis?
Topical corticosteroid dx: Pityriasis rosea papulosquamous eruption proximal areas of the extremities and on the trunk. fall or spring smaller salmon-colored oval patches with a collarette of scale lesions are orientated along the lines of cleavage of the skin and are most evident on the back ("Christmas tree" distribution).
A 20-year-old woman presents to her primary care provider with complaints of severe right eye pain after trying to remove her contact lens. She reports she is very sensitive to light and feels like there is something stuck in her eye. Visual acuity testing with a Snellen chart is unchanged from her baseline. Instillation of topical proparacaine 0.5% solution results in immediate relief. Fluorescein staining shows the finding in the image above. Which of the following treatments is recommended for this patient? A 25-year-old woman presents to the emergency department with a painful red eye. She wears contact lenses and does not always take them out at night. On exam, her right eye is injected with purulent drainage and a central and round 3 mm corneal opacity. Her visual acuity is decreased, and the opacity takes up fluorescein stain. Which of the following topical agents is the recommended treatment? when is it time to see ophthalmologist ?
Topical gentamicin or Moxifloxacin drops corneal abrasion secondary to a contact lens injury and should be treated with topical gentamicindrops for Pseudomonas coverage contact lens wearers Physical examination is often difficult without the installation of a topical anesthetic (e.g., proparacaine) due to severe pain. Installation of the anesthetic should result in immediate pain relief and allows the clinician to perform the subsequent examination. Topical antibiotic therapy with erythromycin ointment is indicated for patients who do not wear contact lenses. Contact lens wearers need antibiotic coverage against Pseudomonas species and should be treated with a topical fluoroquinolone (e.g., ofloxacin, ciprofloxacin) or aminoglycoside (e.g., tobramycin, gentamicin). Patching the eye is contraindicated if a foreign body is unable to be removed or if the patient is a contact lens wearer. Nonsteroidal anti-inflammatory drugs may be given for mild to moderate pain control. Severe pain secondary to large corneal abrasions may be treated with cycloplegic drops (e.g., cyclopentolate) or a short course of oral opioids. Most corneal abrasions heal regardless of treatment within 72 hours A large epithelial defect, purulent discharge, drop in visual acuity of more than one to two lines on a Snellen chart, or a corneal abrasion that has not healed after 72 hours may indicate a retained foreign body, poor healing, or infection and warrants an urgent ophthalmology referral. In addition, the presence of a corneal epithelial defect with a corneal infiltrate or ulcer represents an area of infiltration and warrants a same-day ophthalmology evaluation. Which screening test for corneal disorders is used to evaluate for the presence of anterior chamber leakage into the cornea? Answer: Seidel tes
28-year-old woman presents to the office with severe episodic headaches that have interfered with her attendance at work for the past 3 months. She reports having similar headaches when she was a teenager but at a milder severity. The headache is described as throbbing with associated nausea and sometimes vomiting. She is often sensitive to light during an attack. Headaches have occurred at least seven times in the past month. She has been taking over-the-counter naproxen on a regular basis for relief. She would like something that prevents the headaches altogether but is concerned about potential weight gain. Which of the following is an appropriate first-line treatment for this patient?
Topiramate is an appropriate first-line therapy for patients with episodic migraine who would benefit from prophylaxis. Migraine headache occurs due to primary neuronal dysfunction, which results in cortical spreading depression, activation of the trigeminovascular system, and increased neuronal sensitivity. What are common side effects of topiramate? Answer: Paresthesias, anomia, weight loss, anorexia, metabolic acidosis, and nephrolithiasis. {look at image ] The headache in a migraine is often unilateral and is described as throbbing or pulsatile. Severity increases over hours. Phonophobia, photophobia, and osmophobia are common. Additionally, nausea is frequent and may be accompanied by vomiting. Acute migraine treatment is dependent on the severity of attacks and accompanying features (nausea, vomiting). Simple analgesics, such as naproxen or acetaminophen, are often first-line agents. Patients should be cautioned on the risk of medication-overuse headache. For moderate to severe attacks, a combination of antiemetics and oral, subcutaneous, or nasal triptans may be used.
Which of the following sets of parameters is most consistent with recommended levels on the lipid panel of a healthy person?
Total cholesterol < 200 mg/dL, HDL > 60 mg/dL, LDL < 100 mg/dL, triglycerides < 150 mg/dL What disorder is a severely elevated triglyceride level (> 1,000 mg/dL) a risk factor for? Answer: Pancreatitis.
Typical pneumonia vs Atypical pneumonia
Typical pneumonia, such as that caused by Streptococcus pneumoniae, Haemophilus influenzae, or Klebsiella pneumoniae, presents with a sudden onset of fever, productive cough, pleuritic chest pain, and rigors. The physical exam of a patient with typical pneumonia will reveal signs of consolidation, such as bronchial breath sounds, dullness to percussion, increased tactile fremitus, and egophony. The chest X-ray will reveal an area of consolidation in typical pneumonia. vs Atypical pneumonia, such as that caused by Mycoplasma pneumoniae, Chlamydophila pneumoniae, or Legionella pneumophila, presents with a low-grade fever, nonproductive cough, and extrapulmonary symptoms such as myalgia, sore throat, headache, and nausea. Patients with atypical pneumonia will commonly have a normal physical exam. Abnormal laboratory values may include an elevated white blood cell count, elevated blood urea nitrogen, and elevated procalcitonin. Findings on a chest X-ray for atypical pneumonia are usually nonspecific.
A 22-year-old woman presents to the clinic complaining of a lump in her right breast. Vital signs are normal, and she takes no medications. On physical exam, a firm, semimobile, round, nontender mass is palpable at 8 o'clock. No nipple discharge or skin changes are noted, but a firm, nontender ipsilateral axillary lymph node is present. Which of the following clinical interventions is the best first choice for this patient?
Ultrasound of the right breast What is the best next diagnostic step in a patient with a mammographically proven suspicious breast lesion? Answer: Breast biopsy.
A 21-year-old woman presents to an obstetrician after a positive home pregnancy test. She states she has a history of irregular menses and her last menstrual period was 6 weeks ago. Her beta-human chorionic gonadotropin is 46,000 mIU/mL. A transvaginal ultrasound is shown above and estimates a 7 week gestational age. Which of the following tests should always be ordered at this time? AAlpha-fetoprotein BEstradiol CInhibin A DUrinalysis
Urinalysis intrauterine pregnancy prenatal visits every 4 weeks from 4-28 weeks gestation, every 2 weeks from 28-36 weeks gestation, and weekly after 36 weeks gestation. At each visit, weight, blood pressure, fetal heart rate, and a urinalysis for infection, protein, and glucose should be obtained and the presence of fetal movement or vaginal bleeding evaluated. Fundal height is typically measured at each visit beginning at 20 weeks gestation. Alpha-fetoprotein (A), estradiol (B), inhibin A (C), and human chorionic gonadotropin are tests that comprise the maternal quad screen, which may be performed at 16-20 weeks gestation. At what point of gestation is chorionic villus sampling typically performed? Answer: 10-13 weeks gestation.
Q110 A 21-year-old man presents to the emergency department via ambulance after his mother found him seizing early this morning. The patient's mother reports she heard a choking sound coming from her son's room around 4:00 AM. She reports his arms and legs were jerking for at least 1 minute, and he was not responsive. He urinated on himself. In the emergency department, he appears to be postictal, and the oropharynx examination reveals a tongue laceration. Toxicology screen is negative, and finger stick blood glucose is within normal range. Upon further questioning, the patient's mother reports he had a similar episode 2 years ago, but it was determined he did not need any further intervention at that time. Which of the following is the preferred pharmacologic intervention, given the most likely diagnosis?
Valproate is the first-line drug choice for generalized epileptic seizures. Epilepsy is diagnosed when a patient has at least two unprovoked seizures that occur more than 24 hours apart. For generalized epilepsy, valproate, lamotrigine, and levetiracetam are appropriate first-line options. Ethosuximide is recommended for absence seizures. For focal seizure disorders, lamotrigine, oxcarbazepine, and phenytoin are appropriate first-line options. Carbamazepine and phenytoin can worsen some types of generalized seizures. In addition to pharmacologic therapy, driving restrictions and seizure precautions should be reviewed with the patient.
A 30-year-old man presents with right eye pain and swelling. The patient reports that his symptoms have been worsening over the past 3 days. Physical exam reveals lid erythema, pain with extraocular movements, sluggish right pupil response, and visual acuity of 20/40. He reports a history of an upper respiratory infection 2 weeks ago. The patient has no known drug allergies and no medications. CT scan of the head shows inflammation of the extraocular muscles with fat stranding. No abscess or intracranial involvement is noted. What is the most appropriate intervention for this patient? AMetronidazole BTrimethoprim-sulfamethoxazole CVancomycin DVancomycin and ceftriaxone vs with intracranial involvement?
Vancomycin and ceftriaxone dx Orbital cellulitis is an infection involving the fat and ocular muscles in the orbit but not the globe. present with pain, lid erythema, restricted and painful eye movements, decreased visual acuity, afferent pupillary defect, fever, proptosis, and conjunctival chemosis. An ophthalmologist and an otolaryngologist should be consulted. Treatment is broad-spectrum intravenous antibiotic therapy that includes coverage for methicillin-resistant Staphylococcus aureus. Initial treatment should be aggressive to prevent massive proptosis, meningitis, blindness, or septic cavernous sinus thrombosis. Intravenous vancomycin combined with ceftriaxone or cefotaxime is the recommended initial treatment in a patient without intracranial involvement. For patients with intracranial involvement or evidence of rhinosinusitis, metronidazole is added for anaerobic coverage. Improvement should be seen within 24 to 48 hours of initiating appropriate therapy. Once the patient is afebrile and eyelid and orbit findings have substantially improved, patients can be transitioned to oral therapy as indicated. In patients with poor response to treatment, surgical biopsy may be indicated. Surgical drainage is also indicated for abscesses larger than 10 mm, those that fail to respond to antibiotic therapy, or those that threaten vision. The majority of patients respond rapidly to appropriate therapy. What anatomical structure is infected in preseptal cellulitis? Answer: Anterior portion of the eyelid.
A 52-year-old woman with a past medical history of Raynaud phenomenon presents to the clinic with sudden-onset chest pain, shortness of breath, and anxiety that began 15 minutes ago while she was sleeping and lasted approximately 3 minutes. At the time of this visit, the pain has resolved. Current vital signs are BP 122/80 mm Hg, HR 118 beats per minute, RR 28 breaths per minute, and oxygen saturation 96% on room air. ECG is unremarkable, and serial troponin testing is negative. Which of the following is the most likely diagnosis?
Vasospastic angina The typical patient affected by vasospastic angina is a woman, > 50 years old, and a smoker and often has other vasospastic disorders as well, such as migraines or Raynaud phenomenon. Symptoms include chest pain that often occurs at rest or occurs in the early morning and could be brought on by cold weather or emotional stress. ECG may show transient ST elevations in several leads during an episode of angina but may be otherwise normal. Serial troponins are negativeunless a true MI is present. Vasospastic Angina (Prinzmetal Angina) History of HTN, smoking, DM, obesity, or cocaine use Squeezing, pressure-like chest discomfort at rest ECG will show transient ST segment elevations Cardiac enzymes will be normal Diagnosis is made by cardiac stress test Most commonly caused by coronary artery spasm Treatment is calcium channel blockers and nitrates
A 40-year-old man presents to his primary care provider for an annual physical. He has no significant past medical history and has no concerns today. His blood pressure is 140/80 mm Hg, and all other vitals are within normal limits. Physical exam reveals a man who is obese with a waist circumference of 115 cm. What other value would qualify this patient for metabolic syndrome?
What dietary supplement acts like a low-intensity statin? Answer: Red yeast rice.
A 25-year-old man who is morbidly obese presents to his primary care provider for a routine visit. A lipid profile is drawn and is significant for a triglyceride level of 450 mg/dL. Which of the following is most likely to be seen on physical examination?
Xanthelasmas Triglyceride levels > 1,000 mg/dL are associated with pancreatitis.
A 31-year-old man presents to his primary care provider with complaints of back pain for the past 2 weeks. Which of the following is considered an alarming symptom associated with malignancy?
alarming symptoms. Unexplained weight loss, failure to improve with treatment, pain for more than 6 weeks, or pain at rest or night What response with plantar stimulation is a positive Babinski sign in adults characterized by? Answer: Dorsal extension and fanning of the toes.
A 14-year-old girl presents to the office due to a 1-week history of pain, redness, and swelling to the right middle finger adjacent to the lateral nail. She states she had a hangnail there that she ripped off about 1 week ago, and a few days later, she developed pain and swelling to the area. Upon physical examination, there is erythema, moderate edema, and exquisite tenderness to palpation of the lateral aspect of the nail, with noticeable fluctuance and purulence. Which of the following is the best next step in treatment for this patient?
mmediate incision and drainage dx: Acute paronychia Most commonly caused by S. aureus Treatment is Abx, warm soaks, I&D A Gram stain or culture can be done to help identify the bacterium involved, and a potassium hydroxide smear can help rule out candidal infection. If there is soft tissue swelling without fluctuance, the area can be treated with topical antistaphylococcal antibiotics (e.g., triple antibiotic ointment or mupirocin), warm compresses, warm water or antiseptic soaks (e.g., chlorhexidine, povidone-iodine) for 10-15 minutes multiple times a day. Over-the-counter analgesics may also be used for controlling pain.
1)What is alopecia universalis? Answer: Complete hair loss on all hair-bearing areas throughout the body. 2): Which gallop heard on cardiac auscultation can be a normal finding in patients < 30 years of age? Answer: S3 gallop. 3)What is the name of the fibroadenoma-like tumor with cellular stroma that grows rapidly and has a leaf-like pattern on histology? Answer: Phyllodes tumor. 4)Which hereditary disorder should patients with suspected COPD be evaluated for? Answer: Alpha-1 antitrypsin deficiency. 5)What four diseases are most commonly associated with HLA-B27 positivity? Answer: Psoriatic arthritis, ankylosing spondylitis, inflammatory bowel disease, and reactive arthritis (noted by the mnemonic PAIR). 6)What is the main organ responsible for the conversion of T4 to T3? Answer: The liver. 7)What is Cheyne-Stokes breathing? Answer: An abnormal pattern of breathing characterized by periods of tachypnea and hyperpnea alternated by periods of apnea. 8)What is the recommended approach for women aged 25 and older with low-grade squamous intraepithelial lesions and a positive human papillomavirus test? Answer: Colposcopy. 9)A lesion in which vessel is indicated by ST elevations in leads V1, V2, V3, and V4? Answer: Left anterior descending (LAD). 10)True or false: aspirin is useful for the prevention of stroke due to atrial fibrillation. Answer: False. 11)What sign is characterized by referred left shoulder pain and associated with splenic rupture? Answer: Kehr sign. 12)What are side effects of tadalafil? Answer: Headache, abdominal pain, muscle pain, stuffy nose, and flushing. 13)What are common side effects of topiramate? Answer: Paresthesias, anomia, weight loss, anorexia, metabolic acidosis, and nephrolithiasis. 14)Which reflex originates in the cardiac sensory receptors and promotes parasympathetic activity, resulting in bradycardia, vasodilation, and hypotension? Answer: The Bezold-Jarisch reflex. 15)What disorder is Zollinger-Ellison syndrome closely associated with? Answer: Multiple endocrine neoplasia type 1. 16)What is holiday heart syndrome? Answer: An irregular heart rhythm that presents in an otherwise healthy individual after consuming excessive amounts of alcohol. 17)What are the three other causes of midsystolic murmurs excluding innocent murmurs of childhood? Answer: Hypertrophic cardiomyopathy, aortic stenosis, and pulmonic stenosis. 18)What is Löfgren syndrome? Answer: Combination of erythema nodosum, hilar adenopathy, migratory polyarthralgia, and fever observed in patients with sarcoidosis.
q/A 8
1)What type of diarrhea does ingestion of polyethylene glycol 3350 induce? Answer: Osmotic. 2)What is the most common cause of roseola? Answer: Human herpesvirus 6. Shingles encephalitis is dangerous because it can cause seizures 3) What is the first-line treatment for primary dysmenorrhea? Answer: Nonsteroidal anti-inflammatory drugs (NSAIDs). or acetaminophen 4)What surgical procedure is used as definitive treatment for angle-closure glaucoma? Answer: Peripheral iridotomy. 5)What are two factors that may reduce breast cancer risk? Answer: Breastfeeding and increased regular physical activity 6)What diagnosis is the blue dot sign associated with? Answer: Torsion of the appendix of the testicle. 7)True or false: adult patients should be screened for hypertriglyceridemia. Answer: False. 8)Which antibiotic is used to treat confirmed pertussis infections? Answer: Macrolides, such as azithromycin. 9)What are the risk factors for methicillin-resistant Staphylococcus aureus? Answer: Intravenous drug use, residence in a long-term care facility, discharge from a hospital within the preceding 12 months, and hemodialysis. 10)What is the name of the radiographic finding that describes the metaphyseal triangular portion in a Salter II fracture? Answer: Thurstan Holland fragment.
q/a 1
1)What type of stone is associated with the development of staghorn calculi? Answer: Struvite stones. 2)What antibody is associated with pernicious anemia? Answer: Autoantibodies to intrinsic factor (anti-IF). 3)What is a septic abortion? Answer: Retained products of conception have not been fully expelled, and endometritis develops 4)What is the first-line treatment for bipolar disorder in pregnant patients? Answer: Lamotrigine. 5)Which of the following oral antifungals is no longer recommended as first-line oral treatment due to the risk of drug-induced hepatitis? Answer: Ketoconazole. Pityriasis alba (A) is a benign, self-limited, and common skin condition in children characterized by the presence of hypopigmented macules and patches on the face. It is thought to be related to atopic dermatitis. Hypopigmented areas may be preceded by areas of scaling and erythema. 6)What medical term is used to describe the pain that occurs after a period of rest and is associated with plantar fasciitis? Answer: Poststatic dyskinesia. 7)Which RIPE antibiotic (rifampin, isoniazid, pyrazinamide, and ethambutol) is associated with higher rates of hepatitis? Answer: Isoniazid. 8)What is torticollis? Answer: Lateral contraction of the neck, causing the head to tilt to one side with the chin rotated in the opposite direction. 9)What is the appropriate second dose of amiodarone after the initial 300 mg dose, according to the advanced cardiac life support algorithm? Answer: 150 mg. 10)What are the most common somatic symptoms associated with generalized anxiety disorder? Answer: Headaches and pain in the neck, shoulders, or back. The generalized anxiety disorder seven-item scale (GAD-7) is typically used in primary care to identify patients who may have the disorder and is used as a screening tool. The diagnosis is made when an individual finds it difficult to control worry and when the worry is associated with three of the following: restlessness, easy fatigue, difficulty concentrating, irritability, muscle tension, and sleep disturbance. 11)What is the mechanism of action of clopidogrel? Answer: Irreversible blockage of ADP receptors on the platelet surface, thus inhibiting platelet aggregation. 12) Which surgical procedure is recommended in a patient who is morbidly obese and has symptoms of gastroesophageal reflux disease? Answer: Bariatric surgery. 13)What are the possible adverse effects of using sodium phosphate enemas in older adults? Answer: Volume depletion, hypotension, hyperphosphatemia, hypokalemia, hypocalcemia, metabolic acidosis, and acute kidney injury. 14)When might endoscopy or surgery be indicated in a toxic ingestion? Answer: When the substance ingested is a large amount of heavy metals, a pharmacobezoar, or an illicit drug packet. 15)Which screening test for corneal disorders is used to evaluate for the presence of anterior chamber leakage into the cornea? Answer: Seidel test 16)What is the second most common subtype of melanoma? Answer: Nodular melanoma. Superficial spreading is the most common subtype of melanoma. 17)What is Melkersson-Rosental syndrome? Answer: A rare condition characterized by recurrent episodes of facial swelling and facial paralysis in patients with a fissured tongue. 18)What is the classic finding on fluorescein stain for herpes simplex keratitis? Answer: Dendritic branches. 19)What dietary supplement acts like a low-intensity statin? Answer: Red yeast rice. 20)Which antibiotic is used for the treatment of diarrhea due to Clostridioides difficile? Answer: Oral vancomycin.
q/a 2
1)What assessment tool is used to determine high-risk pneumonia patients? Answer: CURB-65 (confusion, urea nitrogen, respiratory rate, blood pressure, and 65 years or older). 2)Which patients with primary hypertension should not increase dietary potassium intake to lower blood pressure? Answer: Patients on medications that reduce potassium excretion and patients with chronic kidney disease. 3)What drug class is considered first-line pharmacotherapy for dyslipidemia that is recalcitrant to lifestyle modifications? Answer: Statins. 4)What type of hypersensitivity reaction is a transfusion reaction due to ABO incompatibility classified as? Answer: Type II hypersensitivity reaction. 5)What are the differential diagnoses of a pulmonary nodule? Answer: Infectious granuloma, bronchogenic carcinoma, hamartoma, malignancy, and bronchial adenoma. 6)Which guidelines are used in patients with Lynch syndrome-associated colorectal cancers to determine the need for tumor testing for microsatellite instability? Answer: The revised Bethesda guidelines. 7)Is cryptorchidism more likely to occur on the right or left side? Answer: Right side. 8)What is a comorbid disorder that commonly occurs with panic disorder and is characterized by the avoidance of situations where it may be difficult to leave? Answer: Agoraphobia 9)What steroid-sparing agents are appropriate for chronic immune thrombocytopenia in children? Answer: Rituximab, thrombopoietin receptor agonists (romiplostim, eltrombopag), azathioprine, and mycophenolate mofetil. 10)What type of bacteria is the most common cause of acute prostatitis? Answer: Gram-negative rods, specifically Escherichia coli. 11)What type of ventilatory pattern is seen on spirometry among patients with asthma? Answer: Obstructive. 12)What is the location of a Pancoast tumor? Answer: The superior sulcus of the lung. 13)What drug is used for the treatment of Pneumocystis pneumonia? Answer: Trimethoprim-sulfamethoxazole. 14)True or false: nitroglycerin is contraindicated in right ventricular infarction. Answer: True. 15)What are some cardiac dysrhythmias that may be seen on electrocardiogram in patients with syncope? Answer: Brugada syndrome, short QT syndrome, long QT syndrome, preexcitation, and heart block. 16) What is seen on coronary angiography in vasospastic angina? Answer: Coronary vasospasm with IV ergonovine or acetylcholine administration. 17)Which antibody is associated with drug-induced lupus erythematosus? Answer: Antihistone antibody. 18)What is the mainstay of therapy for salicylate poisoning?ASA Answer: Alkalinization with sodium bicarbonate. 19)What radiographic findings are consistent with chronic obstructive pulmonary disease? Answer: Flat diaphragm, increased retrosternal lung space, rapidly tapering vasculature, and bullae. 20)True or false: montelukast is effective in treating exercise-induced asthma. Answer: True.
q/a 3